[ACC Entry] Does The Education System Adequately Serve Advanced Students?

[This is an entry to the Adversarial Collaboration Contest by TracingWoodgrains and Michael Pershan (a k-12 math teacher), on advanced students in the education system]


“What do America’s brightest students hear? Every year, across the nation, students who should be moved ahead at their natural pace of learning are told to stay put. Thousands of students are told to lower their expectations, and put their dreams on hold. Whatever they want to do, their teachers say, it can wait.” A Nation Deceived, p.3

“There is an apparent preference among donors for studying the needs and supporting the welfare of the weak, the vicious, and the incompetent, and a negative disregard of the highly intelligent, leaving them to ‘shift for themselves.'” – Hollingworth, 1926

1. Eager to Learn and Underachieving

Pretend you’re a teacher. With 25 students, who gets your attention during class?

There’s the kid who ask for it, whose hand is constantly up. There’s also the quiet kid in the corner who never says a word, but has been lost in math since October, who will fail if you don’t do something. There’s the student in the middle of the pack, flowing along. Finally, there’s the kid who finishes everything quickly. She’s looking around and wondering, what am I supposed to do now?

In a survey of teachers from 2008, just 23% reported that advanced students were a top priority for them, while 63% reported giving struggling students in their classes the most attention. A 2005 study found the same trend in middle schools, where struggling students receive the bulk of instructional modification and special arrangements. This was true even while 73% agreed that advanced students were too often bored and under-challenged in school. While teachers, it seems, are sympathetic to the smart bored kid, that’s just not a priority for them.

This isn’t to blame teachers who are under all sorts of pressure to carry low-performing students over the threshold and who, in any event, are only trying to do what’s best for their kids. Which is the most urgent concern? If you don’t equip a kid with the skills they need, next year’s class might be a disaster for them. Or maybe they’ll fail out of school. And behavior problems? Often those begin with academic struggles. Gifted children, on the other hand — they’re on the way to becoming gifted adults. They can take care of themselves, for a minute, the logic goes. More often than not, the teacher will encourage the early finisher to go read a book, or start homework, or do anything at all while the teacher works to help the quiet, lost kid in the corner.

If the kids are just a little bored, that’s nothing strange. It’s hard to find someone who wasn’t bored in school sometimes. For many top students, already poised for achievement, this turns out just fine. And yet, there are persistent stories of how the lack of challenge can turn into something more serious.

One version of the story goes like this: from a young age, a student finds the work in school easy. It doesn’t take long for them to expect school to be easy for them — it becomes a point of pride. Over years of floating through school, an identity takes hold. Then, one day, maybe after years of schooling, something finally becomes challenging for the student… but there’s nothing nice about this challenge. The challenge is now a threat. The student begins to find school challenging, and their world falls apart. They feel isolated and misunderstood at school. They lash out. They hate it, and they can’t wait to get out.

When we asked Reddit users and blog readers to describe their experience of school, we heard versions of this story:

  • Miserable waste of time, was almost never offered opportunities to learn. Largely ignored teachers and read books during class. I felt like it was a profound injustice that I was punished for doing so. I now have kids of my own and will be home-schooling them.
  • I was bored. The pace was too slow and work was not interesting. Being forced by law to get up early and go somewhere to learn things I already know means permanent and firm dislike.
  • I went to local public schools for kindergarten through high school, and the experience wasn’t good. Academically, the classes were slow and poorly taught. Even the AP classes were taught at the speed of the slowest student, which made the experience excruciating. The honors and regular classes were even worse: I was consistently one or more grades ahead of the rest of the class in every non-AP class except honors math. I learned not to bother studying or doing homework even in the AP classes which probably wasn’t great for my work ethic.

The stories of student pain and underachievement in school get more intense as we consider cases of extremely precocious children. The pressures on the student increase, and without help a student often experiences isolation from their peers and a whole other host of difficult feelings. Miraca Gross studied students like these in Australia and found that precocious students were often suffering in silence. Speaking particularly about precocious students who underachieve, she writes:

The majority of the extremely gifted young people in my study state frankly that for substantial periods in their school careers they have deliberately concealed their abilities or significantly moderate their scholastic achievement in an attempt to reduce their classmates’ and teachers’ resentment of them. In almost every case, the parents of children retained in the regular classroom with age peers report that the drive to achieve, the delight in intellectual exploration, and the joyful seeking after new knowledge, which characterized their children in the early years, has seriously diminished or disappeared completely. These children display disturbingly low levels of motivation and social self-esteem. They are also more likely to report social rejection by their classmates and state that they frequently underachieve in attempts to gain acceptance by age peers and teachers. Unfortunately, rather than investigating the cause of this, the schools attended by these children have tended to view their decreased motivation, with the attendant drop in academic attainment, as indicators that the child has “leveled out” and is no longer gifted.

What do we make of these stories? How common are such experiences?

From the literature on “gifted underachievement” we get partial confirmation — underachievement is a real phenomenon, supported by numerous case studies. According to a survey of various school practitioners, underachievement is the top concern when it comes to gifted students. By definition, advanced students are only a small percent of each student body, so few are affected in any given place, but on a national scale it becomes a more serious problem.

This is not just a problem for the affluent. It has persistent impacts on Black students, poor students, and students who are learning English, who are less often recommended for gifted programs or special accommodations. Here’s one way this manifests itself: in one study, 44% of poor students identified as gifted in reading in 1st Grade were no longer academically exceptional by 5th Grade. For higher-income families, only 31% of 1st Graders experience this slide.

The lack of attention to this group extends to the research. It’s difficult to pin down the number of students impacted. While underachievement is a real phenomenon, current research doesn’t tell us very much about the factors contributing to gifted underachievement. What studies have been done tend to focus almost entirely on things like whether students with ADHD or unsupportive families underachieve, rather than looking at controllable factors like the sort of teaching students experience in school.

Schools are the institutions in charge of educating kids. Those who rush into school, eager to learn, should not walk out feeling rebuffed and ignored. This is doubly true for talented kids from at-risk populations, who may not have the support structure outside of school to ensure their success if school has no time for them. It’s clear, though, that we cannot degrade the experience of other students to help those who already have an academic leg up. Is there a feasible approach to address this problem without making things worse?

We have good reason to think that personalized attention makes a huge difference to a student’s learning. Research suggests that tutoring that supplements a student’s coursework is a very effective educational intervention. Benjamin Bloom caught people’s attention with the idea of a 2 standard deviation effect in the 1980s. More recent research has lowered that sky-high estimate to more realistic numbers, and a meta-analysis found an effect size of 0.36, still a powerful impact, enough to take a student from the 50th percentile of achievement to the 64th.

If supplemental tutoring works, the dream goes, what if we replaced classroom work entirely with tutoring? Can’t we just do that for gifted underachievers and precocious students? We have tantalizing success stories of this kind in the education for precocious children. In a famous case, John Stuart Mill‘s father decided that the philosophy of utilitarianism needed an advocate, and planned a demanding course for him. Mill didn’t underachieve: he learned Greek at age 3, Latin at age 8, and flourished as a philosopher. László Polgár declared he had discovered the secret of raising “geniuses” and went about showing it by tutoring his daughters in chess from the age of 3. It’s hard to argue with his results: two grandmasters and an international master, one of whom became the 8th ranked chess player in the world and the only woman ever to take a game off the reigning world champion.

Though this sort of tutoring seems like a dream come true for underachieving gifted students, in practice it’s a non-starter in schools. (It lives on in homeschooling, to an extent). In a world where schools are struggling to help every kid learn to read, the ethics of only assigning tutors to gifted students is dubious and almost certainly a political impossibility. The cost of assigning a tutor to every child, meanwhile, would do something special to property taxes. This simple answer, then, can lead to a clearer understanding of the complexity of educational questions: It’s possible to focus on simple practices that work while disregarding nonacademic concerns and political feasibility.

To be useful, educational ideas should be effective, politically feasible, and economical. If tutoring for gifted underachievers isn’t workable, might there be some other way to approximate the benefits of personal, human attention? Here are three of the most common tools that advocates for gifted education propose:

What follows is an evaluation of how promising each of these tools is, both in theory and in practice.

Our favorite one-stop reading on gifted education research: this.

Our favorite one-stop reading on tutoring: this.

2. Ability Grouping (a.k.a Tracking)

The case for placing students of similar abilities together in a classroom seems like it ought to be as simple as the case for tutoring. Teachers will be more effective if their students have similar pacing needs. So, group kids who need more time in one class and those who need less time in another. It’s not tutoring, but it should be the next best thing.

Things in education research are rarely that simple, though.

Bob Slavin, a psychologist who studies education, is one of the most-cited education researchers around. He seems like a compulsively busy fellow. He writes, he runs research centers, he designs programs for schools. (He blogs.) A journalist from The Guardian once asked Slavin for his likes and dislikes, and in case you were wondering he likes work and dislikes complacency.

In the late ’80s and early ’90s, Slavin performed a series of meta-analyses of the existing literature on tracking and between-class ability grouping. Overall, he found no significant benefits from ability grouping, even for “top track” students across elementary, middle, and high schools.

But the other surprising finding of Slavin’s was that nobody was academically hurt by ability grouping — not even the lowest track students. Slavin argued that when you consider all the non-academic concerns, the scales weigh in favor of detracking, i.e. avoiding ability grouping.

What are those non-academic concerns? In the conclusion of his review of the evidence from elementary schools, he writes:

“Ability grouping plans in all forms are repugnant to many educators, who feel uncomfortable making decisions about elementary-aged students that could have long-term effects on their self-esteem and life chances. In desegregated schools, the possibility that ability grouping may create racially identifiable groups or classes is of great concern.” (p.327)

That’s Slavin’s view. So, where is the debate?

One thing that is decidedly not up for debate in the literature is that Slavin’s non-academic concerns are real. Opponents and defenders of tracking alike agree that low-track classes are often chaotic, poorly taught environments where bad behavior is endemic, and that this is a major problem. Tom Loveless is a contemporary defender of tracking, and writes that “even under the best of conditions, low tracks are difficult classrooms. The low tracks that focus on academics often try to remediate through dull, repetitious seatwork.” Jeannie Oakes made a name for herself by carefully documenting the lousiness of a lot of low track classes.

Some tracked schools seem to have done better with their low tracks. Gamoran, an opponent of tracking, speaks highly of how some Catholic schools handle lower tracks. Gutierrez identifies several tracked schools with strong commitments to helping students across the school advance in mathematics, and concludes that “tracking is not the pivotal policy on which student advancement in mathematics depends.” Making these experiences better is an important goal. These difficult dynamics are a genuine and widespread issue, though, and educators are rightly concerned about them.

Slavin’s concerns about exacerbating racism in schools are relatively uncontroversial as well. It’s not so much that race is a factor in track placement. Using a large nationally representative sample and controlling for prior achievement, Lucas and Gamoran found that race wasn’t a factor in track placement. (Though Dauber et al, found that race was a factor in track placement in Baltimore schools, so maybe sometimes racism is a factor in placement.)

But because of existing achievement gaps between e.g. Black and white students, there’s the potential in a racially mixed school that ability groups will effectively sort Black students into the lowest track and expose them to a lot of dynamics that are difficult to quantitatively measure but frequently discussed in education. A school where being Black is associated with poor performance and misbehavior will, according to many educators and researchers, lead to lower expectations and academic self-esteem for all Black students.

(Good news for people who like bad news: school segregation is getting worse, so the interaction between tracking and race is getting better.)

The main controversy surrounds Slavin’s claims about the academic impact of ability grouping. His meta-analyses were part of an extended back-and-forth with Chen-Lin & James Kulik, who wrote several competing analyses on the ability grouping literature. Slavin and the Kuliks each criticized the other’s methodology, but the core point the Kuliks made was that ability grouping did have positive effects on gifted students as long as curriculum was enhanced or accelerated to match, and that this typically did happen in dedicated gifted and talented programs. The Kuliks pointed out that both they and Slavin largely agreed on the data both analyzed, but that Slavin excluded studies of gifted programs from his research while the Kuliks made those studies a focus.

Tom Loveless, senior fellow at the Brookings Institution, summarized one important aspect of their dispute, which is that their debate centers more on values than their read of the extant evidence:

Slavin and Kulik are more sharply opposed on the tracking issue than their other points of agreement would imply. Slavin states that he is philosophically opposed to tracking, regarding it as inegalitarian and anti-democratic. Unless schools can demonstrate that tracking helps someone, Slavin reasons, they should quit using it. Kulik’s position is that since tracking benefits high achieving students and harms no one, its abolition would be a mistake (p.17)

Betts notes the studies the Kuliks and Slavin reviewed in their meta-analyses had some flaws, with relatively small N and non–nationally representative data. Using more nationally representative samples, a number of researchers (Hoffer, Gamoran and Mare, Argys, Rees and Brewer) came to the conclusion that conventional tracking benefits students in the high tracks and hurts those in the low tracks. But it’s really hard to control for the right factors in these definitely non-experimental studies, and Betts and Shkolnik raise questions about the results of these papers. And there was also a recent big meta-meta-analysis that found no benefits for between-class grouping, echoing Slavin, but that did find benefits for special grouping for gifted students, echoing the Kuliks.

Just to mess with everybody, Figlio and Page argue that by attracting stronger students to the school (because parents seek tracking) students in low-tracks benefit, secondarily.

So, in summary, what should we make of all this? Betts, an economist, says in a review of the literature that when it comes to the average impact of tracking or the distribution of achievement “this literature does not provide compelling evidence.” Loveless doesn’t disagree, but notes that for high achievers, the situation is clearer:

“The evidence does not support the charge that tracking is inherently harmful, and there is no clear evidence that abandoning tracking for heterogeneously grouped classes would provide a better education for any student. This being said, tracking’s ardent defenders cannot call on a wealth of research to support their position either. The evidence does not support the claim that tracking benefits most students or that heterogeneous grouping depresses achievement. High achieving students are the exception. For them, tracked classes with an accelerated or enriched curriculum are superior to heterogeneously grouped classes.” (p.22)

At the end of the day, all academic impacts of tracking are mediated by teaching and the curriculum. If a teacher doesn’t change what they teach or how they teach it, no grouping decision will help or hurt a student academically in a significant way. Tracking only could benefit gifted students if it came with some sort of curricular modification.

This is a conclusion with wide-reaching support. Even Slavin, who so staunchly opposed conventional ability grouping, was extremely impressed by something called the Joplin Plan, which involves three core features:

  • Grouping students based on reading ability, regardless of grade level
  • Regular testing and regrouping of students on the basis of the tests
  • A different curriculum for each group of students

Slavin, the Kuliks, and everyone else seemed to agree that students in the plan — at all ability levels — tended to get 2-3 months ahead of students in typical programs over a year of instruction. The Joplin plan involves ability grouping — the good kind of ability grouping.

So in 1986, when the Baltimore School Superintendent turned to Bob Slavin to design a program that would improve the city’s most dysfunctional schools, guess how Slavin grouped students?

Slavin worked with research scientist Nancy Madden (they’re married) to design Success for All for Baltimore, and it’s a prominent program in the school improvement world, implemented in thousands of schools and spreading. Those three features of the Joplin plan — assessment, regrouping along the lines of ability and targeted teaching — are core features of their program.

Success for All isn’t the only example of a successful curriculum implementing these ideas. Direct Instruction was created by Siegfried Engelmann and Wesley Becker in the 1960s, and it also groups students according to their current levels in reading and math while frequently reassessing and regrouping. DI has a strong body of research supporting its efficacy (for one, it was the winner of the famous-in-education Follow Through experiment), but fell largely out of favor outside of remedial classrooms. In early 2018, a new meta-analysis spanning 50 years of research reinvigorated conversation around Direct Instruction. It found an average effect size of 0.51 to 0.66 in English and math over 328 studies (p<0.001), — strong evidence that the program works.

While its effect on student performance is rarely disputed, the program remains controversial. Historian of education Jack Schneider writes: “Direct Instruction works, and I’d never send my kids to a school that uses it. The program narrows the aims of education and leaves little room for creativity, spontaneity and play in the classroom. Although test scores may go up, the improvement is not without a cost.” Ed Realist worries that its pedagogy is unsavory, has not been shown to work for older students, that wealthier parents are voting with their feet against the curriculum, and that DI could exacerbate gaps between students. Supporters, by contrast, paint the picture of a robust, effective system that has been ignored and disregarded.

Success for All and Direct Instruction are not simple programs for schools to adopt. Implementing them amounts to a major organizational change, and pushes at the extremely resilient notion that children in school should be grouped by their ages. Comprehensive ability grouping programs such as these seem to work, but in practice they are rarely used.

Our favorite one-stop source for reading on ability grouping: here, or maybe here to get a broader picture of the controversy.

3. Acceleration

Forget the comprehensive approach, then. Does it work to simply move an individual student (e.g. an underchallenged and frustrated student) through the curriculum at whatever pace seems to make sense?

There are a few different ways schools can help some students access the curriculum more quickly. A kid can skip a full grade, or several grades in extreme cases. They can stay in their grade for some classes, but join higher grade levels for some parts of the day. They might be assigned to two classes in one year (e.g. Algebra 1 and Geometry). Or, in some cases, a young student might start school at an even younger age than is typical.

If a child is ready for a higher level within a subject and studies it instead of the lower level, it’s almost a given that they’ll learn more. The real research questions are (a) from an academic standpoint whether accelerated children do tend to be ready, or if they do poorly in classes post-acceleration) and (b) whether acceleration exposes students to non-academic harm (e.g. stress, demotivation, loss of love for subject, poor self-esteem).

The Study of Mathematically Precocious Youth (SMPY) is an ongoing longitudinal study examining thousands of mathematically gifted students. In one SMPY study, researchers compared the professional STEM accomplishments of mathematically gifted students who skipped a grade to those who remained at grade level. They found that, controlling for a student’s academic profile in a pretty sophisticated way, students who skipped a grade tended to be ahead of the non-skippers in terms of degrees earned, publications, citations accrued, and patents received. From this work it seems skipping a grade in the SMPY cohort did nothing to hurt a kid’s learning or enthusiasm for their passions.

Acceleration has been one of the focuses of SMPY studies. A 1993 piece about SMPY findings reported “there is no evidence that acceleration harms willing students either academically or psychosocially.” This is supported by various meta-analyses, going back to the 1984 Kulik & Kulik paper and confirmed by more recent work such as a 2011 analysis of existing studies. Beyond the “does no harm” findings, these meta-analyses also report academic benefits to students.

It can be confusing, when reading these studies, to keep track of just how gifted the students happen to be. For example, SMPY has studied five cohorts so far, ranging from students who assessed in the top 3% to those who assessed in the top 0.01%. As we consider students farther away from the mean of achievement, the need for acceleration becomes more acute.

Lots of teachers encounter “1 in 100” students every year, but the education of “off the charts” students is necessarily more a matter of feel than policy. Still, there are success stories to learn from, and they show a remarkable sensitivity to both the academic and social well-being of the student.

Terence Tao is a famous success story of this kind. He surprised his parents by discovering how to read before turning two, and as a child he started climbing through math at a blistering rate. He was identified as profoundly gifted from a young age, and his education was carefully tracked by Miraca Gross as part of her longitudinal study of profoundly gifted children:

His parents investigated a number of local schools, seeking one with a principal who would have the necessary flexibility and open-mindedness to accept Terry within the program structure they had in mind. …

This set the pattern for the ‘integrated,’ multi-grade acceleration program which his parents had envisaged and which was adopted, after much thought and discussion, by the school. By early 1982, when Terry was 6 years 6 months old, he was attending grades 3, 4, 6 and 7 for different subjects. On his way through school, he was able to work and socialize with children at each grade level and, because he was progressing at his own pace in each subject, without formal “grade-skipping,” gaps in his subject knowledge were avoided.

His education continued in much the same fashion, culminating in a Ph.D. by the age of 21 and a remarkable and balanced life since. He has since given his own advice on gifted education.

Given the success of acceleration, are we accelerating enough? On the one hand, it appears that acceleration is a widely used tool for giving gifted students what they need. When looking at the top 1 in 10000 students in terms of mathematical ability as identified by the SMPY, nearly half of the group skipped grades, and almost all of them had some form of acceleration, whether that meant advanced classes, early college placement, or other tools. About two-thirds reported being satisfied with their acceleration, rating it favorably across many categories:

[Source: SMPY]

The dissatisfied third of those 1 in 10000 students, for the most part, reported wishing they had been offered more acceleration. And advocates for gifted education strongly endorse the notion that acceleration is under-used. A Nation Deceived is premised on this idea — though besides for “more” the report doesn’t get specific concerning how many students ought to be accelerated, and the report mostly makes a cultural argument in favor of acceleration, citing stories like Martin Luther King Jr. graduating high school at 15.

We wanted to know more about how educators think about acceleration, so we surveyed (via twitter) twenty-one teachers, academic coaches, tutors and administrators. The survey prompted educators to respond to the following scenario:

In your school there is currently a 1st Grader who does math above grade level, e.g. he performs long division in his head. His parents initiated contact with the teacher after hearing their child complain that math at school was boring. They’re concerned that he isn’t being challenged. The classroom teacher knows that he is above grade-level in math, and is trying to meet his needs in class. The parents, however, do not think the current situation is working. The teacher reports that the student is difficult to engage during math class, and that sometimes he misbehaves during math.

From their responses, it certainly seems that acceleration was on the table, but almost always the last option after a number of in-class or non-classroom options (e.g. after school clubs) were explored. That acceleration in math should be a “break in case of emergency” response is also the line offered by the National Council of Teachers in Math: tracking is morally indefensible, acceleration should be viewed with suspicion but can sometimes be appropriate.

In many ways, mainstream education is living in Bob Slavin’s world. He was a leading opponent of tracking, but was impressed by certain forms of ability grouping. He took the research on ability grouping that actually works (through assessment, frequent regrouping, and curricular modification) and used it to create a program for failing schools. He expresses suspicion about acceleration of gifted students in general, but agrees that at times it is a useful and necessary tool. If you broach the conversation about acceleration with your child’s teachers, you might hear some version of Bob Slavin’s take.

There is more to say about where this skepticism comes from. But it’s important to note that just because a student could be accelerated doesn’t always mean that they should. While some gifted students fit the profile we sketched above — frustrated with school, bored and underchallenged, and finding it hard to connect to peers — many equally capable students are happy in their school lives. (We heard some, but not many, happy stories from online commenters.) If a child is happy and successful without acceleration, they are likely to remain happy and successful regardless of whether they are accelerated, and if they don’t want to accelerate, it should not be forced on them. At least some of the suspicion towards acceleration comes from parents who inappropriately push schools to accelerate their happy, satisfied children.

Acceleration is also not the only option. There is much more to learn than is taught in regular courses. Even in a normal class, a well-designed curriculum or an experienced teacher can create “extensions” to the main activity, so that students who are ready for more have something valuable to engage with. Enhancement or exposure to new, similar topics can serve students as well. A student who has jumped ahead in arithmetic may be entranced by a glance at Pascal’s triangle and number theory. One who is fascinated by English might find similar joy in learning Spanish or Chinese. Both of these, alongside acceleration, follow a simple principle: if a child wants to learn more and is able to do so, let them learn more. Overall, the balance of evidence suggests that acceleration is a practical and resource-effective way to help gifted, underchallenged students flourish in schools.

Our favorite one-stop source for reading on acceleration: here.

4. Educational Goals in Conflict

Through acceleration, tutoring, or ability grouping, some kids could learn more. Why aren’t schools aggressively pursuing that? Shouldn’t they be working to teach kids as much as possible? Isn’t that what a school supposed to do? That educators are skeptical of ability grouping or acceleration can be maddening from the perspective of learning maximization: Why are schools leaving learning on the table?

Here’s something we don’t talk about nearly enough: schools are simply not in the learning-maximization business. It turns out that parents, taxpayers and politicians call on schools to perform many jobs. At times, there are trade-offs between the educational goals schools are asked to pursue, and educators are forced to make tough choices.

Historian David Labaree has one way of thinking about these conflicting educational goals, which he expands on at length in Someone Has to Fail. For Labaree, there are three competing educational goals that are responsible for creating system-wide tensions:

  • democratic equality (“education as a mechanism for producing capable citizens”)
  • social efficiency (“education as a mechanism for developing productive workers”)
  • social mobility (“education as a way for individuals to reinforce or improve their social position”)

As Labaree tells it, these goals end up in tension all the time. A lot of things that seem like gross ineptitude or organizational dysfunction are really the result of the mutual exclusivity of these goals:

These educational goals represent the contradictions embedded in any liberal democracy, contradictions that cannot be resolved without removing either the society’s liberalism or its democracy … We ask it to promote social equality, but we want it to do so in a way that doesn’t threaten individual liberty or private interests. We ask it to promote individual opportunity, but we want it to do so in a way that doesn’t threaten the integrity of the nation or the inefficiency of the economy. As a result, the educational system is an abject failure in achieving any one of its primary social goals … The apparent dysfunctional outcomes of the school system, therefore, are not necessarily the result of bad planning, bad administration, or bad teaching; they are an expression of the contradictions in the liberal democratic mind.

Ability grouping and acceleration fit nicely within the tensions Labaree exposes. These learning-maximizing approaches could find support from those who see education as a national investment in our defense or economy. Of course, the strongest demand for acceleration in schools can come from parents, who want schools to give their children every possible opportunity to be upwardly mobile. (“We want to make sure they can go to a good college.”)

Those act as forces in favor of ability grouping and acceleration. But schools also know that they are held responsible for producing equitable outcomes for a citizenry that sees each other as equals. A program that raises achievement for top students without harming others has an appeal an economist could love, but within schools this can count as a problem.

The way this plays out in practice is that many schools are inundated with requests to accelerate a kid. Parents — especially financially well-off, well-connected parents — can typically find ways to apply pressure to schools in hopes of helping their children reach some level of distinction. They’ll sometimes do this even when it wouldn’t benefit a child’s education (it would be educationally inefficient), or when it would exacerbate inequality (by e.g. letting anyone with a rich, pushy parent take Algebra 1 early).

In short, from a school’s standpoint those are two problems with acceleration. First, parents will push for it even when it’s not academically or socially appropriate. Second, it can exacerbate inequalities. That could explain where the culture of skepticism within education comes from.

This is meant entirely in terms of explaining the dynamic. The way this plays out can be incredibly painful. Systems designed to moderate parental demand can keep a kid in a depressing and frustrating situation:

My older son wanted to move up to a more advanced math course for next year. He took two final exams for next year’s course in February and answered all but 1/2 of one question on each. So roughly 90% on both and his request to skip the course was denied. (source)

Districts sometimes have extensive policies that can be incredibly painful to navigate when trying to get a student who truly needs acceleration out of a bad classroom situation. We heard from one educator who had a very young student expressing suicidal ideations. It was all getting exacerbated by the classroom situation — the kid said he felt his teachers and peers hated him because he loved math. The parents and the educator tried to find a better classroom for the child, and were met with all the Labaree-ian layers of resistance. Off the record, the educator advised the parents to get out of dodge and into a local private school that would be more responsive to his needs.

A happy ending: the 4th Grader moved to a private school where he was placed in an 8th Grade Honors class. He likes math class now. He seems happier, he’s growing interested in street art and social justice work.

But without a doubt, there are some unhappy endings out there.

5. Personalization Software

[source: Larry Cuban]

“Ours is an age of science fiction,” Bryan Caplan writes in The Case Against Education. “Almost everyone in rich countries — and about half of the earth’s population — can access machines that answer virtually any question and teach virtually any subject … The Internet provides not just stream-of-consciousness enlightenment, but outstanding formal coursework.”

The dream of using the Internet to replace brick-and-mortar classrooms is a dream that is entirely in sync with the times. This is reflected in the enormous enthusiasm directed towards online learning and personalization software. Bill Gates, Elon Musk, and Mark Zuckerberg have all invested heavily in personalization and teaching software. And the industry as a whole is flush with funding, raising some 8 billion dollars of venture capital in 2017, while reaching 17.7 billion in revenue.

Finally — a way out of the school system and its knot of compromises! If schools are institutions whose goals are in tension with learning-maximization… then let’s stay away from schools and their tensions and give the children the unfettered learning they want. Let’s create the ideal tutor as a piece of software.

This dream isn’t just in sync with our times — it has a long history. This history is particularly well-documented by historian Larry Cuban (author of Teachers and Machines and Tinkering Toward Utopia) and by Audrey Watters (she’s writing a book about it). Watters’ talk “The History of The Future of Education” is as good a representative as any of the major thesis: that the dream is larger than any particular piece of technology. Motion pictures, radio, television, each of these was at times promoted as an educational innovation, able one day to free students from lockstep movement through school and into a personalized education. From Thomas Edison to B.F. Skinner, tech advocates have long envisioned the future that (at least according to Caplan) we’re living in now.

Then again, tech advocates in the past also thought they were living in the age of personalized learning. In 1965, a classroom that used a program called Individually Prescribed Instruction was described this way:

Each pupil sets his own pace. He is listening to records and completing workbooks. When he has completed a unit of work, he is tested, the test is corrected immediately, and if he gets a grade of 85% or better he moves on. If not, the teacher offers a series of alternative activities to correct the weakness, including individual tutoring.

For comparison, here is the NYTimes in 2017, and the headline is A New Kind of Classroom:

Students work at their own pace through worksheets, online lessons and in small group discussions with teachers. They get frequent updates on skills they have learned and those they need to acquire.

The similarity between modern day and historical personalization rhetoric doesn’t settle the matter — in a lot of ways, clearly the Internet is different — but personalization software seems to have arrived at a lot of familiar, very human frustrations.

Anyone who has gone online to learn has, at some point, come face to face with this dilemma: On the internet, you can study almost all human knowledge, but usually you don’t. In a world with virtually every MIT course fully online for free, a world with Khan Academy and Coursera and countless other tools to aid learning, why has the heralded learning revolution not yet arrived?

In a way, the revolution has arrived — it just hasn’t improved things much. Rocketship Schools, a California charter using online learning for about half of its instruction, has had solid results. Lately, though, they’ve moved away from some of their bigger bets on personalization and rediscovered teachers, saying “We’ve seen success with models that get online learning into classrooms where the best teachers are.” School of One was a widely hyped high school model in NYC that was preparing to scale up its offerings… until a fuller picture of the results came in and it was pilloried. Online charter schools, meanwhile, seem to actively depress learning.

Part of the problem is that it’s hard to get solid research on the efficacy of various ed tech products. Many tools, particularly those sold directly to schools or used by online charters, are proprietary and stuck behind paywalls, selectively presenting their best data and limited demos. The ed tech sector in general seems to deliver mixed results to students.

Why is it so hard to make effective teaching software?

For one, teaching is complex. A good human teacher does a lot of complicated things — gets to know their students, responds to the class’ moods and needs, asks “just right” questions, monitors progress, clarifies in real time as a look of confusion dawns on the class, etc., etc. — and it’s simply hard to get a computer to do that.

Maybe, theoretically, a piece of software could be designed that does these things. But in practice, many software designers don’t even try. It’s easier and cheaper to make pedagogical compromises, such as providing instruction entirely through videos. Yes, there are some thoughtful tools made by groups like those at Explorable Explanations, such as this lesson on the Prisoner’s Dilemma. But building high-quality tools well-adapted for a digital environment is difficult and time-consuming, and for prospective designers, destinations like Google or Blizzard tend to be more glamorous than working with schools. In practice, humans currently have a lot of advantages over computers in teaching.

Even if we overcame all the design issues, though, would students be motivated to stick with the program? Studies of online charters point to student engagement as the core challenge. When you put a kid in front of a computer screen, they jump to game websites, YouTube, SlateStarCodex, Google Images — anything other than their assigned learning. Many educational games that try to fix this resort to the “chocolate covered broccoli” tactic, trying to put gamelike mechanics that have nothing to do with learning around increasingly elaborate worksheets.

To be fair, student engagement is also the core challenge of conventional schools. But that’s precisely what the much-maligned structures of school are attempting to confront. The intensely social environment helps children identify as students and internalize a set of social expectations that are supportive of learning. The law compels school attendance, and schools compel class attendance. .And, once a child is in the classroom, their interactions with actual, live human instructors can set high academic expectations that a child will genuinely strive to meet.

The conventional story is that school is incredibly demotivating, but compared to their online counterparts schools are shockingly good at motivation. MOOCs like those on Coursera have an average completion rate of 15 percent — public schools do much better than this. Popular language app Duolingo’s self-reported numbers from 2013 would put their language completion rate at somewhere around 1%. If all a user has to rely on is their daily whim to continue a course, the most focused and conscientious may succeed, but those are the ones who already do well in schools. That’s a big part of why people lock themselves into multi-year commitments full of careful carrots and sticks to get through the learning process. Writers such as Caplan think that people are revealing their true interests when they skip learning to fart around on the web, but we might as well see a commitment to attend school as equally revealing. People need social institutions to help do things we’d truly like to do. As such, even as computers become better teachers, the motivational advantage of schools seems likely to persist.

How might tech-based learning tools address these factors, so they might stand a chance at holding students’ attention long enough to teach them? Art of Problem Solving, an organization promoting advanced math opportunities to children, makes a good case study. It’s found a balance worth examining. First, it provides accessible gamelike online tools that center on a careful sequence of thought-provoking problems. Second, it offers scheduled online classes with the promise of a fast pace, challenging content, and a peer group of similarly passionate students taught by subject matter experts. The online classes are more expensive offerings, but they preserve the human touch.

What does that balance mean for students? If they’re in the conscientious, self-motivated crowd that wants to learn everything yesterday, they can gorge themselves on software designed to be compelling. No barriers keep them from progressing. Software can always point to a next step, a harder problem. On the other hand, if they want to lock a motivational structure around themselves and keep the social benefits of school in a more challenging setting, they can.

Not every successful tool need look identical, but that core idea is worth repeating: software should enable the passion and self-pacing of eager kids, but should not rely on that to replace the power of social, human motivational structures. Yes, sometimes even the same structures used in “regular” schools.

Online learning, then, fits squarely within the history of attempts to automate teaching. Over and again we make the same mistakes and forget the lessons of history: that teaching is more complex than our machines have ever been, that motivation is largely social, and that schools will have a hard time distinguishing between altrustic designers and opportunistic profit-seekers.

For those in the market for online learning there are a lot of mediocre tools available, and many truly bad ones. Right now, there’s nothing that seems ready to serve as a full-on replacement for school without consistent, careful human guidance.

That said, depending on your passions, there are some excellent resources for learning out there. Especially if a student has a caring mentor or a passionate peer group, they can learn a lot online. As educators and designers create more tools that respect both the power and limitations of machines, that potential can grow. But it’s not quite science fiction.

Our algorithm has determined that you should watch the following two videos: here and here to balance realism and idealism

6. Practical Advice

Education is complex and resists easy generalizations. That said, here are some generalizations.

On navigating school for your child:

  • The brightest students do not thrive equally in every setting. Even the best students achieve more with teachers than on their own. Unless tutoring or some other private arrangement is possible, this means that a school is the best place to be for learning.
  • But school right now doesn’t work for all kids. One fix: if a child wants to be accelerated and seems academically prepared for it, acceleration will usually help them.
  • Most schools aren’t in the business of maximizing learning for every student, and in particular they tend to be skeptical of acceleration.
  • Therefore: If your kid needs more than what school is offering, be prepared to be a nudge.
  • But if you think your kid needs to be challenged more and your kid is perfectly happy in school, try really hard not to be a nudge.
  • Don’t fight to move your child to a class that covers the exact same material at the exact same pace but has the word “Honors” next to it. That sort of ability grouping makes no educational difference.
  • Prioritize free, open online tools. Don’t expect online tools to do the work for you or your child. Expect more distraction and less progress if online learning time is unstructured or unsupervised.

For educators:

  • If you are an elementary teacher or administrator and your school is looking to try new things, consider cross-grade ability grouping by subject, especially in math and reading.
  • Gifted kids are usually not equally talented in all fields. Consider options to accelerate to different levels in each subject based on demonstrated skill in that subject.
  • A lot comes easily to smart kids, and sometimes they never get the chance to learn to struggle. Find something they think is hard, academic or not, so they are able to handle more important challenges later.
  • If a child is bored in your class and knows the material, they probably shouldn’t be in your class.

For tech designers and users:

  • If you’re making online tools, make the learning the most interesting part of them. Don’t rely on chocolate-covered broccoli or assume that just presenting the material is enough. Take the problem of motivation seriously.
  • Look for passionate groups with robust communities, whether online or offline. Don’t overlook the social aspect of learning.

And for advocates of educational reform, in general:

  • People almost only talk about educational efficacy. But don’t be fooled — educational debates are only sometimes about what works, and frequently about what we value.

One last thing: if you’re an educator or a parent or just somebody who spends time around children, take their feelings seriously, OK? If a kid is miserable, that’s absolutely a problem that has to be solved, no matter what district policy happens to be.

Acknowledgments: Thanks to /u/Reddit4Play from reddit, JohnBuridan from the SSC community, blogger Education Realist, and many others who read drafts and offered ideas along the way.

This entry was posted in Uncategorized and tagged . Bookmark the permalink.

403 Responses to [ACC Entry] Does The Education System Adequately Serve Advanced Students?

  1. Markus Stoor says:

    Excellent work. (And by that I sadly mean it confirms more or less all of my priors as a reasonably long time high school teacher and graduate student in educational matters as well as documents them with references.)

    If I would offer one suggestion for an even better writeup, it would be to move the part about what percentage of the children the whole thing is about to the beginning and maybe try to expand it a little. As a teacher and educator, there is enormous difference between policy for handling 2 sd+ and 4 sd+ children.

    Since the final writeup seem so conventional to me I would like to direct a question to tracingwoodgrains.

    What have been your experience of the process?

    • TracingWoodgrains says:

      Glad you enjoyed it, and that it read as accurate to you!

      We wrestled a lot with what percentage of children to focus on, and that’s one reason it ended up under-emphasized. It’s surprisingly hard to nail down the number of children who end up frustrated and slowed down in school, particularly since most of the research around “gifted underachievement” focuses on non-school factors, and only a few researchers (Gross, SMPY) usually bother dividing “gifted” kids into more precise categories and hunting down 3+ or 4+ SD kids. You’re right, the policy difference for handling the two groups is huge.

      As for my experience with the process, my first impression is that it was long. We have a 150-page pile of background notes and side conversations. Finding a really good adversarial collaboration partner (which Michael was) creates a really cool environment: you’re both passionate about something, but you’re passionate about different views of it and see different parts as important, which means it only takes a comment or two to spawn pages of discussion and a dive through four or five more research papers.

      A lot of this didn’t end up in the final paper, some due to relevance, some due to length, and some simply because we couldn’t reach a satisfying consensus. Even on what we did include, though, we noticed a compelling phenomenon: we would agree pretty much entirely on the object-level facts, but then take a step back and wrap them in dramatically diverging narratives. For me, generally speaking that meant a cynical view of traditional education and an optimistic view of alternatives; for Michael, the opposite. So once we started writing the paper there was a lot of, “So, we agree completely, right?” “Right!” “Great, I’ll just write this bit, then.” “Wait, no no no, you focused on all the wrong things! We need to go back and redo this completely.”

      It was by far the most fruitful period of learning I’ve had in any context, to be honest. When someone’s questioning the core assumptions behind your views as you research, you end up diving pretty deep in all sorts of directions. Michael was really good about both questioning without attacking and providing useful research, and as a result I was pretty obsessive about the topic during the contest timeframe.

      I wouldn’t say I’m in a rush to do it again. It was a lot of work. But it’s one of the most meaningful opportunities I’ve had, and I’m really grateful Michael spent so much time working with me on this. If there’s interest, I might do a more complete write-up of what the process looked like, with more topic-related specifics.

      • Evan Þ says:

        I’d be very interested in hearing about the process!

        • TracingWoodgrains says:

          My current goal is to sit down within the next few days and write a more detailed post outlining where we started, particular areas of contention or compelling differences of perspective, and a few of the points we talked a lot about but never made it into the essay. Working to present things as a unified front was a fantastic exercise, but as several comments have pointed out, it makes it hard to notice the push and pull of opinions that led to those conclusions, and I expect that for all the collaborations, seeing what needed to happen to reach the endpoint would be a useful exercise.

          If or when I make the post, I’ll likely end up tossing it on SSC’s companion subreddit and in one of the next Open Threads.

  2. lurking says:

    Wow, the other entries are going to have to be pretty damn impressive if this is what they’re competing against.

    Has anybody used clustering algorithms to group classes? You could cluster students according to scores in subjects e.g. every month so that the curriculum could be dynamically adjusted to accommodate the maximum number reasonably possible.

    • roystgnr says:

      Wow, the other entries are going to have to be pretty damn impressive if this is what they’re competing against.

      Seconded. With such a small sample set I’m guessing this is an outlier (because the best writers or the most fruitful topic just happened to get ordered first), but I’m going to be utterly thrilled if it isn’t (because the adversarial collaboration format either attracts the best writers or facilitates or encourages the best writing on the most interesting topics). Even if that’s the case, adversarial collaboration may not be the Holy Grail of replicating rational writing, but it might still be a big improvement over the current “start a great blog, hope some of your audience is inspired to do the same, hope some of those succeed, repeat”.

    • HeelBearCub says:

      You could cluster students according to scores in subjects e.g. every month so that the curriculum could be dynamically adjusted to accommodate the maximum number reasonably possible.

      I think you are misunderstanding the implications of this summary.

      Over and over, we see see that the educators have a very important role or function. The function is properly responding to the overall needs of the students in their class. That requires being able to accurately assess those needs. And the needs don’t exist solely for each individual student, but the class as a whole.

      Switching students every month will amount to a problem of “thrashing”, where every month teachers are spending time re-learning their classroom, instead of serving it.

      • lurking says:

        Switching students every month will amount to a problem of “thrashing”, where every month teachers are spending time re-learning their classroom, instead of serving it.

        I don’t think so. The clusters would probably be quite stable after a few months. But even then, fine, we could say check the clusters every three months. Or just once every year. It’s just that when I went to school kids spent several years with the same group of arbitrarily selected people in every subject, when those groups could have been selected in a so much more logical way.

      • baconbits9 says:

        I think one issue that our current system faces is that there is little pairing of teachers and students, as if teachers are practically fungible and that they don’t have their own preferences or abilities to teach certain students better.

        • Matt M says:

          IANAT (teacher) but I’ve known several. As far as I can tell, the overwhelming majority of teachers would prefer to teach the brighter kids. Throughout my entire educational experience from K-Masters, it seemed like the teachers with the most seniority were always given the top tracks, while the newbies had to “earn their stripes” teaching the remedial courses.

  3. TentativeQuestioning says:

    This may not be the most appropriate place for this so if it isn’t I’m fine with this being deleted.

    But I was wondering, how does unschooling/Sudbury education fit in to all of this? Is there just too little research on it to seriously comment on it? The only even semi-serious research I’ve seen on it are the surveys conducted on graduates from the Sudbury School in Massachusetts, which suffers from limited sample size and author bias, among other things. Is or has there been any attempt to research it? Or do education scholars just not take it seriously? Is it just too radical?

    • JohnBuridan says:

      I don’t think that it’s fringeness makes it ignored; I think it’s fringeness makes it highly variable. In my experience homeschoolers and unschoolers have vastly different outcomes depending on the same factors from the post: presence, or lack thereof, of guidance and attention, conscientiousness, compelling content. Furthermore, I am extremely skeptical of non-rich parents being able to provide an adequate high school education to a gifted child.

      Imagine the set of all homeschoolers in America. Each homeschool/unschool can be lumped into different categories depending on the adults involved. As an educator at a college-model k-12 school, I deal with a lot of former homeschool/unschool parents. Sometimes they respond very well at getting their child the help and support they need, sometimes they do not, sometimes they cannot.

      • baconbits9 says:

        Furthermore, I am extremely skeptical of non-rich parents being able to provide an adequate high school education to a gifted child.

        I’m not sure what you mean by rich/non rich but a handful of home schoolers I have talked to with gifted kids basically sent them to community college classes for more advanced (if not strictly better) instruction for a few hundred dollars a semester. I am also under the impression that gifted+interested students typically learn straight from textbooks/videos with parents doing fairly minimal interventions and encouragements.

        • JohnBuridan says:

          I meant that even super homeschool parents have difficulty teaching beyong the 8th grade level, and so they need help which requires either some excess capital or a rich social network. Often homeschool parents form co-ops in which parents who are knowledgable/enthusiastic about certain subjects teach, and sometimes they hire an outside tutor to teach the classes they don’t have a teacher for. I know of four co-ops in my county. I’m sure there are many more than that. They pool resources are and are very cost-effective, while usually being academically nondemanding.

          Many parents do not have money to spend on tutors or community college classes ($350 a class in my county + book fee + transport) to take care of an entire high school education.

          (Furthermore, co-ops do not have longterm staying power, and usually fold in on themselves due to the diversity goals of adults involved, low investment, and easy exit.)

          • Philipp says:

            It’s possible, John Buridan, in at least some states for students to enroll part-time as homeschoolers and part-time in public school (see, e.g., here for Iowa). That also means that homeschoolers can take advantage of state-funded post-secondary education options to pursue more challenging coursework without paying tuition (though transportation might still be an issue). That’s not enough for a full day’s education, but it would certainly be enough to cover classes that are more intellectually demanding, expensive to do right (chemistry comes to mind), or simply outside the knowledge of the parent-teacher.

            You might know all of this, but, as you don’t mention it, it seems an important qualifier to your comment. There are options for managing high school that do not put the whole burden of cost on either parents or the local school district. Furthermore, I wonder whether you underrate the ability of parents to teach. Surely any “super homeschooler” worthy of the name is going to have at least a Bachelor’s in some subject. Do you think that someone with a BA in History or English couldn’t tutor children in literature or American history (or perhaps: guide his/her children in studying) to at least the level reached in most high schools? I’ll grant, someone with a Master’s in the same field, a dynamic teaching style, and a highly engaged classroom could do substantially better, but how many children are going to find that?

            Further, what do you mean by “rich social network?” If you mean that parents must have rich friends, I don’t think your explanation sustains the claim. If you mean, on the contrary, that parents must have a full and strong set of social connections in order to manage high school, I would guess that that is not uncommon among homeschoolers. Certainly the ones I knew when I was homeschooled seemed to, and the tendency to have 1) fairly strong religious ties, 2) mothers with the flexibility to “network” with others mothers during the day, and 3) a relatively small group of families involved (few enough, in the highly educated, smallish city I know best that every homeschooling family could at least be aware of the rest) would tend to suggest that such connections are common more generally.

            Of course, high school is more complicated than earlier grades, and I think I agree that most parents are unlikely to handle it as well. I would guess that most of the homeschoolers I knew growing up either went to high school full-time at some point or took classes in high school/through PSEO; a few went to college early. But it certainly was not obviously a pragmatic necessity either to be wealthy or to stop homeschooling at ninth grade.

          • JohnBuridan says:

            All good points, Philipp. It’s true that some states allow that. Mine doesn’t nor does the state I border, so I didn’t think to mention it. I believe Indiana one such state that does.

            I do not mean to underrate parents’ ability to teach. I see it all the time. Parents require a “socially rich network,” when there is not a shared public school option. My claim is that no parent has the expertise needed to teach every high school subject. A socially rich network is just a group of friends who have expertise which you don’t have and time to help teach the things you don’t know as well.

            From what I can tell being able to call on someone you trust who can teach or tutor the subject you as a parent are not comfortable with is necessary for successful homeschooling for a family through the years.

          • baconbits9 says:

            Furthermore, co-ops do not have longterm staying power, and usually fold in on themselves due to the diversity goals of adults involved, low investment, and easy exit

            These issues also mean it is easy to start a co-op, especially if you just have some short term needs that you want filled. Beyond that just family and friends can help out a large amount. In high school I lived with my mother’s first cousin who taught french in a Canadian high school for 2 weeks to try to catch me up in a subject I didn’t have much affinity for, and took an advanced biology course at a local university one summer (I was not home schooled) and a few years ago my wife’s aunt and uncle took in one of her home schooled cousins for 3 months to give her a year’s worth of calculus.

            The point here is not that everyone has the right support structure to do all of these things, but that many people probably do or at least could if they put effort into building them. The number of people who are some combination of poor or isolated enough, unable to master the subject matter themselves with the aid of books and the internet and with no friends, relatives or co-ops to turn to who end up with a 3+ sigma child is going to be fairly low.

        • Watchman says:

          Aren’t you making a major assumption that all gifted children have the same learning style there? Not all children learn in the same way, and I’ve seen no reason to think learning style is correlated to gifted (I used to work in school administration including setting up applied systems for recording progress).

          • baconbits9 says:

            Aren’t you making a major assumption that all gifted children have the same learning style there?

            I’m not, as it isn’t an exhaustive list of ways to teach intelligent kids, its just two examples to help define the issue. If you start with the supposition that only the ‘rich’ can afford to pay for decent instruction for talented high school age students then it helps to define what ‘rich’ is. If that means being able to pay $3-400 a class per semester and provide transportation then you are talking about the top portion of what is generally considered lower class income* and up being able to afford 1+ classes a semester. That leaves maybe 15-20% of the population for whom it is genuinely out reach for, meaning the scope of the issue is far less than was implied by the post I was responding to.

            *some areas are low population enough that the transportation would be more onerous but (by definition) relatively few people live in these areas.

        • Plumber says:

          My 13 years-old son is being homeschooled (a classmate trying to stab him was the final straw), and has just started attending a Spanish class at Berkeley City College. 

          Judging by my own experiences in the Berkeley, California public schools in the 1970’s and ’80’s, my two “academic” classes (Cultural Anthropology and European history), and my “vocational’ classes (welding) at “community colleges”, and my time as an indentured apprentice plumber, as well as what I know of my little brothers education (with financial support from his family, including me, and his in-laws he graduated from San Francisco State and now has a white collar job with the State of Maryland) Junior High school should be eliminated, and High School should end no latter than age 16.

          Too much time is wasted, and youths could be better educated elsewhere. 

    • TracingWoodgrains says:

      We actually looked quite a bit at unschooling and Sudbury education during the process! You’re right that a lack of serious research makes it difficult to comment on directly, but a lot can be done by looking at the research comparing direct and exploratory education methods. In almost every study we looked at that compared the two, direct methods beat exploratory ones (the sort unschooling and Sudbury schools endorse) out on every measure of learning. This includes critical thinking processes and adapting knowledge for meaningful use, two points often raised in favor of exploratory learning.

      If your priority is academic learning, or building skills in general, unschooling is the wrong way to go. The motivation problems we mentioned with online school are fully in force when a child is in charge of their own learning, and it’s honestly really valuable to have a structure around you pushing and obligating you to study even topics you’re interested in. Since those are my priorities when it comes to education, I would personally put my children in virtually any other system before turning to a Sudbury-style one. I think it’s great that people keep experimenting with new and unusual systems, though. An ideal school would likely combine the high points of the ethos behind Sudbury schools (inspiring passion in students, focusing on the child’s natural aptitudes and interests) with a structure more suited to learning and gaining skill.

      • TentativeQuestioning says:

        Thanks for the response! I’d be interested to see some more cites on the advantage of direct over exploratory education. Are there some with longer observation periods such as years or decades, instead of weeks (such as this one arguing against direct instruction). A lot of the effects that I see Sudbury/unschooling advocates point to are fuzzy “life skills” like motivation, time management, cooperation, and conflict resolution, the latter two particularly in Sudbury schools. Reportedly these only become evident over the course of years, due to the deschooling effect.

        I think you’re right in terms of solid evidence it’s not the best choice that if you’re looking for academic learning. The argument that advocates give, though, seems to be that this kind of education builds those life skills which then enable people to do self directed learning effectively- in order to, e.g., do online learning, seek tutors, go through college. I’d personally give my kids the choice of what school they want to go to. I do wish there were more data – or even anecdotes – about kids who did Sudbury/unschooling and switched to another method or had negative experiences, and those who come from disadvantaged backgrounds. I also wish there was more info on how Sudbury/unschooling “grads” do compared to their socioeconomic peers. But if wishes were fishes, I’d be running a sushi bar 😉

    • Douglas Knight says:

      The comparison with unschooling demonstrates that ordinary education doesn’t do anything. Thus it is not surprising that tracking doesn’t do anything differently. The surprising result is that unschooling doesn’t do anything, either. Not surprising is that the authors write:

      Don’t fight to move your child to a class that covers the exact same material at the exact same pace but has the word “Honors” next to it. That sort of ability grouping makes no educational difference.

      and yet continue to write about the system as if it were an education system.

      • The Nybbler says:

        Yeah, that surprised me. I went to a tracked middle and high school system, and the courses were different between the tracks. The idea that someone would do tracking with the same material in each track, and expect useful results… my mind boggles. I guess the idea is it’s easier to teach students of similar ability together, but I’m unsurprised it doesn’t change things for the advanced students. I’d expect the only improvement (if any) to be in the middle (by removing potentially disruptive students and students who are a drag on the class)

        • AG says:

          My guess is that they mean within-class pacing, as well.
          For example, if a gifted student goes to a class for the next grade up, the teacher is still teaching to pace with the worst student. Despite the material being a grade up, it’s likely that the gifted student will still be unchallenged.

          To teach to the pace of ability means covering way more material in the same amount of time. Like the difference between 101 classes at MIT vs. a community college.
          An Honors class for 9th grade Physics should not be the exact same curriculum as the non-Honors 10th grade Physics class, it should cover more.
          (Or, AP classes should be worth more than one college credit if paced correctly, not just equivalence, since the former assumes a higher ability grouping than the latter.)

          You want Honors to be y=z*mx, not y=mx+b.

          • skybrian says:

            I understand the article to say that acceleration, while not perfect, often works well. Are you disputing that?

          • AG says:

            I’m addressing the quote that Douglas Knight made, of cases where “tracking” was used, but were not really tracking. The argument is that those particular execution details weren’t really acceleration, as the rate of learning was still the same. Therefore, it is important to make sure that tracking differs in the rate of learning to match the group ability.

  4. Sniffnoy says:

    Here’s a related point whose correctness I’m not too sure of but I thought it was interesting and it’s not mentioned here — Andrei Toom, in his long explanation of how Russia does math instruction better than America, makes (somewhere in there; I’m going by memory, I’m afraid) the point that America’s idea of acceleration isn’t that great, because instead of going on from an easier version of a subject to a harder version, it just gets you from the easy version of one subject to the easy version of a different, later subject. Better than nothing, but still not really the right thing.

    • Randy M says:

      Later subjects, at least in math, at later because they build on earlier subjects, but also are harder, require more analytical thinking or sustained effort, than rudimentary ones. Though this might not be as much the case with elementary math; I could see a gifted student flying through much of the k-5 math in a very short time.

      • axiomsofdominion says:

        This is not correct. There is a lot of evidence that you can move parts of various subjects around, like having pre-calculus even in elementary or middle school rather than as the standard track in highschool.

        • The Nybbler says:

          The course called pre-calculus is kind of a grab bag. It combines trigonometry, limits, logarithms and exponentials, conic sections, and a few other subjects, some of which can be done before Algebra but some of which cannot.

        • Randy M says:

          Hmm… I’m not convinced. Maybe it’s more like a tree structure than a ladder, but I have a hard time picturing geometry or calculus without algebra or arithmetic.

    • TracingWoodgrains says:

      Yeah, this is one of the biggest problems with traditional acceleration, and why I have a hard time endorsing it outright. I focus on it because it’s practical and it’s close to a strict improvement over an alternative of regular classes for advanced children, but I’m much more ardent in support of advanced curricula like the one the folks at Art of Problem Solving use. Their program remains my model for what education for the most advanced students should look like.

    • Douglas Knight says:

      Similarly, Benezet did not actually remove math from the elementary curriculum, only arithmetic exercises.

  5. OptimalSolver says:

    Can’t these gifted kids educate themselves? I’m absolutely middling in intelligence, but as a child I devoured vast amounts of knowledge that had nothing to do with my school curriculum.

    If a kid has intellectual curiosity, they don’t need encouragement, just access to information (e.g. the Internet).

    • zzzzort says:

      Honestly not sure if your post is a deliberate strawman, but you make an assumption here that intellectual curiosity = gifted, or that intelligence and conscientiousness will always be correlated. Smart people can be lazy too. I’m more concerned that when a smart kid is unmotivated in a normal class people think ‘that kid needs to be more challenged’ rather than ‘that kid needs help to be more conscientious.’

      • beleester says:

        I think challenging work is a necessary precondition – conscientiousness helps you most when the homework isn’t something you can breeze through with zero effort.

    • fion says:

      Maybe you’re middling in intelligence but very high in conscientiousness?

      I’m high in intelligence but pretty bad at educating myself. At school and university I took extra classes, but I didn’t really read about all the many subjects that interest me that weren’t covered in classes.

      EDIT: somewhat ninja’d

      • JohnBuridan says:

        In college I tried to take the most difficult classes precisely because I was not likely to do rigorous studying on my own time. 🙂

    • ana53294 says:

      They indicate that gifted kids are gifted in certain areas, but that doesn’t mean they are gifted in every area. Scott has mentioned the Indian mathematician Srinivasa Ramanujan, who was really gifted in math, but apparently did not manage that well in other subjects, IIRC.

      I was good at math, my handwriting was similar to a crazy hen, I hated PE with every fiber of my being, and I absolutely hated music classes because I am tone deaf. If I was left to my own devices, I would just do math, read cool books, and forget everything else.

      Schools are supposed to teach every subject in the curriculum, because we want kids to have a well rounded education. So even if a child loves math and really enjoys it, you may still need to drag the kid so they do at least a bit of exercise (or vice-versa). Children with perfectly all-rounded interests that resemble the school curriculum do not exist, some of the subjects will always be more boring.

      • Cliff says:

        Good thing you spent all that time on handwriting, music and PE, I’m sure that really came in handy later in life 🙂

        • ana53294 says:

          Well, PE is important. I have ruined my health by not doing exercise, and I am slowly learning to do it.

        • The Nybbler says:

          PE was certainly no good for me. Until the testosterone fairy hit, I just couldn’t keep up, at all. I was rather surprised when I got older and was actually able to do some of those things they expected you to do in PE.

          (the testosterone fairy is a heavily built bald (but otherwise hirsute) dude, who is about 6″ tall and sports diaphanous wings, a gold tiara, and a wand with a star on the end. In case you’re wondering)

    • Nancy Lebovitz says:

      It’s harder to educate yourself if large chunks of your time and attention are taken up with schooling you don’t need.

      • JohnBuridan says:

        Of course, this begs the question if ‘being educated’ is different from knowing-a-lot-about-a-subject-you-are-predisposed-to-like…

        I think that the so-called “truly educated person” should be Heinlein-esque character.

        A human being should be able to change a diaper, plan an invasion, butcher a hog, conn a ship, design a building, write a sonnet, balance accounts, build a wall, set a bone, comfort the dying, take orders, give orders, cooperate, act alone, solve equations, analyze a new problem, pitch manure, program a computer, cook a tasty meal, fight efficiently, die gallantly.

        • Jiro says:

          I tend to agree with the criticisms of that Heinlein quote. Much of that list is arbitrary. Some things in it are mainly useful in certain settings. Some things in it are useful in a wider range of settings, but the emphasis is odd. And some of it is probably from Heinlein assuming that things that happened to be useful to himself are universal necessities.

          • JohnBuridan says:

            I agree. But also the “spirit of list” is what I’m getting at. Surely, I wouldn’t defend many or even most items on the list. Would you agree though that an ideal list for you might make for yourself would still be quite broad and include broad skills, specific skills, social know-how, and some quirky aspirations?

          • Simulated Knave says:

            …limited usefulness? Which would you cut?

            The only ones I can see that might not apply generally are conning a ship and building a wall. And even conning a ship isn’t a bad one.

            Changing a diaper is important, if only for understanding what the people who raised you went through.
            Planning an invasion means you understand the military, logistics, and large-scale planning. The last is especially useful.
            Butchering a hog means you understand where meat comes from and the ethical choices involved and know some things about non-human anatomy that may be useful even if all you ever do is try to figure out if your dog is really sick or not.
            Conning a ship…may not be that useful since “give orders” is also on the list. The importance of personal responsibility, perhaps? Quick decision-making? Being cautious with big expensive things? Learning this isn’t bad, it just may be redundant. If he means a spaceship, understanding orbital mechanics is becoming more important given satellites etc and the long-term requirement for the species to survive.
            Design a building’s use is fairly obvious. It’s handy to know how buildings work. I wish more architects and planners DID.
            Write a sonnet – understanding writing principles and use of language as art is a good thing.
            Balance accounts – obvious.
            Build a wall – assuming he means a stone one, I must admit that this isn’t as useful as it once was. If only because there are so many ways to do it it doesn’t mean much. If he’s talking about mortarless stone walls, it can give you an appreciation for history and craftsmanship. Hell, this one could have transformed the last election. 😛
            Set a bone – even if you never need it, it’s good to know.
            Comfort the dying – see above. Also, of course, allow comforting other people.
            Take orders and give orders is extremely obvious. So are cooperate and act alone.
            Solve equations – useful for logical thinking and understanding a lot of higher concepts in physics etc. Also lets you do a lot of these other things better.
            Analyze a new problem – obvious.
            Pitch manure – being able to uncomplainingly do dirty smelly jobs well is an important human skill. Also teaches you the careful handling of dangerous and unhygienic large pointy tools. Is surprisingly hard to do well and makes you really appreciate modern society and the way you don’t have to do this.
            Program a computer – the world grows more computerized, and the understanding of that is increasingly important, if only so you can judge what is being done around you.
            Cook a tasty meal – duh.
            Fight efficiently and die gallantly – if people end up fighting, it’s good if they do it well. And if people understood these last two, we might be less eager to outsource it to others so much. Also, if the principles in fighting efficiently and dying gallantly were applied to large public debates, the world would DEFINITELY improve.

            I am 100% sure that if everyone in the world woke up tomorrow knowing any one of these, the world would be a better place. The least useful would be “building a wall”, but depending what is meant like that even that one’s pretty useful to a lot of people.

          • Randy M says:

            I don’t know what conning a ship is and couldn’t tell you a sonnet from a limerick. I think few need to plan an invasion specifically, though “plan for the future” is important, especially to be able to clear some space for the unexpected. Building a wall and designing a building might be somewhat redundant, and designing a modern building is not anything I’d put on the entrance exam to human beingness–let’s say “erect a structure” maybe.
            Solving equations and programming a computer I can do (at some modest level) but I think it greatly overestimates the lower level human capacity. If a person can balance their finances, that’s probably enough basic math. Logical thinking is important, but computer programming specifically I think we could rephrase as “spot a fallacy” or something like that.

          • Jiro says:

            Design a building’s use is fairly obvious. It’s handy to know …

            Pretty much any skill is useful, if you count indirect requirements that you have to have in order to practice the skill. If those things qualify for the list, the list becomes meaningless. Furthermore, while knowing how a building works is useful, “has at least some use” is too weak a standard for inclusion in the list–by that standard there are dozens of other “know how X works” that also should go in there.

        • silver_swift says:

          That quote lacks punch without the “Specialization is for insects” part. Much as that statement is false when taken literally (humanity has many more specialized roles in our society than any species of insect and it has served us extremely well), I feel like it needs to be part of the quote.

          • JohnBuridan says:

            I submit!

            A human being should be able to change a diaper, plan an invasion, butcher a hog, conn a ship, design a building, write a sonnet, balance accounts, build a wall, set a bone, comfort the dying, take orders, give orders, cooperate, act alone, solve equations, analyze a new problem, pitch manure, program a computer, cook a tasty meal, fight efficiently, die gallantly. Specialization is for insects.

      • arlie says:

        As a (gifted, bored) child, I was told that the purpose of school was to learn how to interact with others. And accelerating me would prevent that, so “suck it up”. That was never taught in any way – somehow being with other children was supposed to do the trick. And those children had to be average – tracking (which I’d heard about, and yearned for) was totally beyond the pale.

        I may (or may not) be better at coping with groups of people who mostly don’t like me, and are sometimes quite nasty about it, as a result. But when I heard about Columbine, I completely understood the motivations of the shooters.

        And I’ve underachieved throughout my adult life, as well as carrying a huge loud of resentment that sometimes wells up as semi-crippling psychological issues. This isn’t obvious – I’ve hugely exceeded my family’s social class attainment, going from working poor to the top 2% or so by income. But I only started learning to work hard, or cope with failure, in my 40s and 50s.

        Confounding factor: Asperger’s syndome – undiagnosed, since I was born in 1957, before most people had ever heard of it. This would be a mixed blessing for me – I took a job that matched a “special interest”, meaning that I might spontaneously put in long hours and huge amounts of effort… but generally only at something I was already very good at. OTOH, it made learning to cope with people, without any instruction, much harder than it might have been with ‘normal’ wiring.

        The only part of my education I enjoyed was university. Undergraduate courses were still easy enough for me that I could basically coast, and I tracked myself by getting admitted to a highly selective university. I still had social problems, but they no longer took the form of bullying, and there were enough other nerds that I didn’t generally notice the people who actively excluded me from their social groups. I was too lazy/afraid of failure to graduate with more than basic honours, but that was good enough for most purposes thereafter.

        • axiomsofdominion says:

          This is the most frustrating thing about school. Like, why are social skills supposed to be magically different. Not only children but also parents have wildly varying social ability as well as ability to pass on positive lessons.

        • Viliam says:

          I was told that the purpose of school was to learn how to interact with others.

          Nah, that’s obvious bullshit.

          In the alternative universe where this statement is meant seriously, elementary schools provide lessons in Nonviolent Communication or something like that. (And for a really low-hanging fruit, bullying is taken much more seriously.)

          Unless “learning how to interact with others” is an euphemism for “learning that nerds are at the bottom of social hierarchy (until they grow up and start making money)”, of course.

    • AnthonyC says:

      From my own experience, in some ways this can exacerbate the problems with engagement. The internet wasn’t really a thing for me until middle school, but I spent a *lot* of time learning things on my own outside class. The result is that school gets even more boring, because you know there’s something else you *could* be doing that would let you *actually* learn things.

      Plus you often understand things at a deep enough level that you spot the flaws in your teacher’s explanations. I didn’t know, at first, how to differentiate when this was due to their not understanding, and when it was a deliberate attempt to simplify content that would be presented more deeply in later years. I was able to learn pretty well the art of telling teachers they were wrong without getting in trouble, while still pretending to pay attention and still doing all the too-easy work. That, plus my district’s attempts to provide lots of enrichment, made it bearable for me, but that had as much to do with my demeanor as anything else.

      Note: school remained very easy for me until around sophomore or junior year of college, and yes, I learned then that I had no idea how to actually study or work hard at something that was difficult for me, not to the level my peers did. Ten years later I still don’t think I’ve ever really overcome that, though I have improved.

    • R4P says:

      This is addressed in the technology section: access to information worth learning is necessary but not sufficient to cause learning. The motivational structure of school is highly effective at causing learning compared to crossing your fingers and hoping kids will teach themselves. It stands to reason that many children would prefer a school that taught what they wanted to learn at the level they wanted to learn it rather than having to go learn it on their own, and that many more children would benefit from such an arrangement whether they preferred it or not.

    • CatCube says:

      They may not need encouragement, but they may need a set of rails. I’m probably pretty smart–I did well on the ASVAB and the ACT, but I don’t know that that indicates much more than I’m “gifted” at standardized tests–but I am an astonishingly lazy motherfucker.

      I did the same thing you did, devouring books outside of class, but never quite managed to actually do the work required without due dates. This wasn’t just when I was a college freshman. When I was in grad school, I had one class that was purely online, where the professor had recorded all of his lectures, put the homework on the site, and said “the due date is the last day of class, and once you turn them all in it will unlock the final”. I had fourteen weeks to do that class, and I put it off until three days before the due date. And the topic wasn’t boring or uninteresting–it was about remote sensing methods in geological investigation, and I signed up for it because I wanted to learn more about it! But that never seemed to translate into actually doing the reading assigned, just reading about other random stuff on the Internet.

      I even do this with things tangentially related to work. I had a co-worker send me a reference for some information that I needed, and I then proceeded to read 45 pages past it over the next half hour (it was about modeling the interaction between water and dam appurtenances during a seismic event). The information was fascinating, but not something that I will ever be using at work within the next year (the reference was to the method we use, the other 45 pages was about a modeling methodology totally inappropriate for the work I’m doing). That half hour was about as useful to my employer as coming here and reading SSC in the grand scheme of things. This is why I will never work remotely. After day three, I’ll be reading about fascinating things on Wikipedia or watching speedruns on YouTube, but no useful work will be occurring. I wish that I could work on what I’m supposed to do without oversight, but I’ve had painful experience show me that that’s not the case.

      • thevoiceofthevoid says:

        +1 to this. After my first year in an undergrad engineering program at a fairly good college, I went home for the summer with plans to teach/reteach myself the content of a discrete math for CS course. It had had a supar professor, but a free online textbook. (I got an A in the course by cramming the assigned readings before quizzes, but he didn’t cover certain topics that the course was supposed to.) I arrived home and promptly…
        Played minecraft for a month, until I managed to get a minimum-wage job to keep myself busy for the rest of the summer.
        The problem obviously isn’t that I don’t understand the content. It isn’t even that I don’t find it interesting. The problem is that when I crack open my laptop, booting up minecraft and building an ore-processing plant is more immediately gratifying than reading a PDF about algorithmic efficiency, even if I would higher-level want myself to do the latter and probably even enjoy it.
        Now I’m back at school, and with the set of rails back underneath me I’m…writing a SSC comment and continuing to put off reviewing/learning the aforementioned content, which I’m now actually being tested on tomorrow.
        I may have a problem.

    • jeqofire says:

      I tried that. I basically read everything on Hyperphysics, played around with Wikipedia, could not get a hold of any of the books cited on either site, and even though Hyperphysics introduced me to the tip of the Iceburgh of Feynman, it never occurred to me to look up any of his work or lectures (and this was before Youtube really took off).
      Result: I became even more of an arrogant crackpot. There was no one around I could discuss this stuff with, so all I got was reenforcement of how smart I was. There was nothing to point out weaknesses or misunderstandings I had, or to point me in the direction of what I was missing. I just kinda assumed I understood everything I’d read after rereading it a couple times, so even when I went to places like physicsforums, I had nothing to say that could have led to further improvement. Maybe this would have gone better if Con Academy / Lesswrong / etc had been around at the time, but I rather doubt. I’d probably have read HPMOR and taken start-of-story Harry as a good example.
      Also, college Gen Physics destroyed me. It turns out that memorizing some equations and learning QM trivia does nothing to prepare one for lab reports or convincing oneself to do the homework now instead of some arbitrary future point in the more-free-time-than-ever-filled week.
      I’d’ve agreed with you prior to 2006, seeing as I hadn’t yet been confronted with how poorly I was going about it.

    • TracingWoodgrains says:

      Everyone “can” educate themselves, but my focus is on what people, when push comes to shove, actually do.

      The same kid who is fascinated by writing can be equally fascinated by Pokemon and anime. The kid who loves math can similarly love sitcoms and comics. And when it comes to a decision of what to actually do on a moment to moment basis, when choosing between two things they like, most people will pick the easier option. This applies to gifted kids every bit as much as it applies to everyone else. It’s why 90% of New Year’s resolutions fade out after January, why self-help is a massive industry, why online learning hasn’t revolutionized the world: People are really, really bad at doing the harder things they want to do. Some superhumans seem to manage it, or perhaps they’re better at faking than the rest of us, but most people need some degree of structure to make progress.

      Even for someone absolutely brilliant and motivated, they’ll usually make it further with help than on their own. Terence Tao is one of the greatest math minds the world currently has, but he went through a structured progression through a math curriculum just the same. A good teacher or coach can help provide a proven structure to help someone gain a skill, point out errors that the individual would miss on their own, and help the child maintain motivation and focus in the moment.

      Being smart doesn’t make a child less human, and unfortunately humans are pretty bad at doing a lot themselves. That’s why I push so hard for people to create structures that make it easier for us to do the meaningful things we want to do. It just doesn’t happen automatically in most cases.

      • mercutio says:

        Why is it intrinsically more useful for a kid to focus on math than on Pokémon? Seriously. Why is following popular culture interests worse than pursuing ivory tower knowledge paths? Particularly for a smart kid who can likely pick up the core curriculum at their leisure?

        I was advanced ~5 grades in math, and enjoyed it through my high school years (with my math classes provided mostly by the neighboring university).

        But I was treated with reverence as a math prodigy by teachers, which I feel, in retrospect, wasn’t really deserved or appropriate. So what that I brought some math competition trophies to my high school? The inappropriate reverence for math led me to being an arrogant prick in school, and I disdained popular culture in a way that wasn’t particularly conducive to building good social skills even if I’d had my obnoxiousness remedied earlier.

        Some of the social value of educating gifted kids just doesn’t resonate with me. If a kid is bored, I’m all in favor of letting them read on their own or offering help. Advancing grades in specific subjects was convenient. Kids who are clearly struggling should be given aid to find reasonable interests and outlets.

        But I would rather the education establishment focus on helping marginal kids, and left the gifted kids to their own devices. Let gifted kids just be kids, don’t track them or put them on a pedestal.

        • faoiseam says:

          I was treated with reverence as a math prodigy by teachers, which I feel, in retrospect, wasn’t really deserved or appropriate. So what that I brought some math competition trophies to my high school? The inappropriate reverence for math led me to being an arrogant prick in school, and I disdained popular culture in a way that wasn’t particularly conducive to building good social skills even if I’d had my obnoxiousness remedied earlier.

          I think that a certain amount of reverence for intellectual achievement is reasonable to balance out the reverence for non-intellectual, and especially sporting achievement. I think you would be shocked if you could live life again as the quarterback or prom queen, and realize just how little actual reverence you received in contrast with others.

          Some of the social value of educating gifted kids just doesn’t resonate with me.

          Most of the technological advances that we have gotten in the last few decades have come from people who earlier were highly educated gifted children. Gould said “I am somehow less interested in the weight and convolutions of Einstein’s brain than in the near certainty that people of equal talent have lived and died in cotton fields and sweatshops.” I think this suggests he believed that education was required for great achievement. I suppose it is possible that in the West we have reached the level where everyone receives enough education to perform optimally. I doubt this, however.

        • TracingWoodgrains says:

          I would divide activities less into popular culture versus ivory tower knowledge and more into active versus passive. Math is more useful inasmuch as it ends up helping the individual contribute meaningfully to useful projects, but it’s possible to do the same with Pokemon, perhaps through art, or storytelling, or mod creation. It’s not possible to do the same, though, sitting and watching TV all day.

          This is biased, of course, by my own experience: I tend to be a passive consumer far more often than an active creator, and I have several projects in mind that I simply don’t have the skill to do justice to right now. I’m poor enough at focusing on my own that I need structure to help me learn, so I have a vested personal interest in encouraging structures that help me personally develop those skills and focus on active over passive activities.

          I’m adamant about the need to push advanced students because I expect some of them, similar to me, to develop bad habits without challenge, instincts they’ll later regret. Things like the assumption they can skate by on their intellect, a reluctance to challenge themselves out of fear of looking stupid, the same arrogance you mention but without real skills accompanying it. A kid doesn’t have to be advanced five grades to notice math is easier for them than everyone around them. Ideally, they would be placed in many environments where they are clearly not the smartest or best in the room. Acceleration helps more than it hurts here.

          I’m not interested in putting people on a pedestal. I’m interested in helping people to develop and improve, and see schools taking an attitude similar to what you’ve expressed–let gifted kids just be kids–even when those kids are crying out for, and would be helped by, genuine challenge in their curriculum. Schools are intended to help people grow, and should be conscious of the growth of kids who just won’t find much value in a standard-paced course.

          • Matt M says:

            I’m adamant about the need to push advanced students because I expect some of them, similar to me, to develop bad habits without challenge, instincts they’ll later regret. Things like the assumption they can skate by on their intellect, a reluctance to challenge themselves out of fear of looking stupid, the same arrogance you mention but without real skills accompanying it. A kid doesn’t have to be advanced five grades to notice math is easier for them than everyone around them. Ideally, they would be placed in many environments where they are clearly not the smartest or best in the room. Acceleration helps more than it hurts here.

            Just want to say that this describes me as well, and I generally agree with everything you’ve said.

      • JohnBuridan says:

        Amen, Mr. Woodgrains.

    • In my experience with non-tracked classes in middle school, I lacked the concept of self-education. At that time, my mindset was that the adults would shape my world, telling me I had to sit here, do this worksheet, I could only choose to react in small, insignificant ways. It didn’t occur to me that I could try to use my time in class to accomplish a long term goal, I just sat there, bored, and occasionally chucked an eraser at the teacher. It was only in high school that I started to substantially educate myself through the internet.

  6. BlindKungFuMaster says:

    “It’s hard to argue with his results: two grandmasters and an international master, one of whom became the 8th ranked chess player in the world and the only woman ever to take a game off the reigning world champion.”

    The linked wiki-article says that she took a game of the reigning number one. Which is not necessarily the world champion. Also, I think this refers to a rapid game against Kasparov (when he was number one but not world champion). In which case you can either argue that she didn’t take a (tournament) game off him, or that there are more women who won games against world champions/number ones.

    Yeah, I know. But for me the whole Google memo thing was also mostly about how Damore claimed to be Fide master in his CV but actually only had a national rating of 1800.

    To also say something on topic:
    In Germany we have tracking, which overall seems to work pretty well. The sorting by ethnic group does happen though, in my first four years I had up to 50% Turkish classmates, after sorting that dwindled to maybe 5% (the schools weren’t close enough to each other for that to be typical, but on average the percentage does more than half).
    I used to think that tracking is a horrible idea that creates social strata that can’t even communicate with each other. Now I think that’s mostly my own social ineptitude and that if the average IQ in my class rooms had been a standard deviation lower, that probably would have been even less fun for me. Regular re-grouping by ability with different curricula would probably be the best thing until we get truly personalised education.

    • oqpvc says:

      In Germany we have tracking, which overall seems to work pretty well.

      What makes you say that? Globally speaking, education in Germany is mediocre. The early and mostly permanent tracking is according to PISA the main reason why educational success in Germany depends on the social background much more than in lots of other countries.

    • TracingWoodgrains says:

      I see your point there. Using “world champion” instead of “number one” was an honest oversight; I don’t follow the chess world too closely and wasn’t aware there was a notable difference between the two. Are there more women who won games against world champions/number ones?

    • educationrealist says:

      Germany has results that in America would be considered intolerable, that we would feel compelled to at least try to address. For example, Turkish immigrants do horribly. I imagine the newest group of MidEastern immigrants will do likewise very poorly.

      • oqpvc says:

        I’m not quite sure I can follow. Immigrants do on average perform worse than non-immigrants – most of which is accounted for by socio-economic background and language spoken at home IIRC, but to insinuate that these issues aren’t being addressed doesn’t seem warranted. Your phrasing also seems to suggest that the US has much better results and I’m wondering what you’re referring to.

      • dark orchid says:

        There’s a graph in Hirsch’s “The schools we need” (cited from Stevenson, Chuansheng and Shin-Ling’s “Mathematics achievment of Chinese, Japanese and American Children: Ten Years Later”) that added some perspective in the tracking/achievement debate for me:

        Japan has no tracking. Their achievement score on a particular mathematics test looks a bit like a normal distribution (mean 21.72, sd 6.59).
        China in this case means the Republic of China, which has tracking in maths. Their graph is bimodal with peaks around 18 and 34. (mean 24.10, sd 9.47).
        America … has a single peak with mean 13.39 and sd 7.06. Despite having no tracking (at least in the schools studied) the mean is below the lower of the two peaks in the RoC one.
        In each of the countries, N was a bit above 1000.

  7. Robert Jones says:

    The Joplin plan involves ability grouping — the good kind of ability grouping.

    I’m guessing this meant to say something else, but I’m not sure what.

  8. fion says:

    Very nice piece, and thanks for putting all the effort into preparing and writing it. I learned a lot from reading it. I have two questions:

    Do you have any objection to this being shared outside SSC? (I’m guessing not, since you didn’t say so, but just being safe…)

    Could you clarify in what sense this was “adversarial”? What things did you disagree on? (I think it’s a compliment of the piece that this isn’t obvious from reading it.) Have your opinions changed at all as a result of doing this?

    • Michael Pershan says:

      I intend on sharing this outside of SSC so I certainly have no issue with other people doing so!

      You can see our original terms of disagreement here, though once we confirmed via email that our disagreements seemed large and persistent, I requested not to offer any more clarity on my priors. On laying out my priors with confidence levels, I wrote: “I actually suspect that laying out my assumptions so clearly might make me more hard-headed later on in the discussion, so (unless the project calls for it) I’d rather abstain.”

      Looking back, I think I moved a lot towards acceleration, something that I hadn’t thought much about before and wasn’t really a fan of initially. I also have spent a lot of time professionally thinking about the needs of my most struggling students and this project forced me to spend a lot more time thinking about my students on the other end of the performance spectrum.

      There’s a lot more to say about the depth and shades of our disagreements and how they’ve changed and (mostly) clarified.

    • Randy M says:

      As far as I understand the project, adversarial collaboration isn’t about debating differences in the final piece, but bringing together people with different agendas or preliminary views of the evidence to present a balanced and accurate view of what is currently known in the subject. So we shouldn’t necessarily expect to see the adversarial nature manifest in the final pieces, except to strip out unsupported assertions from either side.

  9. Robert Jones says:

    But schools also know that they are held responsible for producing equitable outcomes for a citizenry that sees each other as equals. A program that raises achievement for top students without harming others has an appeal an economist could love, but within schools this can count as a problem.

    What is the mechanism by which schools are held responsible for producing equitable outcomes? Who is going to get penalised for raising the achievement of top students without harming others?

    • HeelBearCub says:

      Partly because the “without harming others” isn’t self-explanatory. If student A gets opportunities and instruction that students B, C and D do not, and the school is under any budgetary pressure at all (which it will be), then those opportunities necessarily come at the expense of others.

    • baconbits9 says:

      The current standardized testing automatically does this. Schools are largely judged on getting kids past specific, relatively low, thresholds. Getting enough children one step over a bar is explicitly worth something, getting them from one step over to two steps over is worth a ton less. Getting the kids who pass those tests easily a step higher is worth extremely little in many cases.

    • Matt M says:

      Probably not the teachers, but probably the principal, superintendent, or school board (whichever level made the decision to engage in tracking).

      Like everything else in life, it comes down to relative status. Improving the lot of one group without improving the lot of another group won’t be thought of as “Isn’t it great that we improved things for some people” but rather “How horribly unfair that these others are being left behind!”

      And it certainly doesn’t help that the others being left behind often cluster into racial, ethnic, or socioeconomic groups.

  10. jasmith79 says:

    I have always blamed my school-age social struggles on grade skipping and being almost 2 years younger than my classmates. In retrospect, after reading this with the skeptical mind of adulthood, I kind of wonder if it was just a ready excuse for poor social skills.

    • JohnBuridan says:

      Supposing you are male, I am one of those administrators who tell parents of male students they should not let them skip a grade level, if we can find another way to academically challenge them for the course of the year.

      Boys hit puberty later than girls and a boy with peers older than him will hit puberty later than the other boys. Socially this is a recipe for chronic social underconfidence and retreat. I have a 9th grader right now who is very smart and funny and half a head shorter than everyone else; he is constantly concerned about bullying, and bullying is his go to example of “bad things that can happen to a person.”

      P. S. I have no reason to suspect this peppy likable student ever actually gets bullied. It seems to be a hidden insecurity.

      • Nicholas Weininger says:

        Counterpoint: chronic social underconfidence and retreat in teenagehood is not the end of the world. I skipped first and eighth grades and so was 12 in ninth grade when everyone else was 14, quite definitely prepubescent (e.g. alto voice which didn’t change till the spring of that year) and very painfully aware of the difference. You are absolutely right about the social effects– I had very few friends and did not date until college and spent a fair bit of time crying over that. But my high school classes were amazing, challenged me in ways that would have been very difficult to replicate otherwise, and today I feel very socially successful (having found my wonderful nerdy niche in a world more friendly to nerdy niches than I could have imagined 25 years ago) as well as successful in life generally and wouldn’t trade my upbringing for any other.

        Note also that larger age differences can actually be less socially awkward, e.g. when I was 10-11 in middle school and took some high school classes with 16-17 year olds, I was a sort of odd pet/curiosity to my classmates and it wasn’t nearly as socially difficult.

        • JohnBuridan says:

          Thanks for the counterpoint! It always depends on the situation and individual tradeoffs people are willing to make.

          If those tears have, even indirectly, led to your excellent choral music, then let younger you know that they have brought this commenter joy. 🙂

          • Nicholas Weininger says:

            Thanks. 🙂 There may be an indirect connection in my choice of certain high-pathos texts. Certainly there is a connection in that choir was one of the highlights of my high school experience, in large part because the little social world of choir was so oriented toward the passionate discipline of music-making, and that was a discipline I was actually good at, so I got to both be higher-status than usual and to not have to worry about most of the usual rules of social interaction. I suspect sports teams play a similar role for differently-skilled kids. I wish every kid, of whatever skill type and level, the good fortune to find some such little world.

      • marshwiggle says:

        I’m male and my grade skipping was before puberty. It helped my social development. It didn’t give me equals – even after the skip I was blatantly better at academics – but it did give me people I could talk with. To be sure, I did end up in a role where all the girls treated me like a younger brother to be protected, which I can understand being frustrating from a dating market perspective. But protection was very welcome. Combined with sufficient devotion to improving my skills at semi-pacifist combat, that protection let me learn social skills without too much cost. The skip really did not damage my confidence. It made me a leader in both academic and nonacademic matters by giving me people with whom acting confidently had some chance of success.

        Of course, I wasn’t academically challenged even after the skip, so technically I would have passed your ‘can he be challenged without a skip’ test.

      • TracingWoodgrains says:

        Like the others who replied here, I skipped a grade, and the only lasting friendships I made during my school years came immediately after the skip.

        I guess that’s a big reason I’m personally skeptical of strong social concerns with grade-skipping: many kids who are in a position to need grade skips academically are already going to be the odd ones out socially. Being around same-aged students with alien interests, goals, and ways of interacting is no less isolating than being around older students, and there are some cases where moving to a group of older students can actually help.

        Of course, my own push would be to abandon a focus on strict age-segregated schooling, but in the meantime, grade skipping seems no more harmful socially than keeping the kid with their age group.

        • Matt M says:

          Right. I think the biggest social problem with grade skipping is that it’s so generally rare, it’s seen, in and of itself, as evidence of a kid being weird or strange, which potentially harms them socially.

          But if it wasn’t so uncommon, it wouldn’t be considered weird-by-default. If classes were, in general, sorted based on ability rather than age, nobody would really think twice about the fact that someone a couple years younger or older than them was in their same class.

          • baconbits9 says:

            I don’t think it just makes you seem weird, I think it can actually make you weird. I didn’t skip but had my birthday near the cutoff for my school year, and one of my friends through 8th grade was more than a year older than me and in the same class (I don’t recall why he had started a year later, but it isn’t uncommon) and I remember pretending to like girls, and even picking out a girl to have a fake crush on sometime around 5th or 6th grade because he was into girls before I was. I was also between short and below average height until I hit average height (a little over 5’9) at 19 or 20.

            Had I skipped a grade I would have been very short and very developmentally behind for all of my schooling, and graduated a few months before my 17th birthday, that would have been a lot more pressure towards the weird (and I was/am at least a little weird).

      • carvenvisage says:

        P. S. I have no reason to suspect this peppy likable student ever actually gets bullied. It seems to be a hidden insecurity.

        I realise it’s easier for a bystander to be vicariously paranoid and fastidiously correct than a busy administrator juggling educational balls, but the ‘optics’ of appearing approachable seem like a topic of professional-interest, so I’ll venture to tell you how badly it looks to me; which is like someone finding ways to paint a student’s concerns as silly, to ‘correct’ them on things they haven’t said (did they claim to be bullied?), and to dismiss the possibility of any bullying they haven’t personally witnessed or had reported.

        To be clear, this isn’t to assert that your level of concern, which surely has something to do with experience and eyes on the ground, is wrong, or that if a correct perception, it would be the end of the world (not every teacher has to fill every social role. if one person is the happy go lucky fountain of optimism, and another is the ‘serious listener’, well that’s two bases covered between the pair), only how this way of reporting the ‘phew, probably nothing’ reaction struck me. (-Very badly, but I realise what a limited (and primed in a certain direction) view I have.)

    • jasmith79 says:

      Yes, my own daughter is lucky that she gets to join higher grade classes for the couple of subjects she excels in but is otherwise with her peer group. I agree (at least based on my own experience) that skipping an entire grade rarely works out well. At the same time, I was still bored in my higher grade classes. They just didn’t have a good way to deal with kids like me 30 years ago. This was at a private school too, goodness only knows how bad I’d have had it at John Q. Public elementary school. I probably would have been fine with advanced Math/Science/English but with everyone else for everything else.

      And I did get bullied, but in retrospect not all that much. It helps that I’m a largish person (6’1″ 215lbs) and always have been so I was at least physically similar to the kids in the grade I was in.

      Definitely was lacking in emotional maturity though, and it showed.

    • HeelBearCub says:

      Interestingly, we held my younger daughter back for an extra year of kindergarten, as she had musculoskeletal issues. The pre-school teacher (who also happens to have been my Mom) had also wanted to hold my older daughter back.

      They are both as smart as whips. The older one struggled socially and mental health wise the entire time, my younger one much less so, even though they are both very much in the geek/nerd category.

      In retrospect, I probably wish we had held my older daughter back.

  11. ana53294 says:

    Accelerating can create a lot of problems for the social development of the child, or so they say. I wasn’t accelerated, even though I was bored and miserable, because I was lagging in my social development.

    When you move a 12 year old to a class of 14 year olds, there will be a couple of students who are 15-16 because they were forced to repeat a grade (this is very common in Spain for inmigrants who don’t know the language and academically lagging students). Kids go through a lot of problems when they go through puberty, and exposing socially underdeveloped kids to all these issues may lead to undesirable exposure to teenage activities (alcohol, tobacco, sex, drugs).

    The interactive explanation of the prisoner’s dilemma is great, but it looks like it took a lot of effort. I find that that is the problem with programs frequently; there are always ways to do a task more efficiently, but learning or writing a program that does it takes more time than you will spend on that task in an entire year.

  12. TracingWoodgrains says:

    Hey guys, TracingWoodgrains here. I’m not going to be available for the next few hours, but I’ll be around to join the conversation and answer questions later today. Feel free to ask here directly if you want clarification or further explanation about anything.

  13. Chalid says:

    If tutoring is such a great model, except for the expense, then why don’t more rich people use private tutors? IME, rich people seem to send their kids to a good traditional school and then maybe supplement with an hour or two a week of private tutoring. I don’t think I’ve ever met anyone who just hired a teacher to educate their kids nearly full-time.

    • ana53294 says:

      The really rich may not necessarily prioritize academic education, either. Sure, they will look for the best schools – but the criteria for those schools will include learning and other things, such as interacting with royalty and aristocracy, or other rich kids, or a diverse multi-racial group of peers, or whatever the parents’ priority happens to be.

    • jasmith79 says:

      What is the purpose of such education? It is usually to give kids a leg up in a competition that the rich have already won. Indeed, I would guess given the popularity of boarding schools amongst the wealthy that inculcating them in the performance of upper-classness is actually the goal of their education rather than academic achievement.

      Or maybe we’re talking about different levels of wealth? Do you mean the “works at a high paying job” wealthy or the “never worked and will never have to” wealthy?

      • Education Hero says:

        It is usually to give kids a leg up in a competition that the rich have already won.

        Some of my clients send their children to private boarding schools, and my experiences working with them suggest the opposite.

        To the contrary, the purpose is typically a desperate attempt to ensure that their children can replicate/exceed their parents’ socioeconomic success, in an uphill battle against regression to the mean, reduced motivation due to being raised in comfort, rising competition, and a rapidly evolving game board.

        Indeed, I would guess given the popularity of boarding schools amongst the wealthy that inculcating them in the performance of upper-classness is actually the goal of their education rather than academic achievement.

        The contrary evidence would be that elite boarding schools actually do have high academic standards.

        The standards may not be quite as high as (non-boarding) private schools dominated by the children of successful immigrants, but this is counterbalanced by increased school-sponsored extracurricular requirements.

    • AnthonyC says:

      At some level, the benefit from additional academic achievement through tutoring is much less than the benefit of interaction with well-connected peers.

      Similarly, when you go to a very highly regarded college with very competitive admissions, you generally learn the same material you’d learn taking a good free online course, but you don’t get to interact with a huge network of current-and-future-high-powered-individuals who can help you get future opportunities and advice.

      Unfortunately, no one explicitly tells students this, and so the people who need it most – the ones who *didn’t* already absorb this growing up in an affluent environment, who *don’t* already have vast connected personal networks – often miss out on the opportunity. I mostly fell into this group even though I had a very privileged childhood, simply because it wasn’t how my parents viewed the world and I never realized it was even a thing until later.

      • Tarpitz says:

        Couldn’t agree harder. I think I’m pretty damn good at my job, but my most important career advancement moment to date had nothing to do with my competence and everything to do with someone who was in a position to hire me having been a school friend of my brother. I don’t think that’s especially unusual (for people lucky enough to have gone to those sorts of schools).

        • Plumber says:

          From what I’ve read, attending a selective college has the least benefit for science majors, and the most for business majors in terms of increasing future income.

          Also, my main beef with resources going towards colleges is that those are resources that could go towards all citizens instead of a privileged few, but it seems that college benefits marginal students.

          High school was hellish, but my two academic (instead of vocational) classes at community college were great, my apprenticeship classes were better than high school, and my times sneaking into the Morrison reading room at UC Berkeley were even better. 

          If I was King of California I’d tax the Hell out of those who’ve bid up housing,  cut short high school, expand apprenticeship programs so everyone could learn a skilled trade, and provide a free liberal arts education befitting voters in a democracy (which would mean the end of my monarchy) for all those who want it, including food, housing and medical care.

          I want everyone else (especially the kids who were like me) to have what rich kids had, time to read and think.

          Also, open up those University libraries to all Californians.

          If that doesn’t happen then I want the Universities closed, their books dispersed to public libraries, and the professors made to teach high school students. 

          At gunpoint if necessary.

    • Chalid says:

      Response to all: there is nothing stopping our hypothetical rich person’s kids from socializing with other rich people’s kids. If there was a norm of tutoring among the very wealthy then this would be common. And I think it *used* to be the norm e.g. in Victorian times.

      I don’t know much about the really really elite private schools, so I don’t know what they’re like, but from where I sit it seems like the schools with the wealthiest student bodies also have the most homework and intense testing. (e.g. the $50k/year schools in NYC are pretty intense, as are the schools in the richest suburbs.) If one accepts that socializing is the reason for rich people to send their kids to school, then one would not expect this to be the case.

      (Here defining “rich” as “rich enough to hire a tutor without it being a major financial burden.” Looking at NYC that’s probably a family income around $500k/year and there are many tens of thousands of families that earn that.)

      • Viliam says:

        there is nothing stopping our hypothetical rich person’s kids from socializing with other rich people’s kids.

        At school, you can do this socializing thing every day without having to be too strategic about it. The school provides you common topics, common problems to overcome, opportunities to help each other… you just have to use them to create connections.

        the wealthiest student bodies also have the most homework and intense testing

        That too is an obstacle for an outsider to socialize with them. When you can’t go to a party because you have too much homework, you will still meet your classmates at school.

    • Plumber says:

      They do, but it’s considered “shameful”, if broadcast it would show that many of the rich are just rich, not a “cognitive elite”.

      Edit: Another link on the subjest

      • Douglas Knight says:

        Those articles are about tutors in addition to school, not tutors in place of school.

        • Plumber says:

          But if you just use a tutor then the child is only learning things, not “receiving an education”, which is a about getting a certification.

    • faoiseam says:

      The relatively rich people I know, almost all, have tutors for their children, often different tutors for different subjects. The kids still go to regular school, sometimes public, and the tutors, who are mainly teachers, tutor after school. As a result, teachers correct essays written, sometimes in part, sometimes completely, by other teachers.

      This starts in middle school, as it, according to the administration, is impossible to get into the top math track unless you have outside instruction. It is more common than not in top track classes by high school. The schools where I live do not have URMs, so there is no problem with tracking. Tracking begins in 7th grade or earlier. The earliest I remember is first grade with three tracks in English and Math. This is a little extreme.

      • Chalid says:

        Sure, but even the most heavily tutored kids are spending much much more time in traditional classrooms than with their tutors. Why not skip the traditional classroom, and have the kid spend six hours a day with a tutor?

        • faoiseam says:

          Most parents care deeply about getting their kids into a good college, and believe, perhaps correctly, that attending school is necessary for this. People send their kids to school so that a high school teacher can grade them, and say that they meet a certain standard. Many schools in the Bay Area primarily judge what children have learned outside school, as opposed to teaching children.

          Even the very, very rich do no more than take a year off during high school, for fear that admissions committees will disapprove. It is necessary to go to a good college to meet a reasonable mate, so even if money if not an object, people still need to go to college. It should be possible to get tutors for your child, and buy their way into college, but I do not know anyone who has followed that path. I know lots of people who bought their way into college, but none who saw the opportunity to use this as a way to skip high school.

          The closest I have seen is people who have taken high school online, because they were too rich to not live on a yacht, or because they were running a large Chinese company in their teens, so did not have time to go to regular school. This is rare, but more common that just relying on a tutor, as you still have an institution to issue a transcript, although online schools are not well received by admission committees, or at least, not by those whose internals I have access to.

          • Randy M says:

            It is necessary to go to a good college to meet a reasonable mate

            Are you saying this is the view of the rich, or that this is generally true?
            Do the rich actually tend to marry people they met in college?

          • Chalid says:

            I think I agree about high school being important to get into college, but that’s only four years out of 13 years of education. You can certainly get into a very good high school without going to a traditional middle school.

            That said, if it were a common thing for people with seven-figure incomes to educate their kids outside of the formal school system, colleges would change their minds pretty quickly about the need for a high school degree from a traditional school.

          • faoiseam says:

            The rich don’t necessarily marry people they met in college, but they do marry people from the social group they developed in college. Rich people sometimes marry late, but I cannot think of a pretty woman like situation in the people I know. All of the spouses come from elite(ish) colleges. The rich definitely would not like their children to marry someone who went to a third tier college, like the University of Idaho. I think that attending a college like that was one of the major knocks on Ms Palin.

          • Randy M says:

            I guess when you are rich it doesn’t matter if your sorting mechanism is economically highly inefficient.
            Rather, that’s probably part of the requirements for a sorting mechanism among the very rich.

          • Education Hero says:

            The inefficiency is a trade-off made to gain increased assurance that you are building genuine relationships.

        • John Schilling says:

          Because then they would spend six hours a day hanging out with a middle-class tutor when they could have spent that time hanging out with future rich people. And if you’re going to be a rich person, knowing lots of facts about math and science and even economics is probably not nearly so valuable as knowing a lot of other rich people. And knowing how to hang out with rich people and not look foolish or feel out of place.

          • faoiseam says:

            Perhaps on the East Coast it is possible to hang out with future rich people, but in the Bay Area, where I am based, the best you can hope for is to hang out with the children of currently rich people. The opportunity for this is limited, as the kids of the rich are hugely over-scheduled. Most, if not all of them, have a sport that they need to practice, which greatly limits the time for just hanging around.

            Sadly, the children of the rich look foolish and feel out of place when they do get to hang around with other teens, that is, when they are not have huge fits about some minor issue.

          • Chalid says:

            “Sitting in a classroom next to” is not the same as “hanging out with.” Most of the kids interactions are with their middle-class teacher.

            With tutoring, you could do all the traditional learning of a classroom in less time, leaving more time for whatever rich-person activities are seen to be most important. It’s not clear to me why four hours tutoring plus three hours a day at the local Harvard Club with other rich kids wouldn’t be lots better for socialization and for learning than spending those seven hours at Dalton.

          • Chalid says:

            Also a lot of this discussion (including some of my own contributions) is overstating the social class necessary to do this. If you’re dynastically wealthy, and are passing down your $5 billion dollar company to your heirs, then sure maybe the most important thing for your kids is to interact properly with similar people. But hiring a full-time tutor employee is the sort of thing that could be done by a family with two doctors, or two Googlers, or one moderately successful banker.

          • John Schilling says:

            “Sitting in a classroom next to” is not the same as “hanging out with.” Most of the kids interactions are with their middle-class teacher.

            Except for in the lunch room, at recess and on sports teams, in the library or study hall, in the hallways, during various group activities, and even illicitly during the lectures.

            For most students who aren’t nerds, there’s an awful lot of social interaction in traditional schooling, much of it with peers or near-peers, and a great deal of practical education in applied social skills. It is one thing to legitimately criticize schools and teachers for taking a sink-or-swim approach in this area and ignoring the kids who sink. But the ones who swim are getting real value, and it is equally wrong to tell their parents they are doing it wrong and should be hiring tutors for private academic education.

          • Lambert says:

            If kids are interacting with the teacher more than their peers, I doubt they’re being taught all that well.
            There’s only one teacher to go around, but plenty of other kids to bounce concepts and ideas off.

          • Chalid says:

            Yes, yes, no one denies that there is some amount of social skills practice that occurs in a traditional school, but it’s substitutable for a lot less than the cost of hiring a tutor to handle the academic component of school.

          • Chalid says:

            If kids are interacting with the teacher more than their peers, I doubt they’re being taught all that well.

            Maybe, but sitting and listening to a teacher give a lecture is a very large part of any normal school.

          • Randy M says:

            overstating the social class necessary to do this

            Being able to support a full-time professional in your employ (without making any money from their services) qualifies one for very rich in my book.

          • Chalid says:

            Being able to support a full-time professional in your employ (without making any money from their services) qualifies one for very rich in my book.

            Completely agreed. But I don’t think that it is extremely important for the child of two doctors or a couple mid-level Googlers (combined family income plausibly ~$500+k in both cases) to hang out with the children of very rich people. The social class where that is extremely important is way higher, to the point where I don’t even have a clear idea where it would start.

          • thevoiceofthevoid says:

            @Chalid

            Yes, yes, no one denies that there is some amount of social skills practice that occurs in a traditional school, but it’s substitutable for a lot less than the cost of hiring a tutor to handle the academic component of school.

            What exactly is your proposed substitute?

          • Chalid says:

            Depending on age, inclination, and parental resources, offhand, you could do some combination of team sports, scouting, activity clubs (e.g. chess, D&D, hiking, martial arts, music…), volunteering, camps, a bit of more traditional schooling (selected classes that are best done externally), church stuff, science/math team competitions or projects, meetups for particular interests, part-time work, interacting with the parents’ social group and their children…

        • Plumber says:

          Because they’res not enough tutors for every student.

          Or are you just talking about the kids of rich parents?

          If that’s the case, I really don’t care.

    • Most children of the rich are not gifted, but are merely above average in intelligence. But for those who do have Ricky and Morty tier IQs, I’d bet they are much more likely to have a private tutor or skip a grade or graduate high school at fifteen than equally smart kids born to middle class parents.

  14. IdleKing says:

    This reads to me like a “position paper” more than an adversarial collaboration. I.e., it seems like the authors are making a strenuous case in a specific direction. As it happens, I agree with that case — but I also know that there are people out there who strongly disagree with it (“tracking” is considered a dirty word in some US education circles), and this write-up doesn’t help me understand why.

    Maybe that’s fine — maybe the “other side” doesn’t have any real evidence on their side. But it makes me curious about the process that generated this piece. Authors — how much real disagreement was there between you? (Did you discover that you basically agreed with each other from the beginning? Did Michael end up adopting TracingWoodgrains’ perspective fully? Or am I misreading the diversity of perspective represented by this piece?)

    In any case, even if it is a position paper, it’s a really well-done one!

    • fr8train_ssc says:

      Seconded. This was great, but I’m curious which (or both) of the authors “changed their mind” or took consideration of an opposite viewpoint, what original evidence they had that supported their viewpoint (even if it’s just media reports or something that inspired the position rather than defend it) and then how they came about to change their mind.

    • Randy M says:

      Why people oppose tracking, from this piece:
      It doesn’t offer any improvement for most students.
      It increases segregation along racial lines.
      It could effect students self-esteem and life options based on early (presumably fallible) assessment.
      It may inhibit the equalizing effect of schools, which are used for more than simply maximizing learning.

      • IdleKing says:

        Yep, but it reads to me like an article written by an advocate for “tracking”, who is trying to be fair by occasionally describing some of those alien arguments for “not-tracking”. It doesn’t read to me like an article that an opponent of tracking actually signed off on.

    • AG says:

      I’m guessing that, a lot of times, the adversity is in definitions. Hypothetically, A opposed anything that looked like tracking, B was for anything that looked like tracking. By showing each other the evidence they gathered, they were able to find a more nuanced form of ability grouping. A learned that the studies that showed the harms in tracking only applied to certain executions of it. B learned that certain executions of tracking caused harms. They learned that maybe “tracking” is too vague a category, and that the devil is in the details.

      I expect most adversarial collaborations will conclude this way.

      • IdleKing says:

        Yep, that sounds great. I’d find it helpful to have an explicit account of that process from the authors.

    • Aftagley says:

      I’d like to second this. Adding an opening summary of what each author’s position was and what they say as the matter of disagreement they were collaborating in spite of would, IMO, improve the contest a bit. (potentially a point about how this experience possibly shaped the author’s opinions, if at all, would also be nice.)

      Maybe I’m thick, but this piece was so seamlessly written that I didn’t really see a single point about which the two authors weren’t on the same page about.

      • Michael Pershan says:

        For better or for worse, we worked really really really hard to write an essay that we were on the same page about, despite our deeply different perspectives.

        • Scott says:

          I think it would be really helpful as a postscript to this piece to know what those differing perspectives are. After all this collaborative effort and research, where do you two still disagree in substance and in interpretation? It would be interesting to learn more of the background of the two people who wrote this piece, but more than that I think it would really strengthen the core message and recommendations of this piece to put it in contrast to where things aren’t so clear cut.

          • Michael Pershan says:

            I think TracingWoodgrains is much, much more optimistic than I am about the future potential of personalization software. I’m essentially pessimistic about the idea and think there is better potential for progress in other areas of education. TracingWoodgrains is essentially optimistic.

            I think TracingWoodgrains is much, much more excited by individual success stories like Tao or JSM than I am. I don’t really feel as if I know what we can learn from them, though I feel the cases are interesting to think about and speculate on.

            I think that given the collective desires for school there is not a lot of room for school to look radically different while still being successful. I’m passionate about trying to figure out ways to do good within this system. I think TracingWoodgrains is passionate about rethinking this system, and is much more optimistic than I am about experiments that would e.g. disrupt age-grading and allow students to progress through school at their own paces.

            Our biggest area of agreement is that advanced students have needs that don’t get enough attention in education.

            I don’t know — does this sound right to you, TracingWoodgrains?

          • TracingWoodgrains says:

            Yeah, that sounds like a great summary of the biggest differences we kept running into. I would add another difference, not in core beliefs, but priorities coming in: as a frustrated student, my focus was more on advanced students and the ways the school system slowed them down; my impression is that, as a teacher, you tended towards much more focus on struggling and disadvantaged students. I think we’ve both always agreed that both groups are clearly important, but that sort of priority difference has a lot of impact in a discussion like this.

    • IdleKing says:

      After reading the authors’ responses here, I realize that my list of potential disagreement-narratives was missing an option. The authors discovered a subset of the topic on which they substantially agreed, and wrote a “position paper” on that.

      I think this piece is fantastic, and I’m grateful to the authors for it. But it’s not what I thought an “adversarial collaboration” meant. I’d be really interested to read a report by these authors on the broader issue on which they disagree (something like “would the education system benefit from an attempt at radical restructuring along such-and-such lines”). That way, if they can agree on how to describe each other’s arguments, I’ll trust that the end result gives a good exploration of at least 2 sides of the question.

      • Michael Pershan says:

        What’s important to me is that the subset we agree and wrote about DIRECTLY informs the region of our disagreement.

        I actually think we hewed pretty closely to the call for an adversarial collaboration, so I’m trying to think what is throwing some commenters for a loop. I think it might be that we took care not just to find facts we could both live with but also narratives to agree on.

        I think we chose to look for narratives for a few reasons. We wanted people to read our thing and enjoy reading it. We also wanted to search for a way to understand our own disagreements. Why was it that we kept on agreeing on the importance of a study and then taking it in wildly different directions?

        For example, I now understand TracingWoodgrains’ optimism about edu software in a much deeper way than I did before. He agrees with the narrative that the existing edtech landscape is a disaster and that the present is largely repeating the efforts of the past. In fact, that’s WHY he’s optimistic — good stuff has hardly been tried. (Whereas I think that there’s a good reason why good stuff has hardly been tried.)

        That sort of thing. Tell me if I’m wrong or mangling your views, TW.

        • IdleKing says:

          It’s awesome and impressive that you found narratives you both agree with. And you make a great case for those narratives and support them well with really excellent research. I guess I expect something different out of an adversarial collaboration.

          For example: let’s say a nuclear power advocate and an anti-nuclear activist were going to collaborate on a report. Option A: they write up all the arguments for and against building more nuclear power plants, compromising to find a description of each other’s arguments that they can both agree with. Option B: they discover that they both agree that the current nuclear-waste-handling system is dangerously outdated, and write up an impassioned, persuasive argument for change on that narrower issue. I can imagine wanting to read either of these. I’d only call Option A an adversarial collaboration.

          Here’s a question: would Adam Gamoran sign off on your piece as a fair representation of the state of the debate?

          • TracingWoodgrains says:

            I hope so! I don’t imagine he’d agree with all of our conclusions, but specifically on tracking, we didn’t go in agreeing on a position at all. We did as thorough a literature review as we could manage, and only really formed solid conclusions after examining it all. It’s not perfect, and both of us are amateurs, but I’m confident we have presented a factually accurate look at the state of the debate from the knowledge base both sides are working with, even while forming conclusions about it.

            I’d be intrigued to hear Gamoran or someone similar comment on it–I’m sure he’d find places to disagree, particularly with our refusal to condemn tracking, and could help us refine it further, but I’m confident in our presentation of the issue.

            For Michael’s earlier piece that focused specifically on tracking, see here.

          • IdleKing says:

            Cool. So at the start you didn’t agree on tracking, and to some extent you still don’t — but this report contains the elements on which you can agree (on that and other elements of the broader school-disruption debate). Specifically, one place you really agree is in the idea that the current school setup isn’t doing right by gifted students. So that’s the focus of the piece — that’s the “shared narrative” — but as part of that narrative you bring in lots of other research for which you have a shared interpretation. Is that a good description of what’s going on?

          • TracingWoodgrains says:

            Yeah, that’s correct.

  15. Athreeren says:

    So what is the effect of cross-grade ability grouping in terms of racial and social segregation? Are the highest classes all white? What is the age range in the lower levels?

    • AG says:

      In America, you’d have a concentration of Asian Americans.

      I went to an accerelated high school program that gathered top students in the state (a Southern state), and the modal last name was “Chang,” though we joked that was only because you had Li and Lee counted separately.

  16. quitelikelyblog says:

    That was a great read. Definitely sells me on the “adversarial collaboration” model of creating summaries of areas of knowledge. Keep em coming!

  17. ADifferentAnonymous says:

    Excellent work!

    Had there been any research on extrinsically-mitivated online learning, i.e. paying people for passing courses? Obviously there are lots of ways it could go wrong, but curious what’s been tried.

    • matthewravery says:

      Traditional models of education refer to this as “Scholarships”. I’m not sure I’ve heard of scholarships specifically for online coursework, though.

  18. gwern says:

    Coincidentally, I’ve been putting up fulltext of all the SMPY papers/books if anyone wants to look into it in more detail: https://www.gwern.net/SMPY

    Here’s one way this manifests itself: in one study, 44% of poor students identified as gifted in reading in 1st Grade were no longer academically exceptional by 5th Grade. For higher-income families, only 31% of 1st Graders experience this slide.

    Isn’t this already explained by regression to the mean?

    • JohnBuridan says:

      Good question. Two of my priors are in conflict with each other here.
      1) PreK – 4th grade students develop at non-linear highly variable rates cognitively and physically, but some type of developmental regression toward the mean-rate-of-development occurs.
      2) Students who learn to read well early, easily retain and even expand their advantage years into the future, provided they continue reading.

      I am not sure how to resolve this. -_-

  19. educationrealist says:

    THis is a great piece that I hope gets published elsewhere. As mentioned, I responded to early drafts, and I can’t find my comments, Michael, so if you could send me the link to the draft, if you have it, just so I could dig up my comments I”d appreciate it..

    “this write-up doesn’t help me understand why.”

    Then you don’t understand the import of “there’s the potential in a racially mixed school that ability groups will effectively sort Black students into the lowest track and expose them to a lot of dynamics that are difficult to quantitatively measure but frequently discussed in education.”

    I think they didn’t go further on this because they understand that most readers know exactly what that means, but perhaps they didn’t understand the degree to which this statement doesn’t sink in. I also don’t read this piece to be making a strenuous case for acceleration or even tracking. (those are my answers, not the authors’)

    To put my subsequent observations in some context, I am a teacher of non-gifted kids in three subjects. I raised a kid with an IQ in the 130s/140s. My own IQ puts me in the 4+SD BGI study of a few years back.

    First, and this can’t be stated enough: the fact that a bunch of obnoxious people on the internet were bored in school is simply educationally irrelevant. In a this article, I point out that in my own life and my son’s, as well as my experience with bright kids in school, I see nothing but teachers being supportive of bright kids. I’m sure there are exceptions, but the more likely common factor is a bunch of wise-ass kids with crappy social skills who probably aren’t as bright as they think they are. Now if you’re the parent of such a kid, who is genuinely miserable and acting out in math class and causing trouble because he’s not allowed to move ahead, you can pay a bunch of money for private school or you can pay for an enrichment tutor, but in either case you should start by making sure your kid behaves in school so that others can learn. I’m sure there will be all the sob stories following this, but please save them. Seriously. I get it. You were miserable. So? There’s no evidence you would have achieved more had you been able to rush through the curriculum. Get over it. There is an argument for the exceptionally bright, but few people complaining qualify for that.

    I’m just not a fan of acceleration in the current education environment, because acceleration just means giving kids more math earlier, rather than give demanding tasks that only bright kids can take on with the math they know. And this is a problem for two reasons. First, sending a ton of math one way isn’t something that only bright kids can learn, it’s also something that swots can learn. And there’s a big difference between “bright” and “a swot”, but right now, our math system in America is not only actively disadvantaging bright kids by going so fast, but rewarding the dutiful swot. So if you have a bright kid, you’re not helping them much by letting them take algebra in sixth grade. And we have ABSOLUTELY no concept of how to educate bright kids verbally, but then most of these efficiency nuts don’t give a damn about that, they’re all about “I could have had calculus when I was was 8 but for those damn teachers!”

    If you consider the exceptional math intellect, we know how to educate them, and they’ll figure out the challenges for themselves. They’ll find the question. But that’s not the average bright kid. And we don’t really know how to educate the average bright kid, IQ of 130-145.

    In fact, the biggest problem we have right now with bright kids is that they fold at the very mention of a challenge, because they’ve spent 8 years in elementary school bored because they “got” everything. But in fact, these kids spend a lot of time avoiding anything they don’t “get”, because they get uncomfortable. That’s why the good teacher, and the good parent, will find something that kid doesn’t know how to do and can’t easily master, and make him or her miserable.

    Because remember this: some kids learn faster than others, but there’s a wall out there. If you treat your kid as brilliant and accelerate him through middle school, he or she could hit a wall and learn oh, awit, I’m just kind of ordinary. That’s a really tough lesson to learn at 15, particularly if your parents set such a store on how brilliant you were. Past tense.

    I know I had more thoughts on this but need to dig up my responses, if I can.

    What I see as the tension in this paper is between Woodgrains, who I believe is the pro-acceleration person, and Pershan, who understands (like most teachers do) the complexity of the task of education–that is, education is not about efficiency in America. Love the mention of Labaree–I’m reading How to Get Educated Without Really Learning. He’s on the right track.

    The paper’s value lies in this understanding of why schools and teachers aren’t always eager to accelerate, and in the balanced treatment of bright kids.

    • Randy M says:

      he or she could hit a wall and learn oh, awit, I’m just kind of ordinary. That’s a really tough lesson to learn at 15, particularly if your parents set such a store on how brilliant you were. Past tense.

      Hard, but seems better to learn early than later.

    • Jiro says:

      First, and this can’t be stated enough: the fact that a bunch of obnoxious people on the internet were bored in school is simply educationally irrelevant. In a this article, I point out that in my own life and my son’s, as well as my experience with bright kids in school, I see nothing but teachers being supportive of bright kids.

      “A bunch of obnoxious people on the Internet” went to different schools than you and your son. Just because you personally saw something doesn’t mean it’s the rule or even typical.

      • oqpvc says:

        You ignored the important part:

        You were miserable. So? There’s no evidence you would have achieved more had you been able to rush through the curriculum. Get over it. There is an argument for the exceptionally bright, but few people complaining qualify for that.

        I have my own sob stories to tell, but the question is if they’re relevant enough to demand change. School curbed my curiosity and fucked me up pretty bad as long as I attended – and then I graduated, went to uni and pursued a PhD. There’s nothing to suggest that a more pleasant school experience would have made me more successful. educationrealist’s argument is that this is a typical scenario for gifted children who are unhappy in school. It’s university folklore that a genius will survive every university system and eventually achieve greatness, no matter who it caters to – which is why we focus on maybe the 60th-90th percentile. So maybe the same is true for school?

        While a nicer experience in the school system for gifted kids is certainly nice and contributes to overall happiness, there’s the question how relevant it is: How many gifted kids are underachievers at school? How many of them remain underachievers afterwards? Are there persistent negative psychological effects? How expensive would measures to mitigate this be?

        • The Nybbler says:

          At least be an advocate for your own interests. If poor children or minority children or female children or disabled children were to be “fucked up pretty bad” the whole time they were in school, there’d be groups all over that, and in fact there are. And while some may deny there is such a problem, nobody says that if there is, these children should just suffer and it’s not an important problem. Certainly no one expects _them_ to think so, even after the fact. So why should it be different for gifted children?

      • matthewravery says:

        I, too, was struck that a lot of the premise for this article (ie, Section 1) boiled down to a self-selected group of folks from two of the weirder parts of the internet (SSC reddit and Twitter) complaining about how unfair the world was to them when they were 12. There wasn’t any effort to quantify how many people are in this potentially under-served category nor how much harm was done to them that could’ve been mitigated via the approaches proposed. Nor was there any investigation into the costs of this approach. Are resources mis-allocated? The tone of the article read like they thought a lot of effort was being spent on under-performing students, but I don’t know if the authors thought it would have been better to spend this on the one-in-10,000 students instead.

        I also feel like the reactions in this thread also missed something. The point that math skill and conscientiousness aren’t super-well correlated didn’t seem to sink in. If you’re 3-4 SD above the mean in math, you’re probably not 3-4 SD above the mean in conscientiousness. The fact that you observed that you struggled later in life when you “hit a wall” and things were no longer trivially easy isn’t the system’s fault.

        So all of that sounds pretty harsh. It shouldn’t. I enjoyed reading this. I thought the writing was clear and easy to follow. It appears well-researched. I feel like I have a much better understanding of this topic from having read this. I learned something that was contrary to what I thought was true: Mixing students of different achievement levels in a classroom does not improve anyone’s performance. (My belief was the opposite going in; I’m pretty sure a teacher friend of mine had told me that 10 years ago.)

        So “Bravo!” to the authors! A good effort, and surely more than my lazy ass would bother contributing.

        One suggestion that I think may have helped with some of the issues I pointed to above: Your question is ill-posed. “Adequately serve” is a squishy metric. Nail it down better with something like, “Should more educational resources be allocated to advanced students?” And you could even go further and lay out exactly what “advanced” means.

        • TracingWoodgrains says:

          Glad you enjoyed reading it!

          We could have been more explicit in the essay in mentioning that we worked to quantify the amount of people in that particular under-served category, but finding reliable numbers is extremely difficult. By definition, these students perform well on a lot of standard measures of success in school, with high test scores, usually decent grades, and higher than average outcomes later in life. IQ or its proxies are rarely used as a demographic measure for surveys or research, and even research into the specific phenomenon of gifted underachievement typically focuses on non-school factors such as home environment. It’s a blind spot in the research that’s proven difficult to quantify.

          The question was added, I believe by Scott, to summarize our topic for posting here. Our initial areas of investigation were primarily ability grouping, the role technology and gamification in education, and the merits of different teaching styles. It evolved in a roundabout way into the final topic as seen here. The question you propose would be an excellent one for a focused collaboration.

    • The Nybbler says:

      I’m sure there are exceptions, but the more likely common factor is a bunch of wise-ass kids with crappy social skills who probably aren’t as bright as they think they are.

      While I’m sure it’s fun to tell people that they (or their kids) ain’t that bright, it seems less than useful. What evidence would you accept that someone _was_ that bright?

      In fact, the biggest problem we have right now with bright kids is that they fold at the very mention of a challenge, because they’ve spent 8 years in elementary school bored because they “got” everything. But in fact, these kids spend a lot of time avoiding anything they don’t “get”, because they get uncomfortable.

      And why would acceleration not alleviate this problem, by getting all the stuff the kids “get” out of the way so more advanced stuff that they can’t immediately “get” can be taught on top of it?

      • educationrealist says:

        Because acceleration doesn’t mean you do harder stuff, just the usual stuff faster. It often isn’t challenging at all.

        I didn’t say they weren’t bright.

        • The Nybbler says:

          Once you’re done with “the usual stuff”, would you not move on to harder stuff? Or are you saying the kids get through the entire primary and secondary curriculum without ever hitting anything hard?

          I didn’t say they weren’t bright.

          You said they weren’t “as bright as they think they are”.

          • educationrealist says:

            Yes, which isn’t the same thing.

            And yes, kids get through the entire primary curriculum without hitting anything that your average bright kid would consider difficult. I wrote about that here. And moving faster through material doesn’t necessarily help, because not everyone remembers what they learn. Going deeper, rather than faster, would be best for bright students.

          • faoiseam says:

            I have never understood what “going deeper, not faster” means in math. I would have thought that deeper meant going further, but it seems that there is some other meaning to deeper, maybe doing more word problems.

            I very much doubt that there really is a deeper form of Algebra 1 than Algebra 2.

          • helloo says:

            It could just mean trickier and less formulatic problems.

            As in, a basic probability question is
            “Given a fair 6-sided dice. What is the P(dice count >3)?”
            Slightly harder-
            “Given 3 rolls of a fair 6-sided dice. What is the P(sum >10)?”
            Harder-
            “Player A rolls 2 fair 6-sided dice. Player B rolls 3 fair 4-sided dice. What is the P(sum of A > sum of B)?”
            A much more challenging one would be
            “Given 4 rolls of a fair 6-sided dice. What is the P(sum of the three highest >10)?”

            There are more advanced math than may help answer (or simply/formulate an equation) the latter ones, but it isn’t necessary.

            I’m not sure that this type of “deepness” is all that useful in actually teaching people, but at least should prove more challenging/interesting.

          • marshwiggle says:

            Deeper can include mastery of the material instead of a superficial understanding – being able to independently apply it or extend it in nontrivial ways.

            Deeper can include being able to prove why a method works instead of knowing how to execute it. Better still would be deriving the method instead of being given it.

            Deeper can include being able to use the method with a 99% success rate on problems with large and inconvenient numbers/expressions instead of a 90% success rate on problems with easy ones.

            But with any of those things you’ve got to start asking what fraction of math teachers are capable of them.

          • faoiseam says:

            I don’t really understand what mastery is as applied to solving a linear equation in one unknown. I suppose some students could solve it with small integer values, but have problems with fractions, but that does not sound like a problem with linear equations, rather it is a problem with arithmetic.

            I totally see how being able to prove something works is beyond knowing that it works, but I cannot see how this can be applied pre-Calculus. I can prove that the real field is Euclidean, and the rationals are not, but sixth graders are unlikely to appreciate a proof, at least until they hit Geometry, when proofs are introduced in the US system.

            Getting to 99% accuracy seems like practicing rote computations. I knew a student of Curry, who was once locked in Curry’s office and made reduce combinators for 3 hours. Half way through, Curry’s wife unlocked the door and brought the student a cup of tea, and said: “My husband is not a bad man.” There is some benefit from being able to compute things, but it can be taken to extremes.

            The example “Given 4 rolls of a fair 6-sided dice. What is the P(sum of the three highest >10)?” does seem harder, but seems to rely on more math than the others, unless the intention is to write out the 1296 possibilities and count the ones that add to more than 10 (and divide by 1296). If each problem is done in the same way, counting the positive instances, and dividing by the number of different rolls, they are all very similar, save for how high the student needs to count. If the student uses tricks to simplify things, then they are learning combinatorial tricks, which are a subject in themselves.

            Alcumus is a great resource for learning tricks, but my experience is that it teaches competition math, which is fun but consists of tricks and recognizing certain patterns, rather than additional content over regular math classes. These tricks are the stuff of later math, but in no way are a deeper kind of algebra.

            Sadly, getting math teachers that can teach material that is not on the syllabus is very hard in high income areas, as anyone who can go beyond a text book can be paid a lot more in other jobs.

          • marshwiggle says:

            My standard test to detect weak algebra skills is to have people solve mgh=1/2 m v^2 for one of the variables. You’d be surprised how many people can’t, because it isn’t exactly the same as the algebra they were given. Have you really learned the algrebra if you can’t apply it? Ok, that one isn’t linear, but I bet plenty of people who fail the quadratic would fail a linear with variables. I bet plenty of people who fail the linear with variables could do all kinds of precalculus stuff as long as the problem looked exactly like the ones they were given in school. It is a real problem.

            As for 99% accuracy with hard numbers, getting there is the opposite of rote learning, at least beyond arithmetic. For most kids it requires developing mental processes, checks, and ability to move numbers around which easy numbers and accepting small numbers of errors does not. It’s by far my least preferred route to getting depth – it is simply in there because it is the easiest for a bad math teacher to implement.

          • Sniffnoy says:

            So, as for the question of what “deeper, not faster” means, Andrei Toom wrote quite a bit about this here (warning: 98 pages long, although there seems to be a 23-page version here.)

          • helloo says:

            Probably should have just made the last one = 10 and not >10.

            There’s multiple ways to approach it and the 3rd one (which is even bigger matrix if you try to solve it raw 6^2*4^3).
            One way would be to first see that it’s equal to P(sum-lowest) = P(sum = y+x)*P(smallest=x | sum=y+x) for each y > 10 and dice x.
            Another would be to assume one die is the lowest and calculate P(sum of 3 > 10 | dice all >= x) for each x and remove the duplicates.

            It’s still rather calculation intensive but much less so and requires some non-trivial insights. Finding these insights and tricks aren’t really taught at any level of “standard curriculum” besides maybe college elective courses.

            And though you mention that these are for competition math – are competitions not how people generally find and “quantify” highly talented mathematical individuals? Or do you feel that doing better at those are not proof they are more capable of math in general?

          • faoiseam says:

            And though you mention that these are for competition math – are competitions not how people generally find and “quantify” highly talented mathematical individuals? Or do you feel that doing better at those are not proof they are more capable of math in general?

            Competition math is very much a clique. When Math Olympiad tutors meet, they compare Erdos numbers, which should give you a sense of quite how small the community is. Especially for girls, Math Olympiad is concentrated in a very small number of schools, which is sad, as it is much more widely applicable. The biggest issue in growing the number of people involved is access to tutors who can do it.

            I don’t think, notable exceptions aside, that most people who are good at math are identified by Math competitions. Perhaps 5% are found that way, but I do not have hard numbers, and my guess is biased by the math department that I know best, which might skew strangely.

            There are other ways to approach the problem you gave, that don’t use conditional probability, but rather use some combinatorics to count the number of ways of 4 things adding to 10 (or less than 10) as the smallest number must be 2, as 3*4 is 12. This gives two paths, one of which must be 2,2,3,3 or 2,2,2,4. There are 3! ways of arranging these choices, etc. So students could get by with knowing factorial and some combinatorics, rather than knowing conditional probability. Either way, this relies on the student going faster, that is learning new things like conditional probability, rather than deeper.

        • beleester says:

          Isn’t the seventh-grade “usual stuff” typically more challenging than the sixth-grade “usual stuff?” Meaning that doing the usual stuff faster will eventually get you to a grade level where the stuff you’re doing is challenging?

          How are you defining “harder stuff” such that it doesn’t vary based on grade level?

    • Plumber says:

      I have little doubt that most children could learn academics earlier (and probably better) under the right conditions, but the right conditions require a great deal of one on one instruction from a good teacher for most of a child’s waking hours, instead of one adult with 20 to 40 kids for a few hours, a portion of the year.

      Besides being avalible, and a good teacher, there’s a limit due to the shortage of adults who know the subjects in the first place to overcome.

      If, however, the goal is to teach how to duck, dodge, run, what it feels like to be punched into unconsciousness, plus how trapped we are by the social class we’re born into, then the schools I attended did a fine job.

    • John Schilling says:

      I’m sure there are exceptions, but the more likely common factor is a bunch of wise-ass kids with crappy social skills who probably aren’t as bright as they think they are. […] Seriously. I get it. You were miserable. So? There’s no evidence you would have achieved more had you been able to rush through the curriculum. Get over it. There is an argument for the exceptionally bright, but few people complaining qualify for that.

      OK, but I’m one of those people. This blog disproportionately attracts those people. And this post is about how the educational system ought to serve those people. Do you have anything useful to say here, or are you just using this as an opportunity to dump a canned rant on one of your pet peeves?

      • educationrealist says:

        The point is that the paper, by taking these complaints seriously, is beginning from a flawed premise that it happily overcomes. You are, from an educational policy standpoint, irrelevant. You aren’t a problem that needs fixing because it’s fairly obvious that bright kids are well-served by public schools, even if a small percentage of them feel poorly treated.

        Moreover, bright kids are given HUGE educational subsidies for post-secondary education, whereas non-bright kids are not.

        • The Nybbler says:

          Alternative view: bright kids are ill-served by public schools, but being bright they still get better out of this ill-service than less-bright kids do.

          • educationrealist says:

            If you read what I wrote in its entirety,you’ll see that I think we can do better with bright kids. And both bright and not-bright kids do well out of our system.

        • John Schilling says:

          @educationrealist: Yep. You’re deliberately trying to be as condescending and insulting as possible without being conspicuously rude or banworthy, and IMO failing on the “conspicuously rude” part. Enjoy the feeling of smug superiority, which is the only positive thing that will come of your presence here, but I’d rather you go someplace else for your next dose.

          • educationrealist says:

            I have no idea what bug got stuck somewhere in your alimentary system, but I’m not trying to be condescending and I don’t really care what “a lot” of the people here are like. I’m talking about educational policy writ large, and all over the internet, not just here, are people who think it’s somehow school’s problem that their school lives sucked.

            And I spent quite a bit of time commenting on and suggesting edits to the article, so I’ll continue commenting here. I don’t believe I’ve been rude to anyone.

          • Mark V Anderson says:

            @John. I very much appreciate ER’s comments. After hearing comment after comment on this thread by folks that felt ill treated by schools that didn’t challenge them, it was time for a contrary opinion. To me, SSC is about folks having unpopular opinions that might get one banned elsewhere. Instead they start good discussions, as long as the unpopular opinion is written in a coherent and rational way, and not just as a rant. That’s what ER has done here.

            I would like ER to explain his comment that these complaints are irrelevant to education policy. It is kind of rude to make comments against others’ postings like that without explaining it at all. But otherwise, ER is making a valuable contribution, and I don’t want him to go away.

          • The Nybbler says:

            I would like ER to explain his comment that these complaints are irrelevant to education policy.

            Seems clear enough — the advanced students are smart people, they’ll learn enough. That life sucks for them for 12 years and they could learn a lot more is not relevant to education policy, which is about getting as many people as possible to hit the minimums, not running up the numbers for those well over the bar already.

          • arlie says:

            Seems clear enough — the advanced students are smart people, they’ll learn enough. That life sucks for them for 12 years and they could learn a lot more is not relevant to education policy, which is about getting as many people as possible to hit the minimums, not running up the numbers for those well over the bar already.

            Why? Is it because the people making the policy dislike “elites” and want to reduce everyone to the lowest common denominator? Is it because they dislike smart kids and enjoy making them miserable?

            Is it because they figure society is better served having lots of drones, suitable for working jobs requiring little more than the 3 Rs, and likely unemployable with automation, rather than e.g. more people without even those 3 Rs – equally unemployable as those with only the 3Rs – and at the same time more people capable of doing something likely to actually be wanted? (I.e. they are set on producing what would have been “good workers” in the 1950s, and are now lucky if they can find a job at all.)

            Or perhaps it might be because those making the policy want more sheeple, because those who manage to learn critical thinking might require those they elect to provide more than feel good soundbites – and likewise might not be good customers for customers that invest more in advertising than in producing decent products?

            Crankiness aside, and my own personal history aside, it’s not clear to me that this is a good policy in any utilitarian or quasi utilitarian sense.

            Bonus question – I’m pretty sure that most motivated children with modestly high IQs (130) could pass the GED tests long before they reach the age at which they are officially permitted to leave school. If the goal is simply to bring everyone to a minimum – with the GED clearly above the minimum actually acheived ;-( – why is it *never* an option for a 12 or 14 year old to prove they’ve learned that minimum, and so get excused from farther mandatory education?

          • The Nybbler says:

            why is it *never* an option for a 12 or 14 year old to prove they’ve learned that minimum, and so get excused from farther mandatory education?

            Two reasons

            1) What do you do with a high school educated 12 year old? Unless you send them to college (which sometimes happens), this person is useless and needs to be kept out of trouble during the day.

            2) The schools get money for every student. They’re loathe to let any go.

          • faoiseam says:

            Seems clear enough — the advanced students are smart people, they’ll learn enough.

            People do not seen to have this attitude towards athletes. I would love to see a school whose coach told the quarterback that he was good enough to get by, and that for the rest of the year the team was going to focus on the worst individuals, as it is important that everyone meet the minimum standard of football that is necessary to live in American society. Strangely, tracking is completely normal in sports, where cut teams are acceptable. No-one gets cut from math. Indeed, no-one even worries about racial demographics in sports. This can’t be because sports is more important for society, as it even applies to weird sports that we would be better off without like lacrosse.

          • marshwiggle says:

            Very good point about the sports thing. It’s almost as if sports success is high status, and being good at academics in the school system is low status.

          • Viliam says:

            There are two good points ER made, but communicated them in a horrible way:

            1) It is possible that the experience of smart kids that suffered in school does not represent a typical smart kid’s experience. Perhaps people who suffered are just more likely to share their stories, and more likely to even read discussions about smart kids vs educational system.

            However, ER takes this for granted, not just a hypothesis to be examined, and provides insults instead of arguments. (If your experience is different than ER’s, you are either less smart than you believe to be, or too weird to be taken seriously. Checkmate!) This was probably intended to project an aura of “my opinion is so obvious I don’t even need to defend it”, but to me it feels like “I don’t actually know how to defend my opinion, so I will make it unpleasant to join the debate”.

            2) Acceleration is a suboptimal solution. Yes, it allows you to increase information received per unit of time, but it increases the width of information rather than depth. Instead of spending one year learning “A”, you can learn “A, B, C” in one year. But perhaps it would be better to learn “A, A1, A2” instead, where A1 and A2 could be some edge cases or connections with other topics, which the average student doesn’t have time to explore during one year, but a gifted student could.

            For example, after learning how to calculate an area of a square and a volume of a cube, smart students could spend some time thinking about how it could generalize for higher dimensions, instead of immediately moving to the next lesson aimed at average students, which would be e.g. an area of a triangle.

            Maybe not the best example, but the general idea is that authors of the curriculum had to omit many interesting sidenotes to fit it into the average student’s yearly learning capacity. If you read the same stuff, only faster, those sidenotes are still not there; you read the same mediocre text, only faster… and then you congratulate yourself on your ability to read mediocre texts quickly, which is not the same as the ability to read advanced texts. Later, you will find the advanced texts at university, and it will be a completely new experience.

          • Viliam says:

            @Nybbler

            What do you do with a high school educated 12 year old? Unless you send them to college (which sometimes happens), this person is useless and needs to be kept out of trouble during the day.

            If I imagine myself being 12 years old again, knowing all high school stuff and having a certificate for it, but not being ready for college…

            There is a lot of useful stuff I could do. Most obviously, I could learn programming — that’s not just what someone teaches you at school, but also a lot of practice. Perhaps I could even make something useful and start selling it, thus gaining independent income before college. I could try to write a book, or some other project that would benefit from having a lot of uninterrupted time.

            Or I could get knowledge about a wide range of topics that would not be provided by one college. Or learn non-academic things, such as how to cook, how to fix things, how to handle finances.

          • Michael Watts says:

            Instead of spending one year learning “A”, you can learn “A, B, C” in one year. But perhaps it would be better to learn “A, A1, A2” instead, where A1 and A2 could be some edge cases or connections with other topics, which the average student doesn’t have time to explore during one year, but a gifted student could.

            For example, after learning how to calculate an area of a square and a volume of a cube, smart students could spend some time thinking about how it could generalize for higher dimensions, instead of immediately moving to the next lesson aimed at average students, which would be e.g. an area of a triangle.

            Maybe not the best example

            That is… definitely not the best example. In the specific case, the n-dimensional volume of an n-dimensional cube generalizes exactly the way you’d expect, and takes less than one minute to understand fully.

            You could think about the n-dimensional volume of shapes that aren’t so obvious. And people do! That very topic is covered in more advanced math classes that already exist. The distinction between A,B,C and A1,A2,A3 is not well defined as you’re attempting to describe it here. More math is deeper math — or more accurately, in order to progress to the deeper math, you need to learn the more math. An n-dimensional cube is a toy problem that anyone can learn immediately. An n-dimensional sphere is a problem that takes multivariable calculus.

            The problem I see with acceleration is that it ignores the idea that people can learn at different rates. You have a kid who is, by some sort of magic, ahead of his class. You can advance him to the point where he’s no longer ahead of his class. But the term “acceleration” is misapplied, because the new class moves at the same speed, targeted to average students, as the old one. That kid doesn’t need to be promoted into a full-year geometry course. He needs to be promoted into a 3-month geometry course, and then another 3-month course, and then a third. “Acceleration” was an improvement in that you went from wasting 100% of his time in math class to only wasting 67%, but that’s not something to be proud of.

          • John Schilling says:

            @Mark:

            To me, SSC is about folks having unpopular opinions that might get one banned elsewhere. Instead they start good discussions, as long as the unpopular opinion is written in a coherent and rational way, and not just as a rant

            “You are irrelevant and you should stop complaining” and “your suffering should not be alleviated because you’ll get over it”, certainly qualify as unpopular opinions, but I don’t think they are productive even if they are written in a coherent or rational way. At best, they are useful for driving away people you don’t want participating in the discussion. And if there’s a good discussion going on, it is in spite of comments like that rather than because of them.

            About the only thing ER has to say that isn’t some variation on one of those two themes, is that bright kids suffer greatly from not being adequately challenged in primary education. That’s absolutely true. But it’s not an unpopular opinion, nor is it novel – indeed Woodgrains and Pershan raise the issue in their original post. And address it with potentially effective solutions like tracking with curriculum adjustment, and grade advancement. Leading to an informative discussion of e.g. the social costs and benefits of grade advancement.

            ER, just tells us that because these people aren’t challenged they become miserable insufferable complainers, and that because he’s tired of their insufferable complaining he doesn’t care about their misery. Maybe I’m missing something, but I don’t see one positive, concrete suggestion on how the education might serve advanced students anywhere in his rant. Which, yes, is a rant.

            And if it appeals to you as an “unpopular opinion”, you’re welcome to it. Because he seems to be willing to offer up plenty more, whereas I am sufficiently annoyed by the whole thing to see no value in continuing and regret offering even this response.

            But I’ll hit “post comment” anyway.

          • Mark Atwood says:

            What do you do with a high school educated 12 year old? Unless you send them to college (which sometimes happens), this person is useless and needs to be kept out of trouble during the day.

            Apprentice them to a technical trade. Sign them up for music lessons. Enroll them in an atelier. Put them in a dojo. Put them in Boy Scouts and challenge them to get every merit badge.

            Why should this be hard?

          • educationrealist says:

            So the John Schilling character doesn’t understand my comments in the slightest, and in many cases has it backwards. I am untroubled.

            The Viliam character correctly captures two of my positions and restates them for anyone who can’t understand me, or who is upset that I don’t value their pain.

            Villiam sez I assert without evidence that smart kids who were miserable in high school are a minority. But the whole premise of the article is that smart kids ALL are miserable in high school, on the basis of a reddit and blog query. They are the ones who are required to prove their assertion. I’m countering their assertion that has no proof.

            And support (not proof) for my counter: 1) time and again, results show that our society is broadly sorting by IQ into relevant occupations. 2) our much less aggressive tracking is consistently producing a better economy than Europe, where tracking smart people is the norm 3) anecdotally, I–with several hundred to a thousand samples more than the individuals here who have a data point of 1 or 2–observe that the people considered to be the broad swath of mainstream in this conversation are, in fact, fringe.

            Viliam is incorrect that I am using my own experience to dismiss others, as I’ve said at least twice, I had a tough high school experience.

            Villiam accurately captures what I mean by go deeper, thanks. Michael Watts, so completely typically for SSC, obsesses about whatever ridiculous technical objection he had, then assumes that means he can dismiss “go deeper” and returns to the same really bad reason for assertions.

            The suggestions for what 12 year olds could do are both ridiculous and typical.

          • marshwiggle says:

            So what if the majority of smart kids don’t really mind it if noone challenges them in school? That fringe that does mind it is kind of important. So what if there aren’t a huge number of them. The smartest and the most motivated to learn might just have an outsized impact on society. Plus I imagine that lots of people in this little corner of the internet find it easier to have empathy with that fringe than you do. You yourself are providing a repeated example of the lack of empathy, caring, and respect that teachers all too often have with a smart kid who wants to learn.

            I’m seriously not against a policy of depth rather than acceleration. But I can’t see it working unless the teachers implementing it are capable of both depth themselves and of empathy with a smart kid.

          • TracingWoodgrains says:

            We worked to be pretty explicit in the article that not all smart kids are miserable, and not all of them struggle or underachieve in the way we were concerned about. We were just focused specifically on the group of smart, underachieving, frustrated kids.

          • arlie says:

            @marshwiggle

            It’s almost as if sports success is high status …

            Almost? My high school praised athletes in assemblies quite regularly. I never saw any public praise for anything else. The message about relative worth was loud and clear, and if I hadn’t had adults in my environments pointing out that in the long run, the sports stars would be asking my peers “do you want fries with that?” I would probably have made some really bad decisions.

          • Viliam says:

            @educationrealist
            This humble character would like to point towards the concept of reversed stupidity. Specifically, the role of IQ at gathering knowledge can be simultaneously greater than 0% and smaller than 100%.

        • marshwiggle says:

          “You aren’t a problem that needs fixing”

          If bright kids are well-served by public schools, it’s just as fairly obvious that poor inner city people are well served by the system of policing in the United States. But saying “You aren’t a problem that needs fixing” to the small percentage of poor people that get unjustly shot by the police, or their families, is rude at best. Saying that poor inner city people could also get various forms of affirmative action to enter the police force is also rude at best when said to people who have been unjustly shot by the police.

    • Robert Jones says:

      First, sending a ton of math one way isn’t something that only bright kids can learn, it’s also something that swots can learn. And there’s a big difference between “bright” and “a swot”, but right now, our math system in America is not only actively disadvantaging bright kids by going so fast, but rewarding the dutiful swot.

      I don’t see the problem. If acceleration benefits both bright children and swots, it seems like a win for everybody.

      Also, I think you skipped “second”.

    • marshwiggle says:

      I may not be being sufficiently charitable here to your post, but it seems like it has a serious undercurrent of not caring if we educate either the gifted or the really gifted. Your plan really does seem like ‘hope that the really gifted kids teach themselves, because we sure don’t want to waste resources on both the gifted kids and the ones parents want to think are gifted’.

      I’m not sure how accelerating a kid so he hits the wall faster is worse than letting him find everything easy for even more years.

      I agree that bright kids avoiding challenges is a serious problem. But I don’t think just letting them sit there without acceleration exactly helps that. Anything that really challenges them without acceleration is going to take a lot of specially tailored effort devoted to one gifted kid. Is that really going to happen in most classrooms? I think not. It’s even worse with a really bright kid – especially in math and science, anything that would challenge such a kid, most teachers aren’t capable of themselves. I agree that really challenging a kid with deeper instruction would be way better than acceleration. But if we can’t get that, acceleration at least is something schools can do without taking resources away from non-gifted kids.

      • Robert Jones says:

        Resources do exist for challenging pupils within the scope of the school curriculum: I think of the work of the UK Maths Trust, although this is limited and composing such questions is difficult. Doing interesting and challenging questions is enjoyable for the pupils (as I suspect many of us can vouch), but in the end what is being achieved? By definition, the pupil is not learning anything new. Is there any evidence that, say, participation in the IMO makes people better mathematicians?

        • Lambert says:

          Second the UKMT for maths questions that are hard but rewarding in some hard to define way.
          Would recommend showing past papers to bright kids who need a challenge.
          Or just doing them yourself if you feel like thinking really deeply about something for a while.

          The maths itself is never that advanced, but you have to it in nonobvious ways.

          https://www.ukmt.org.uk/individual-competitions/

          • marshwiggle says:

            That stuff looks great, and in the US at least there are also many fine math competitions students with motivated parents can enroll them in. There’s lots of math questions online. Could and would teachers in a random classroom use those problems in a constructive way? At least for the US, even very good teachers in a very good school district… don’t. It would take time, and it would help the education of a gifted student, and either one is usually enough to stop a proposal like that. But at least parents can and should give resources like that to their gifted kids.

      • educationrealist says:

        “it seems like it has a serious undercurrent of not caring if we educate either the gifted or the really gifted. ”

        We do educate them. And for the most part, we educate them quite well. I would like us to do better.

        But “better” is not achieved by thinking that all the people who were bored in school and are now complaining about it on the internet represent the source of the problem, much less ideas for a solution.

        • marshwiggle says:

          Is this the definition of ‘do educate them’ which might not be even as good as completely unschooling them? I’ve seen far too many genuinely talented kids come through the public school system without the least mastery even of the most basic algebra, genuinely thinking that no more is possible, and protesting (truly but counterproductively) that they are completely capable of everything that the public school system ever asked them to do. Does that count as educated, and well?

          But, I believe that if it was costless, you would want us to educate such people better. I wish that went without saying, but it doesn’t.

          I agree that not all complaints come with solutions, much less viable ones. But in the face of an educational establishment that really seems not to care about the emotional or academic wellbeing of its gifted students, perhaps that complaint is necessary. I’m not including you in that – I think you would actually prefer that gifted students receive more education rather than the same amount or less as the average student. The sad reality though is that both at the local and national levels, not much gets done if there isn’t complaint.

          • educationrealist says:

            ” I’ve seen far too many genuinely talented kids come through the public school system without the least mastery even of the most basic algebra, genuinely thinking that no more is possible, and protesting (truly but counterproductively) that they are completely capable of everything that the public school system ever asked them to do.”

            I’d want to see data, and the data would look something like this: 19 year olds with IQs over 120 and SAT math scores of 400.

          • marshwiggle says:

            Ah, my fault for being unclear. I’m defining mastery here as the ability to use the math for practical purposes and to extend its use beyond what one has been taught. Because of teaching to the test and all, SAT math questions are enough like what people have seen in school. I’m guessing the kids I’m referring to would be getting 700+ on their math SATs. But can they use the math to do something they were not showed exactly how to do? It’s like one out of four IQ 120+ students who were not accelerated who can do that. I think that is a serious problem.

            If all you meant by educated was ‘can get 400+ on sections of the SAT’, then I can understand what you mean. Myself, I think our society needs people better educated than that, and I think the people with the best chance of doing that are the ones with 120+ IQ.

          • educationrealist says:

            Well, part of that is the problem of acceleration and swotting that I described before.

            The other part of the problem is that if you don’t have tests that capture the student’s inability, you can’t really claim they weren’t taught algebra.

          • marshwiggle says:

            The kids I’m thinking of weren’t accelerated. They have a variety of swotting levels. I don’t use standardized tests, but when I teach physics classes I give out pretests not of physics knowledge but of math knowledge. I’ve designed them precisely to capture the math knowledge most necessary to have any chance at doing physics problems. So yes, I do have a test, and a ridiculous number of bright kids fail it. These are kids who got mostly As, perhaps some Bs, in public school math. The ones who don’t fail it had some sort of math education outside of school, some sort of acceleration, or are in the top 10% of conscientiousness.

            All it takes to get kids to fail my pretests is to put the same math they know how to do in a school context into a different context. That is a bad sign.

            So no, I don’t have an n=10000 study. I don’t have a standardized test. I don’t have a p-val.

            But I am basing my statements on data. That is data I collected not to prove a point but because I suspected I had a problem and I wanted to know more exactly the shape of it so I could teach physics more effectively.

    • TracingWoodgrains says:

      Thanks for the comment here, and once again for your help with the drafts. Your perspective is, as always, acerbic and thoughtful. Let me know if Michael hasn’t gotten you the link to the draft–I can send it over to you, but I’m not sure the best place to do so.

      If you consider the exceptional math intellect, we know how to educate them, and they’ll figure out the challenges for themselves. They’ll find the question.

      I just don’t agree with this. People don’t exist in vacuums. Not even exceptional math intellects, though we sometimes like to think they do. One reason I find Gross’s work so compelling is its demonstration that even among the most highly selected groups (4+ SD), the level of challenge in their education relates closely to their later achievement and academic interest. People just don’t usually educate themselves, and even the most brilliant need well-suited environments.

      In fact, the biggest problem we have right now with bright kids is that they fold at the very mention of a challenge, because they’ve spent 8 years in elementary school bored because they “got” everything. But in fact, these kids spend a lot of time avoiding anything they don’t “get”, because they get uncomfortable. That’s why the good teacher, and the good parent, will find something that kid doesn’t know how to do and can’t easily master, and make him or her miserable.

      We’re in 100% agreement here, and I would happily bind this statement, publish it, and hand it to the parent of every bright kid around. It’s a huge problem, and honestly, it’s one of the core problems that led me to raise this topic. I’m much less certain about your dismissal of the direct result of this problem, as stated: a bunch of obnoxious people on the internet who were bored in school. People don’t develop good or bad social skills in vacuums, either, and a roomful of people whose only similarity to you is being the same size as you, struggling to learn something you’re desperately bored with, is a brilliantly designed system to create a bunch of frustrated, obnoxious people who can’t stop talking about how tedious everything was.

      Acceleration within the standard system is a band-aid at best. But as far as we can tell, it’s better than nothing. Significantly so. I’m not sure where you’re getting the point that most of the people complaining online wouldn’t have achieved more if they were accelerated. 2-3 SDs isn’t that much, and I’d expect the benefits to extend to most people in that group. Basically: they might be obnoxious, but they’re not wrong.

      You know as well as I do that significantly expanding resources directed towards top students just isn’t happening within the current climate, and a genuinely good learning environment for e.g. math would more-or-less require that. Art of Problem Solving exists, but good luck putting it into many schools. Teachers care, and parents care, and there is almost nobody in the system who doesn’t care or want to help, but caring only goes so far in a system carefully avoiding providing some kids a meaningful challenge lest they get too far ahead. So I push for acceleration as the minimum-cost, minimum-effort solution in settings where there isn’t a better option available. It’s not about treating kids as brilliant. It’s about finding them a setting where they won’t feel brilliant, and won’t develop the host of bad habits that come along with being “the smart kid.”

      • Mark V Anderson says:

        Excellent reply to ER. Doesn’t solve anything, because it is just about different opinions at this point, but it lays out the issues well.

      • marshwiggle says:

        This is a great reply, and not just because I agree with it. It is great because it looks like it could help us get some civility back in the discussion. I say that having not been entirely peaceful myself.

      • educationrealist says:

        “thoughtful but acerbic”

        Hey, I like that. Up there with “cynical but believable” (courtesy spotted toad) as a two-adjective aphoristic description.

        When I talk about people learning on their own, I’m referring to the exceptionally to profoundly gifted: Feynmann, Einstein, Tao and so on. People who teach themselves advanced math in their teens because they are driven to do so.

        If they aren’t driven, I’m unconvinced that the 4SD intellect, which is exceptionally bright, will be damaged by waiting the normal amount of time to become interested in math. Moreover, I’m not convinced that a kid who isn’t driven to pick up calculus on his own will pick up calculus in a classroom *in a way that makes him or her more productive for doing so*. The Chinese are teaching their kids calculus at that age, but it’s not giving them a big leg up.

        “I’m much less certain about your dismissal of the direct result of this problem, as stated: a bunch of obnoxious people on the internet who were bored in school. ”

        I absolutely reject your premise. Kids who were so miserable in school that they routinely complain about their agony on the internet are NOT a direct result of the problem. Many exceptionally bright kids might have been bored in school, or found it easy, or weren’t sufficiently challenged without claiming the Dickensian levels of misery that I’m less than sympathetic to. I’m not saying “oh, boo hoo, all the poor little bright kids were so sad in school” but “Thousands and thousands of exceptionally bright kids aren’t learning as much as they can every minute and they’re neither bored nor unhappy and therefore we need to consider the complainers fringe, not representative.”

        It is for *this* reason that I say that the bored kids are educationally irrelevant. How we handle the smart, smarter, smartest, and brilliant children of our country is a worthwhile policy question. Making kids who think they’re too smart for their classroom happy just because they think they deserve better, not so much. For starters, I don’t have to accept as giveni that everyone who says they were unhappy and bored in school was much past 1SD over average. I don’t have to accept that everyone who says they have a PhD from an Ivy is, in fact, a PhD from an Ivy.

        “Some smart kids were unhappy in school” isn’t a policy prolbem unless you can quantify it in ways past commenting on a website and prove that it’s endemic. And here research isn’t really helpful because, as I think you both point out, we don’t quantify IQ consistently when evaluating bright kids. (the word “gifted” is absurd, really.)

        So could we do better? Yes. Is our failure to completely capture these kids’ intelligence at an early age quantifiable? Not yet, because a bunch of people complaining about how bored they are isn’t a valid metric.

        Begin with the fact that not everyone in this situation hates school, fails to live up to their potential. Remember, these students will qualify for huge chunks of free or subsidized tax dollars when they get to post-secondary education. All data suggests that IQ is hugely influential in academic outcomes, so its’ a safe bet we aren’t losing huge numbers of advanced intellect to the field of ditchdigging or Starbucks barrista.

        So when arguing that they should have even more attention than they do now, the “I was miserable ignoring the teacher and reading a book but when I spat on her and her bourgeois notions of education, I had to go to the principal’s office where the bullies beat the crap out of me and stuck my head in the toilet” anecdata is not a reason to move forward.

        None of this means I don’t think there are reasons. Just not that one.

        We have huge challenges if we are to educate by intellect, two of which you identified. If we track, the lowest track will be a huge challenge and we are a diverse country that would have tracks with clear and depressing racial challenges. Second, how do we identify students for the highest track? Left unmentioned is that we’d probably have three tracks, because parents of the non-brilliant would actively and reasonably seek to get away from the least intelligent. But the societal unhappiness with any properly enforced track is almost insurmountable, so whatever sorting we do will have to be in some way that the tracks aren’t identified. We’re so far from this it’s almost funny. We’re going *backwards*. We’re killing college as a reasonable form of sorting.

        . I think the problem lies again in that you and others here are focused on a relatively small group of kids whose parents said oh, heavens, my kid’s really smart so let’s do something, and then were annoying enough (or the kid unusual enough) that the school sighed and made some adjustments.

        This is such a tiny fraction of the overall chunk of kids with 125+ IQs that it’s just laughable to cite research on how different approaches work. Look, I don’t teach advanced kids, I’m in a Title I school with no special resources and yet every year I run into exceptional intellects (only occasionally in my own classrooms). These are kids who were bored in elementary school, spend time going through the motions in high school, but are on their way to exceptional colleges to be world dominators. That’s the norm for really smart kids.

        Those are the kids I want to do more for. And acceleration is simply useless as a broad policy objective. It’s fine for annoying parents and snowflakes, and also for the legitimate corner cases. But in my view the weakest part of your paper was the case for acceleration, because it views the treatment as the norm, rather than fringe.

        So the last really big question is how can we identify and educate smart kids in way that isn’t merely dumping more info on them faster, which is educationally pretty useless?

        I am certain this comment is repetitive and less than coherent. I will try and get a post out about it, for my own peace of mind.

        • TracingWoodgrains says:

          When I talk about people needing structures, I’m also talking about the exceptionally to profoundly gifted, specifically and directly Tao. Have you looked into the longitudinal study Tao was involved in? He received an enormous amount of support from an early age, and the results of the study suggest that, without a supportive, flexible academic environment, he would still be mind-bendingly brilliant but he probably would not be a Fields medal-winning mathematician. Impossible to be certain about a statement like that, of course, but I’m not sure how you can take the example of someone who went through an accelerated, structured math curriculum while being actively studied and compared against similar kids who weren’t as an example in support of your thesis.

          I’m honestly still not sure where you’re getting the point of uselessness of acceleration as a broader policy objective. From all the research I’ve seen on the topic, it creates noticeably higher outcomes both short- and long-term, despite being essentially a hacked together solution most times.

          Anyway, you’ll be unsurprised to hear my background here: I was one of those miserable, frustrated kids in school. Not on Tao’s level by any means, but an outlier. Growing up, I constantly looked for more challenging settings, including taking advanced classes, skipping a grade, taking a year of online school, and attending an early college high school, and the whole time it felt like I was pushing against a very powerful machine designed for people completely unlike me.

          Five years ago, I was a scholarship student in college, writing about my problems with the school system and trying to figure out a proper work ethic for the first time, then predicting that when I was really tested, I would fold like a paper doll. And, honestly, I folded. I couldn’t figure real life out, and dropped out of college to find a setting where I could stay sane. I’ll keep it to that for privacy reasons, but I should be clear in saying that this is, and has always been, personal for me.

          Clearly it’s not just intelligence playing into situations like this, like I was convinced when I was younger. The best model I have right now is a two-axis one between intelligence and a rough measure of conscientiousness, where intelligent, conscientious students excel, other conscientious people are well served by school and do reasonably well, intelligent people with low conscientiousness coast and then hit walls, and those given neither are placed in a genuinely impossible setting. The standard system breaks down hard in both of the latter categories, but the smart kids in that group can at least usually find a setting they do alright in later on, so even though they’re bored and frustrated through their childhood it pretty much works out in the end.

          Here’s where I agree: this isn’t the most important policy question in the world. More data is both absolutely needed for better conclusions, and incredibly frustrating to find (if you have any leads here, please let me know–Gross’s research is the only solid work I’ve found that really even investigates this as a group). Culture presents huge barriers for anything resembling a real solution to any of this.

          But when I’m talking about acceleration, I’m looking mainly at parents with kids in those impossible situations, frustrated and bored and being marched through a long process designed for anyone but them. And I’m saying–hey, look, in a lot of similar situations, this is one thing that helps. And I’m looking at educators and saying–hey, for a certain set of kids in your room, just being a good teacher and trying to meet their needs isn’t going to help much. They’ll fit better elsewhere. I’m asking people, mainly, to acknowledge that the group of frustrated smart kids exists, even if it’s hard to figure out specific numbers or anything like that, and that there are tools that help them more than asking them to sit in a regular classroom reading a book and going through the motions when the teacher snaps at them.

          It’s a minor problem compared to some others, but it’s my minor problem, and one that I very rarely see raised in a cogent way by educators. I completely agree with your final question, and would love to see whatever posts you end up writing on the topic. I’ve consistently enjoyed reading your blog as one of the few places where I can find honest, accurate assessments of education as it stands, and it would be great to see a more complete form of your thoughts on all this.

          • educationrealist says:

            I’m honestly still not sure where you’re getting the point of uselessness of acceleration as a broader policy objective.

            Imagine that you take a second grader and put her in third grade–and she fails. How do you grade that? Do you give her credit for trying? Do you not grade her at all?

            Now bump it to ninth grade, where a kid tests out of algebra I and geometry and goes straight into algebra 2. He fails. How’s that work? He’s lost a whole year in school, has no math credits. Do you let him take A2 again, or send him back to easier classes so he can build up credits?

            Oh,and don’t forget the diversity machine. If you let anyone who wants to test out of a class and accelerate, someone eventually is going to notice the gender and racial imbalance and either put a stop to it, or demand that the standards be lowered.

            Or what about the fact that right now, acceleration is for corner cases so it’s a huge burden to get accomplished, but once it’s not a rare thing, parents will start demanding it, and start pressuring to let their little snowflake in, and so on.

            So acceleration will inevitably, of a certainty, introduce the same equity lawsuits if it’s ever turned into formal policy.

            And for what? For a broad-based instruction that simply lets a smart kid glide through the curriculum more quickly? Nothing is achieved.

            Look, I didn’t learn how to learn until I’d finished college. One of the reasons I decided to major in English after trying out three different majors is because I simply couldn’t learn anything that didn’t come instantly. I wrote about learning how to learn here. I, too, suffered from bullying in middle school (it’s the “real life trauma” I reference in this piece. My parents made a choice that resulted in my *not* going to Philips Exeter, an insanely rare opportunity for a working class kid, and one it took me years to stop fully resenting. My reaction to the middle school trauma and the loss of a prestigious boarding school was to stop caring about academic performance in high school–and without trying, I still graduated in the top third of my class and was one of only four National Merit semifinalists our school produced that year, as well as getting a 5 in AP English Lang & Lit (much rarer 40 years ago), and an 800 on the English lit subject test (still very rare). On the other hand, I got Ds in math and science, although I did pass the AP Calc exam.

            I never felt the strong resentment and outrage that I see reflected here, probably because my life was extremely dysfunctional in other ways and I was simply happy to survive it. But many MANY years ago, I began to realize that in order for my life outcome to be dramatically different, worthy of my not-inconsiderable intellect, one of three things would have had to be true.

            1. I would have had to come from a much different, higher social class, with parents in prestigious jobs and lots of contacts.

            2. I would have had to be extremely ambitious and driven.

            3. I would have to be smart in a manner more obviously congruent with the needs of industry. For me, that would mean I would have had to be better in advanced math, or (and this is probably the biggest what-if) I would have had to go to lawschool.

            None of these are true. I regret that no one ever mentioned the possibility of lawschool to me, but I doubt it would have mattered.

            I also know, however, that in various ways my intellect has served me well. I have been making my income off of it my entire life, and people have been paying me for my opinions or my intelligence for all my working life. And I never had to work too hard. Also, people with my history who don’t qualify on one of those three vectors often end up much, much worse, so I probably am more conscious of that.

            So when I hear of people who feel misused by high school or middle school, I figure that most of them are, like me, people who are missing one of those factors–or have some that I don’t have (aspergers, over-confidence, whatever)–but focus their ire on the educational issue.

            But once you spend a lot of time in school, you will see tons of utterly brilliant kids who know 90% of everything from the first day and thrive in school, despite being much smarter than everyone else. You’ll see others who are extremely bright and unambitious but not traumatized or outraged by their boredom, as was mostly the case for me. And then you’ll be skeptical of the received wisdom that school is hurting bright kids.

          • Education Hero says:

            I regret that no one ever mentioned the possibility of lawschool to me, but I doubt it would have mattered.

            Given your critical reasoning ability, writing skills, and demonstrated personality traits, you’d seem a better fit for business school, especially with an eye towards audit or management consulting.

          • TracingWoodgrains says:

            Thanks for the reminder of your story. The academic side is very similar to my own, but I was lucky enough to be in a comfortable, sheltered, nearly problem-free setting otherwise. But problems swell to fit the available space, and so the education issue loomed throughout my childhood as a hugely influential, persistent frustration.

            Is it possible that you’re focused too much on long-term outcomes here? For me, the more serious problem is that twelve-year span where some kids end up frustrated, bored, and miserable in preventable ways. “Statistically speaking, you’ll turn out ok in the end” is effective from a policy standpoint but pretty wretched for the kid while it’s going on.

            For what it’s worth, I agree that acceleration is a tough sell as a focus for a national policy change and has steep barriers. I think it’s worth mentioning because a) some kids are desperately looking for more, b) given its current rarity teachers and parents are almost certainly underapplying it in situations where it could be useful, and c) it can happen right now in many places, with no policy change beyond a recognition that it’s a fairly reliable stopgap to help kids out. So: telling people that it’s good, and that it is one way to help a set of edge cases that receives less attention than it merits, is immediately useful.

            Oh, yeah, and it focuses people’s attention on the simplest principle in learning, which has somehow become mangled beyond recognition at all levels: people should be taught what they do not already know. I like pointing to examples of radical acceleration to examine the outer barriers of learning speed and to see what happens when exceptionally bright kids are taught at their natural pace. But the acceleration isn’t the point. Pacing to the child is the point.

            This is why I’m intent on turning my own focus towards educational technology. If you create a program that works, like AoPS, you sidestep a ton of institutional barriers and give people a reasonable, easy way out that doesn’t take focus or money away from other students. It becomes an implementation problem more than a political one.

            Anyway, people who fulfill your criteria flourish in the system we have, and you’re right that more brilliant kids than my grade school self would have ever believed do just fine. But me? I still want to learn, and I want to do so in structured, reliable ways that don’t rely on the vagaries of my own motivation. And for predictable, understandable ways, what was available wasn’t what I needed. I see the same desire and the same in other resentful, outraged students past and present, which is why I take their side more often than not.

            What we have doesn’t work for some kids (and no, I’m not talking only about bright ones here, and yes, I realize the irony in writing this as a reply to you), and it’s not the fault of educators or the parents or the kids as much as an inevitability in a complex system that is broken in predictable ways for entirely human reasons. And if there’s a straightforward solution that can help a struggling kid, even if that kid is smart and should, statistically speaking, turn out ok in the end, I see much more value in providing that solution than telling them their problem doesn’t matter.

        • Robert Jones says:

          I don’t have to accept that everyone who says they have a PhD from an Ivy is, in fact, a PhD from an Ivy.

          That level of distrust seems both excessive and counterproductive. It’s reasonable to question whether someone’s subjective assessment of their own intelligence is accurate, but it’s not reasonable to call someone a liar.

          • educationrealist says:

            Why on earth would you unquestioningly accept everything people tell you in a low trust setting if you’re trying to do research? I’m not calling you or anyone a liar. I’m simply saying that it’s absurd to accept Internet comments at face value when assessing the need to educate advanced students.

          • Robert Jones says:

            If somebody says, “I’ve got a PhD from Cornell”, why wouldn’t you believe them? It’s not a particularly implausible claim: lots of people do have PhDs from Cornell. It’s not the sort of thing that somebody is likely to be mistaken about. To doubt your interlocutor in a matter which is clearly within their knowledge and outside yours is such a drastic level of distrust as to make the conversation pointless.

            You prefaced your own comment by saying, “I am a teacher of non-gifted kids in three subjects. I raised a kid with an IQ in the 130s/140s. My own IQ puts me in the 4+SD BGI study of a few years back.” Presumably you expect people to believe you?

            I agree that education policy should not be based on a random sample of anecdotes: I don’t think that’s controversial.

          • educationrealist says:

            Presumably you expect people to believe you?

            People are welcome not to. I’m pretty sure my blog establishes them all in various ways.

            If somebody says, “I’ve got a PhD from Cornell”, why wouldn’t you believe them?

            I’m not talking about personal interactions, when I would presumably accept assurances at face value unless something made me doubt it. And this is the last time on this particular go-round. People who demand that you believe everything you read online squick me out.

          • arlie says:

            Roflmao.

            IIRC, there was a comparatively recent incident involving a highly opinionated person getting fired from Google. His LinkedIn profile claimed a doctorate he’d never earned – he’d dropped out of his doctoral program when hired by google. After several suspicious people fact checked the profile, and the falsehood got into the public news stream, it was changed.

            Existence proof – at least one person with fervent opinions and an insistence on talking about them with strangers lied about a doctorate. IIRC, the institution was either Ivy league, or the equivalent among tech specialized universities.

            Hypothesis – majorly opinionated posters may be more likely than others to make false claims about credentials, and the more trollishly they behave, the more likely this is. Completely insufficient evidence to support this, but it would account for Robert’s perceptions of rarity, and the comment of the person they responded to.

        • baconbits9 says:

          I don’t have to accept as giveni that everyone who says they were unhappy and bored in school was much past 1SD over average

          It doesn’t matter, its still a failure of the educational system either way.

          • Robert Jones says:

            Yes, I just don’t understand the point being made here. It’s plausible that people overestimate both their own intelligence and their counterfactual success, but it’s implausible that people are mistaken about having been miserable in school, particularly if they feel the need to rant about it to strangers on the internet years afterwards. If it’s a common complaint (which I haven’t observed myself, but that’s what educationrealist says), then it seems reasonable to conclude there’s some problem.

          • educationrealist says:

            No, it’s not.

          • baconbits9 says:

            No, it’s not.

            Either you are a genetic determinist where you think people’s outcomes are so heavily biologically based that as long as they aren’t being actively abused the system doesn’t matter at all or you think that 10 year olds showing ought to know what type of schooling is best for them personally while also being capable of demanding it. Which is it?

        • educationrealist says:

          I have trouble with these threaded conversations. But Tracing Woodgrains, you are correct that you acknowledged that in the piece. I knew that–in fact, I think I mentioned it as an edit.Sorry.

      • educationrealist says:

        In reading back through all the comments here and downthread, I want to make it clear that when I spoke of the obnoxious people on the Internet who were bored at school and teachers were mean to them, I wasn’t focusing solely, or even primarily, on SSC readers. I reference this meme all the time because it’s so prevalent everywhere in the comments threads of blogs throughout the universe. I linked in a piece that mentions the same sentiment–not that you had to read it, just to demonstrate I’ve often expressed this sentiment:

        So next time someone tells you a tale of woe about how his teachers were jealous of his tremendous intellect and treated him with petty malice, allow for the possibility that maybe he’s just obnoxious. Sure, there are mean, petty teachers. Just not all that many.

        Now, I have to also admit that one of my own private subtitles for Slate Star Codex commenters is People Who Thought They Were Too Smart for High School And Are Still Pissed Off About It, so certainly SSC was, er, on the list of sites that I might be thinking of first.

        In other words, you’re welcome to take my comment about obnoxious internet folks whining about high school personally, but I wasn’t ranting specifically about SSC an certainly had no one specific in mind.

        (Also, while more than a few people have complained about me, I’d point out that right below this thread is a guy saying this paper was “quite bad” following it up with an incredibly insulting comment about how, you know, easy it is to edit.)

        • A Mom says:

          Longtime SSC reader, first-time commenter, dealing with a 9-year-old who reminds me how much she hates school on a daily basis (and was much happier during the summer when we were working on geometry theorems and proofs). I’m commenting specifically to thank you for your comments, as well as TW and Michael Pershan for writing the original piece.

          Now, the dilemma I’ve been wrestling with for a while now is different from the question either the original work or your comment is addressing. All the work here is focused on educational policy, whereas my question is “What should I do for my unhappy kid?” Her unhappiness is not necessarily a problem for her school / district / community to solve. (She was at a Title I school, where my younger daughter still is; older daughter is now at a “magnet” school within the same district supposed to be aimed at “high achievers” — tracking at the district level, I guess — although frankly I haven’t seen a lot of difference between the two schools, save more busywork.)

          (W/r/t your comment about teachers: my experience at our Title I school has been that teachers have been generally quite positive towards my girl but lacking in tools to address her specific needs. Last year the school tried to get both my girls subject-accelerated, and both times got thwarted by district policies.)

          There’s a separate discussion about how much individual parents should hold themselves responsible for education policy and public-school outcomes, but I still feel pretty comfortable saying that, whatever responsibility the school / district / education infrastructure has towards my individual kids, I have a greater one.

          Where your comments in particular are helpful is helping me work towards a solution. Right now I’m leaning towards homeschooling, but have two big arguments against it. The first is intrinsic motivation; I’m not very good at it (in part because things came so easily to me in school, especially elementary school) and my older daughter isn’t either (with the exceptions of drawing, Scratch, and all things Harry Potter-related). If I pull her I’ve got to both work out a motivation system that doesn’t replace intrinsic motivation for her, and model it myself, which, see previous sentence. The other is that you pair a kid who’s smart, passionate, emotional, and very dramatic, with a mom who’s also emotional and a people-pleaser, and you have a setup ripe for manipulation. I haven’t yet figured out how to solve that yet, other than to assume that I’m going to have to grow a spine and there will be spine-growing-related pain for both of us. So a more acerbic (to borrow the term) view of smart kids saying that they’re miserable is helpful.

          • educationrealist says:

            Thanks so much for your response. I agree that having a bored and angry kid at school is a parental problem.

            “W/r/t your comment about teachers: my experience at our Title I school has been that teachers have been generally quite positive towards my girl but lacking in tools to address her specific needs.”

            I’m glad. That’s consistent with what I see, at any rate. I don’t know what her specific needs are, but since she’s already at a school for advanced kids, I would start by considering the possibility–one that so many parents forget–is that you can’t make her happy. This isn’t a problem you can necessarily fix. That’s a hard one for parents.

        • The Nybbler says:

          I reference this meme all the time because it’s so prevalent everywhere in the comments threads of blogs throughout the universe.

          Perhaps this is because the meme reflects a common reality.

          • educationrealist says:

            A common reality reflected among people using blog comments would almost certainly not be reflecting the overall pool of smart people, just one subset–the subset, oddly, with the characteristics that might make them miserable in high school no matter what.

          • faoiseam says:

            the subset, oddly, with the characteristics that might make them miserable in high school no matter what.

            I have worked for many years in a university department, so I meet a large number of incoming Ph.D. students. The American ones invariably had one of two experiences in high school. They either hated it, were bored and socially excluded, and seemed scarred when they speak about it, or they went to an very pushy majority Asian school, where they studied way too much, and found undergraduate much easier than high school. International students do not have this pattern at all.

            Obviously, this is a very selected set of students, but it does suggest that the top 0.1% or 0.01% of students are not having a good time in high school. By the way, both patterns are bad, the former because of the psychic damage, which sometimes is not cured by undergraduate life, and the latter by the bad habits it encourages during undergraduate time.

            As the pattern does no appear in international students, by and large, there might be improvements that could be made to the US system.

          • educationrealist says:

            International students come from countries that track. And despite coming from countries that track, they aren’t achieving spectacularly more than America, which largely doesn’t track.

            And phD students are getting massive amounts of funding from taxpayers either directly or indirectly, so if the American school system got them to a Phd, the idea that the system failed them is pretty absurd.

    • rahien.din says:

      I really wish I had read this before commenting below! My favorite part :

      In fact, the biggest problem we have right now with bright kids is that they fold at the very mention of a challenge, because they’ve spent 8 years in elementary school bored because they “got” everything. But in fact, these kids spend a lot of time avoiding anything they don’t “get”, because they get uncomfortable. That’s why the good teacher, and the good parent, will find something that kid doesn’t know how to do and can’t easily master, and make him or her miserable.

      Amen. I hope my kids have educators who hold to the same core values.

      • Matt M says:

        It’s amazing to me that looking back, all of the teachers I’ve had that I would consider good are the ones who really challenged me and made me work. Which was rare.

        The odd thing is that I don’t get the impression that most of my peers feel this way (even the smart ones). Their favorite teachers were, and remain, the ones who allowed everyone to coast through with an easy A and spent all class talking about what movies they saw over the weekend.

        I hated those teachers but everyone else loved them.

        Meanwhile, one year we had a brand new English teacher. The class started with 30 students. After about two weeks, there were 15. They all dropped the class because it seemed too hard. I had a lot of friends in it originally and just about all of them dropped. I took two things away from that class:

        1. I learned more in that class than any other English class I ever took

        2. I remember feeling so bad for the teacher, like she must have felt like she was doing something wrong or people didn’t like her or whatever, but she was just trying to do a good job – and ultimately succeeded.

        • pontifex says:

          My understanding is that teachers are often pressured to keep pass rates high, even when the students aren’t learning. Obviously this varies from place to place, but it seems depressingly common. I can also confirm hearing about this secondhand from a relative who was a teacher.

          • Viliam says:

            My understanding is that teachers are often pressured to keep pass rates high, even when the students aren’t learning.

            Former teacher here: you are right. When one student fails, it’s their problem. When many students fail, the institution has a problem (and will punish the teacher for not preventing it). So when too many students can’t reach the goal, you keep making the goal simpler and simpler, until most of them reach it.

            The most extreme case I saw was when the exam consisted of dozen “yes or no” question, and you had answer half of them correctly to pass. (That means that simply flipping a coin would give you 50% chance to pass.) During the semester, each lesson was essentially explaining why the answer to a particular question is “yes” or “no”. Despite that, the success rates were no better than flipping the coin, so a quarter of students failed a repeated exam. So, in order to make their third (and last) attempt more successful, we had to also introduce a few “bonus questions” which provided +1 point if you answered them correctly, but no penalty if you answered them incorrectly. (So at the end you had to get 6 of 18 “yes or no” questions right.)

            I guess we succeeded to keep some infinitesimal plausible deniability while allowing most students to pass with barely any knowledge.

          • Michael Watts says:

            The most extreme case I saw was when the exam consisted of dozen “yes or no” question, and you had answer half of them correctly to pass. (That means that simply flipping a coin would give you 50% chance to pass.)

            Assuming flipping the coin gives you a 50% chance at each question individually, flipping a coin for all twelve questions gives you a 61% chance of getting at least 6 right answers.

          • Randy M says:

            When I taught Chemistry, the practice among the faculty was to go over a practice test the days before the exam, that was, in fact, identical to the real exam.
            I’m not sure if the answers were erased from the board or not before giving the exam.

    • pontifex says:

      There are biological realities at play here, though. If I learn Mandarin today, in my 40s, I will always speak it with an accent. This is true whether I spend 1,000, 10,000, or 100,000 hours on it. If I had learned it as a child, I would have been able to speak it fluently. (Yes, yes, there are a few people who are exceptionally gifted with languages and can avoid this. But almost everyone can’t.)

      I think the same thing is true about subjects like math or logic. You can’t just dive in at 18 and say “ok, now babysitting is over, and real education begins.” Your brain is just better able to learn certain things at an earlier age. It seems like all of the truly gifted people in mathematics, programming, or even music found a way to get involved with those subjects before college. And usually in spite of the school system, rather than because of it.

      • educationrealist says:

        If you are a genuine genius at math, I believe you will be able to learn what you need or get what you need (depending on preferences) in the American school system.

        If you’re just very bright, then you won’t be making any amazing insights into math, and you will be just fine if you never go into advanced math until college. And the reality is that even average bright kids are taking calculus in today’s high school, so that’s not ever going to happen.

        • pontifex says:

          All of the people I know who are even close to genius level in mathematics had some exposure to advanced math prior to college. In some cases, even prior to high school. We also know for a fact that many other skills must be acquired in childhood to be learned well– I gave some examples earlier.

          As this ACC entry says, there just isn’t a lot of data about math geniuses. There aren’t many of them anyway, and there isn’t a lot of documentation or studies about how they grew up. It’s clear that school administration doesn’t prioritize them for a bunch of (possibly valid) reasons. How can we be sure that we’re doing the right thing? It seems like the purest guess to me. And if I had to guess, I’d guess in the opposite direction. It seems just as likely (or maybe even more likely) that we could have more math geniuses than we do if we had more math instruction for them earlier in life.

    • static says:

      ” Seriously. I get it. You were miserable. So? There’s no evidence you would have achieved more had you been able to rush through the curriculum. Get over it.”

      Here’s some evidence. I was a high IQ kid, or at least precocious. I was exceedingly bored in math early on, even more than in reading. I remember in 3rd grade I spent a lot of time being told by the teacher to just help out the kids that were struggling. I moved to a school where math was self-paced. I was progressing at about 2.5 grade levels per year. I started working with a teacher an hour before school every day and did very well on the math team, winning state competitions and placing nationally. I moved again to a school where every class takes one year, and was pushed back two years because they didn’t have any classes at my level. I went to an advanced high school where I was then 3 or 4 years behind the most advanced kids and missed out on a lot of opportunities, to take advanced classes, to compete, etc. despite knowing that I was just as smart as they were. I stopped doing homework, but kept getting 100% on tests and even accused of cheating. Lack of homework completion led to getting a bunch of low grades in classes that were simple for me. I got 5 on 7 AP tests, but only got an A in one of those classes. I was so disappointed in being behind everyone else and not having a shot at a top school that I didn’t even apply to college and was planning to just not go. Luckily one school was trying to pull up their average SAT scores and called me one night asking if I would accept a scholarship. The school was way too easy, but I managed to get over my frustration and transfer to a better school that actually

      It’s not rushing through the curriculum, it’s learning at your own pace, passing tests and coming back to topics with spaced repetition for those that you haven’t mastered the first time around. It’s easily achievable via software and the traditional class should largely just go away at this point. My kids have been attending (private) schools that use this approach and seem to enjoy it, but the public schools in the next county over use it too, and it seems to be working.

      But sure, let’s just pretend it takes one year for everyone to learn the same material, not 6 months for some and 24 for others.

      • educationrealist says:

        I don’t think you understand what “achievement” means in this context, so your sarcasm at the end just makes you look a tad doltish.

    • baconbits9 says:

      First, and this can’t be stated enough: the fact that a bunch of obnoxious people on the internet were bored in school is simply educationally irrelevant. In a this article, I point out that in my own life and my son’s, as well as my experience with bright kids in school, I see nothing but teachers being supportive of bright kids. I’m sure there are exceptions, but the more likely common factor is a bunch of wise-ass kids with crappy social skills who probably aren’t as bright as they think they are.

      Unless your argument is that these complainers are just rare but loud on the internet this is a pretty poor argument. These kids were pushed into the education system and if they came out as wise asses with crappy social skills it is on the education system that is claiming its responsibility (and even right) to educating them. It doesn’t matter if your assessment or their assement is true, either way they are examples of failure of the system.

      Seriously. I get it. You were miserable. So? There’s no evidence you would have achieved more had you been able to rush through the curriculum. Get over it.

      It was, after all, well over a decade worth of these people’s lives. This is excellent evidence that you don’t have empathy for these kids because you would never look at someone that you genuinely thought had been actively made miserable for 10 years and straight tell them to get over it, these people aren’t claiming that things were great except that one year and that messed them up.

      Now if you’re the parent of such a kid, who is genuinely miserable and acting out in math class and causing trouble because he’s not allowed to move ahead, you can pay a bunch of money for private school or you can pay for an enrichment tutor, but in either case you should start by making sure your kid behaves in school so that others can learn.

      There’s no evidence you would have achieved more had you been able to rush through the curriculum.

      So I am confused, it doesn’t really matter how fast individuals learn stuff, but it matters that your kid doesn’t slow down the rest of the class? If learning speeds don’t matter much why would slowing down a whole class actually matter? Beyond this our current system punishes kids who do keep out of the way, I got in trouble for reading unrelated material in class routinely, then I got in trouble for sleeping in class in later years (I would have stayed awake fine had I been allowed to read what I wanted).

      Because remember this: some kids learn faster than others, but there’s a wall out there. If you treat your kid as brilliant and accelerate him through middle school, he or she could hit a wall and learn oh, awit, I’m just kind of ordinary. That’s a really tough lesson to learn at 15, particularly if your parents set such a store on how brilliant you were. Past tense.

      And we have ABSOLUTELY no concept of how to educate bright kids verbally

      You contradict yourself again. If we don’t know how to educate bright kids then how can we know there is a wall? I hit my “wall” in high school with calculus, and struggled in statistics when I took it in college. A few years later I taught myself statistics well enough to make a living as a professional poker player… well OK this is chicken and the egg because I was already playing and winning at poker but learning statistics coincided with moving up to higher stakes where I could make better money. Then I taught myself calculus from the book well enough to get a B (needed to pass the course) on a final for a class I had barely attended all year (when I stopped working to play poker professionally I picked up a full load of courses so my resume wouldn’t be blank for a stretch if the poker thing didn’t work out).

      We don’t really know much about the actual existence of these walls because the system is “if you aren’t noticeably good at this stuff in the specific way we teach by 12, or if you struggle for a year after that at all you pretty much get booted from the special group”.

      • educationrealist says:

        Yeah, this whole response is kind of pointless. Have fun.

        • marshwiggle says:

          Pointless because you personally are not listening? I can see roughly how you’d disagree with some of his arguments. I can see dismissing personal anecdote as irrelevant. But pointless? It really does look like something in striking distance of a polite and good faith response. Or are you treating disagreement with you and with the school system as a personal attack on you?

      • educationrealist says:

        “Unless your argument is that these complainers are just rare but loud on the internet this is a pretty poor argument. ”

        How is it not obvious that this is what I am saying? People are confusing a loud part with a common part.

        “It was, after all, well over a decade worth of these people’s lives.”

        Even if you assume that the kid got 100% of his education outside of school, which is incredibly unlikely, if the worst that happens to that kid is that he was smart enough to be bored for ten years of school, that’s a lucky kid. But in general, that’s not what actually happens, so I find your offering of this a tad hyperbolic.

        “This is excellent evidence that you don’t have empathy for these kids because you would never look at someone that you genuinely thought had been actively made miserable for 10 years and straight tell them to get over it, these people aren’t claiming that things were great except that one year and that messed them up.”

        This is excellent evidence that you don’t write very well. And since I myself had the same issues, you might want to consider that you don’t know what excellent evidence is.

        “So I am confused, it doesn’t really matter how fast individuals learn stuff, but it matters that your kid doesn’t slow down the rest of the class?”

        No. That’s not what I said at all. It matters that your kid doesn’t disrupt the class. You do know the difference, right?

        “I got in trouble for reading unrelated material in class routinely, then I got in trouble for sleeping in class in later years (I would have stayed awake fine had I been allowed to read what I wanted).”

        Quite right. You shouldn’t do either one. Presumably the teacher gave you something else to do, or told you to do work you hadn’t done. If the teacher didn’t give you any work and said the work you had done was correct, then by all means go to the principal and demand to be given extra work.

        “You contradict yourself again. If we don’t know how to educate bright kids then how can we know there is a wall? ”

        Is it your impression that when I say “we don’t know how to educate bright kids” it means that they learn absolultely nothing, even less than non-bright kids? That seems a tad literal.

        Obviously, I meant we don’t know how to educate bright kids in a way that allows them to develop and challenge their intellect, NOT just rush them through a curriculum. and build (often temporarily) their knowledge.

        “Hitting a wall” on the other hand, is talking about knowledge, and the problems that occur when a kid who has had it easy all through school is suddenly face with an intellectual challenge he or she doesn’t instantly understand. Students who have never faced a challenge often don’t know how to learn.

        Every single person who read this understood what it meant. Except you.

        “We don’t really know much about the actual existence of these walls because the system is “if you aren’t noticeably good at this stuff in the specific way we teach by 12, or if you struggle for a year after that at all you pretty much get booted from the special group”.”

        We know a lot about the existence of the walls, but America’s resistance to tracking is precisely why kids DON’T get booted from the special group.

        You really don’t appear to understand the issues at all, which is why I didn’t bother responding to you the first time.

        • arlie says:

          “It was, after all, well over a decade worth of these people’s lives.”

          Even if you assume that the kid got 100% of his education outside of school, which is incredibly unlikely, if the worst that happens to that kid is that he was smart enough to be bored for ten years of school, that’s a lucky kid. But in general, that’s not what actually happens, so I find your offering of this a tad hyperbolic.

          I don’t know about “hyperbolic”. But I remember, as a child, wishing that the habeas corpus act applied to those under the age of majority, so that the state would have to provide at least some evidence of criminal behaviour before incarcerating me in an “educational institution”. I was quite clear that as a child, I was nothing but property – sometimes property of my parents, and sometimes of the state, but never to be allowed self determination, any more than any of the slaves in societies I read about in library books outside of the totally useless history classes.

          Of course the fact that I felt this way would just be more evidence of how I was immature and needed to be protected from the bad decisions I would otherwise have made. And to most people the argument that people under the local age of adulthood need this kind of protection is obviously valid, unlike similar arguments once equally earnestly applied to people with dark skins, or female bodies – and accepted as valid at the time even by some of those affected.

          For the record, I was probably post-puberty by the time my ideas became this clear – i.e. old enough to be raising my own children, in past societies with different ideas about the age at which children beome adults. Or at least close to it. And I was also aware of such historical differences. So we’re not talking about an infant here, whatever the term “child” may be implying to people.

  20. VolumeWarrior says:

    I thought this post was quite bad. Regardless of my opinion on the content, there was an extreme amount of very basic exposition. Like the authors were determined to signal how careful and thorough they were being by enumerating the simplest most milquetoast points over and over again. Even the conclusions are boring and common-sense. What did this post teach anyone that can’t be guessed?

    The entire post can be probably summarized by:

    “Many people are frustrated that school is too easy. Unfortunately, there is nothing obvious parents can do to change educators’ behaviour. Private schooling options and self-study are the only realistic ways to significantly advance your education”.

    Bonus:

    “There are tons of free lectures available online from the best educators in the world. Virtually no one uses these, which implies that high levels of education are not important to most people. In reality, people tend to have specific pragmatic goals, and advice for how to achieve them flows very easily. For example, there is a simple, actionable, and reliable plan for how to get into med school. I understand this is very boring and does not serve the blogger’s purpose to virtue signal about a subject, but it is true nonetheless.”

    I’d love to say something prosocial like that I appreciate the effort that went into this post, but I didn’t learn anything. It’s 99 pages long and just seems like a blemish on the SSC community.

    • roystgnr says:

      It definitely would have been improved by a pass editing for length… but not as much as you’d expect on first glance, thanks to a catch-22 applicable to most writing on controversial topics: Either you anticipate and write something to preclude every objection, or you get pounced on with every potential objection whose prebuttal you omitted, and by the time the ensuing back-and-forth is done you’ve written much more than you omitted.

      • VolumeWarrior says:

        I don’t think the authors said much controversial. They emphasize tautological statements like:

        if a child wants to be accelerated and seems academically prepared for it, acceleration will usually help them

        With regards to anticipating (spiking) objections, maybe this is fine in small doses. But you should be really suspicious if you need to read a nauseating amount of boilerplate just to be told that students who are ready for advanced courses can benefit from advanced courses.

        • pontifex says:

          “If a child wants to be accelerated and seems academically prepared for it, acceleration will usually help them” is not a tautology.

          For example, when a kid advances to 5th grade and skips 4th grade, he might leave behind his whole peer group and have to make new friends. When a kid goes to college at 16 rather than 18, she will be smaller than the other kids and less mature in a lot of ways. I think it’s good how the authors presented data and arguments in favor of acceleration.

    • JohnBuridan says:

      I thought the authors made a clear case for seeing school as a social environment which leverages human sociality to get them to learn non-obvious school subjects. There is no inherent drive to do Algebra or learn online.

      I thought the part about online learning was especially good, because it pointed out that online learning is unsuccessful not because school is “virtue signalling”, because sustained online learning is difficult even for high IQ kids.

      • Matt M says:

        The problem with online learning is mostly a lack of motivation.

        The authors concede that public schools solve this problem in the most direct and obvious way possible. They throw you in jail if your children don’t show up. Which highly motivates parents to highly motivate their children (also often backed up with physical force when necessary) to show up.

        And if you’re going to be forcibly confined to a location where people are trying to teach you things for 8 hours a day, you’ll probably learn some things in the process, even if you don’t really want to, because you don’t really have much of an alternative.

        If we were able/willing to lock kids away in a closet for 8 hours a day 9 months a year with nothing but a computer and a khan academy connection, they’d probably learn some math, too.

        • pontifex says:

          If we were able/willing to lock kids away in a closet for 8 hours a day 9 months a year with nothing but a computer and a khan academy connection, they’d probably learn some math, too.

          We did that experiment, and all we got was 4chan.

        • Nornagest says:

          IME, the worst problem with online education is that it only works for stuff that can be evaluated algorithmically or damn close. That’s great if you’re teaching linear algebra or something, but it chokes on anything where you need to write English or any substantial code that can’t conform to a very strict I/O spec. WRT the latter, I’ve taken both Andrew Ng’s machine learning course and Dan Boneh’s cryptography course; the latter stuck with me much better, because Ng had me writing twenty-line Matlab snippets for matrix manipulation inside a giant machine learning edifice that had mostly been done for me, and Boneh had me writing C++ from scratch and just looking at the output for the actual graded material.

          And I’m probably the ideal student for both courses: I have real academic experience with the former (albeit several years ago) and industry experience with the latter. I’d expect someone without that kind of background to end up with even worse retention.

      • VolumeWarrior says:

        I said their article was virtue signaling, not that going to school itself is virtue signaling.

        I thought the authors made a clear case for seeing school as a social environment which leverages human sociality to get them to learn non-obvious school subjects. There is no inherent drive to do Algebra or learn online.

        Online education changes two variables wrt public school. The social aspect, and the extracurricular aspect. The rarity of group-extracurricular learning implies that studying for its own sake is perceived as a waste of time.

    • Plumber says:

      I found the essay useful as it’s making me reconsider our decision to homeschool our son.

      • TracingWoodgrains says:

        Reconsider in favor or against? I’m a pretty big fan of homeschooling as an option if the parents have the means to do so, and I see a lot to recommend it as one of the best ways to navigate the tradeoffs within the system.

        • Plumber says:

          Re-considering against.

          He’s been homeschooled for over a year now (a classmate trying to stab him was the final straw) but the studies linked in Scott’s post seem to argue against homeschooling.

          • Randy M says:

            FYi, in case it’s not obvious, this post is not written nor necessarily endorsed by Scott Alexander but by TracingWoodgrains and Michael Pershan.

          • Skivverus says:

            I’d stick with the homeschooling absent a viable non-stabby school alternative. Whatever socialization benefits public schooling in general might have, in your specific case I’d look elsewhere.
            Mind you, I’m biased; was homeschooled myself through high school, and I’m reasonably satisfied with how I turned out.
            Don’t know what the homeschooling community is like over in your (part of your) state, but it’s worth a look to maybe reach out, per the “socially rich” discussion higher up: having other parents who can teach the subjects you’re not as confident on is handy.
            Also depending on how far along your kid is, look into community college or trade school courses (13, now that I’ve reviewed, means “probably not yet”).
            There are also online courses out there; I’d experiment with one to see how he does with it focus-wise; if you’re satisfied with the results, you can go for more, and if not, you can stop. (Though again, positively biased from personal experience here)

          • Plumber says:

            ^ Thanks for the correction @Randy M, they did a fine job.

            ^ Thanks for the advice @Skivverus, that’s probably what will be done.

    • R4P says:

      Hi, I’m one of the people in the acknowledgments section at the end so I have a bit of insight into the drafting process the authors used.

      First, I would like to say that research projects of a literature review nature like this tend to have a common problem among those who don’t write papers for a living: spending too long researching and then having to make a mad dash right before the deadline to edit what you’ve found into something roughly presentable. (Some tell me that professionals have this problem, too, though I imagine it is less common.) In this regard I would suggest giving the authors a break since this entry was submitted by the original deadline and not the extended deadline.

      As for the specifics of it being too long, I don’t know what the state of the project was before it got to me, but as an example I saw a draft about two weeks before the deadline where the first section was 1400 words long. This ballooned to 1800 words a week later (a result of the research stage going too long, as I mentioned). I proposed a re-write of that section about 1500 words long as a first pass suggestion, and in the intervening week the authors managed to whittle it down to the 1700 words you see before you in section 1. The other sections were similar. Turning this 20 page research paper into the 15 or 10 page research paper it probably should have been is not something easily done in your time after work in a single week, especially fresh from writing it where you will be attached to every word.

      That said, it’s still shorter than some published literature reviews on similarly comprehensive topics so I think the length is not unreasonably long even if it could have clearly benefitted from 2 or 3 more weeks of editing in my opinion.

      Secondly, I often see criticisms of “no duh” research. This is research with conclusions like “romantic relationships go better if the two people in them are temperamentally similar.” The problem is that “no duh” research is not so obvious as it looks — geese of a feather, yet opposites attract. Resolving these supposedly common-sense notions empirically is one of the most common and important things the academic project is about. As an example, Kahneman won the Nobel prize in economics for engaging in some “no duh” research. Of course everyone is a rational economic agent trying to maximize expected value for themselves. Oops, guess not.

      While I am not happy with the current state of educational research by a long shot, the fact remains that there is a debate on this topic and a lot of smart and knowledgeable people come down on different sides of it (as well as the ignorant and uninformed). Gates is banking on technology for the future of education, this paper says otherwise. Slavin says tracking shouldn’t be in schools, this paper suggests a caveat about how sometimes it should be. Given how routinely technical authors badly overestimate the existing knowledge of their audience I think including the basics was probably for the best. Even so, as a professional educator myself I found some of the evidence presented here to be new and informative, like the 2018 Direct Instruction meta-analysis.

      The point of the adversarial collaboration contest, as I understand it, is to get two people with two different opinions on a topic to perform a literature review that comes to what seem like the most accurate overall conclusions about the state of the research on that topic. I do think this project ultimately did a good job of that, even if I also think it was a bit long and the action steps in the conclusion were necessarily somewhat frustrating owing to the subject chosen. It certainly wasn’t 99 pages long (as mentioned, it was shorter than many similar research papers), it clearly endorsed several specific positions in an existing debate, and to call it a “blemish” almost indistinguishable from intellectual preening seems much too harsh.

      • VolumeWarrior says:

        Turning this 20 page research paper into the 15 or 10 page research paper it probably should have been is not something easily done in your time after work in a single week, especially fresh from writing it where you will be attached to every word.

        I understand that I am coming off as cartoonishly cynical, but for all intents and purposes I rewrote the entire post above in a couple of sentences. It doesn’t have citations, but I’m arguing that the citations serve to signal the status and authority of the paper, rather than actually navigate a difficult topic.

        What, in your mind, is a complicated feature of the topic that requires X-thousand words and a bunch of citations?

        Why does all this complicated hemming and hawing lead to the most boring common sense conclusion that anyone could reach without studying?

        That said, it’s still shorter than some published literature reviews on similarly comprehensive topics so I think the length is not unreasonably long even if it could have clearly benefitted from 2 or 3 more weeks of editing in my opinion.

        We will not debate the average quality of published literature reviews.

        As an example, Kahneman won the Nobel prize in economics for engaging in some “no duh” research. Of course everyone is a rational economic agent trying to maximize expected value for themselves. Oops, guess not.

        I agree. It’s not obvious that opposites attract, or that geese of a feather stay together. However I do know an obvious statement when I see one, and that is when students are ready for accelerated classes, accelerated classes can help them. This is basically tautological.

        Don’t eat too many pop tarts.
        People accustomed to exercise will not find exercise overly difficult
        The president is doing a good job if his policies and actions are good.

        How, pray tell, would you do #science to investigate any of these claims?

        I found some of the evidence presented here to be new and informative, like the 2018 Direct Instruction meta-analysis.

        This is fair. It is possible that you’d want to shine light on the debate between direct instruction vs. transfer learning. But this collapses once you have specific goals. If you want to get into med school, study for those tests. If you want to keep your options open and become generally smart, study things in general.

        Once you have an objective, you have a gradient. The gradient tells you what to do. Debating things from the position of “how can schools educate advanced students better” is vague and confusing.

        there is a debate on this topic and a lot of smart and knowledgeable people come down on different sides of it

        If there is one thing you can rely on intelligent people to do, it is to signal their intelligence.

        Gates is banking on technology for the future of education, this paper says otherwise. Slavin says tracking shouldn’t be in schools, this paper suggests a caveat about how sometimes it should be.

        I admit that questions can become complicated very quickly. For example, probably no one knows the effects of teaching algebra +/- 1 year in the curricula. It doesn’t mean we should spend a lot of energy figuring out the answer to that question. The stakes are low, and again, the goal is poorly defined.

        If you want to learn algebra, the common sense approach is to try to start learning it. If it’s too hard, go to its prerequisite topics and fortify your weaknesses.

        I challenge you to find any way in which writing thousands of words and citing dozens of papers can significantly improve that algorithm.

        Given how routinely technical authors badly overestimate the existing knowledge of their audience I think including the basics was probably for the best.

        Basics are great! But basics are basic. That’s my point.

        I also think it was a bit long and the action steps in the conclusion were necessarily somewhat frustrating owing to the subject chosen.

        So, you agree that it obvious weaknesses. But you still can’t shake the feeling that it was overall good.

        • R4P says:

          While I am impressed with your summarizing ability I do not think it is within the spirit of the contest to write without citing your sources. The point of the contest as I understand it is to review the literature, which means you must cite the literature. Having talked with at least one of the authors I do not think they were embarking on a quest to show off how smart they are but rather simply trying to do a thorough and scholarly job. Perhaps you find the citations to be redundant because the educational literature itself that is being cited is not very good, but that is not the fault of the authors here.

          I also do not think it is fair to ignore the quality of similar publications as you suggest we do. By the standards of similar publications they have done what looks to be a fairly good job given the short time frame. If you think professional literature reviews are all badly written then my only suggestion is perhaps you should go into academia and show them all how it is done.

          As for what is or is not obvious, I would recommend reading the paper again more thoroughly to identify the non-obvious conclusions offered. Certainly “when you are ready for accelerated classes you should take accelerated classes” is pretty obvious, but it is premised upon something much less obvious: that tracked or accelerated classes can work under specific circumstances that the authors provide. (If you think it is obvious that they do or do not work under such-and-such circumstances then that is good for you, but on balance the rest of the planet has not yet come to consensus.)

          Once you have an objective, you have a gradient. The gradient tells you what to do. Debating things from the position of “how can schools educate advanced students better” is vague and confusing.

          I’m afraid I do not know exactly what you mean by this. If it is an important point then you will have to explain it further.

          If you want to learn algebra, the common sense approach is to try to start learning it. If it’s too hard, go to its prerequisite topics and fortify your weaknesses. I challenge you to find any way in which writing thousands of words and citing dozens of papers can significantly improve that algorithm.

          Well, that’s right as far as it goes, but it does not go very far. Someone wanting to learn Algebra who did not know where to start would not get very far with advice like “just learn it.” If you can appreciate that learning involves actions that are more specific than “just learning it” then you should also be able to appreciate that this is what the above article was trying to figure out: the ways in which “just learning it” are different if you are really intelligent, if any, and what we should do about that, if anything.

          So, you agree that it obvious weaknesses. But you still can’t shake the feeling that it was overall good.

          Good work can have flaws in it. In fact, most good work does. This is a good summary of the state of the research on how learning and teaching are different for gifted and talented students and their teachers and what we ought to do about it. It is a bit too long and it didn’t have a silver bullet, but it did the job it set out to do perfectly well. If you knew all of this already that is fine but that just means you were not the paper’s target audience.

          • VolumeWarrior says:

            I do not think they were embarking on a quest to show off how smart they are but rather simply trying to do a thorough and scholarly job.

            It’s the same thing.

            I also do not think it is fair to ignore the quality of similar publications as you suggest we do. By the standards of similar publications they have done what looks to be a fairly good job given the short time frame. If you think professional literature reviews are all badly written then my only suggestion is perhaps you should go into academia and show them all how it is done.

            Win that big cash prize for writing simple and easy to understand articles…

            that tracked or accelerated classes can work under specific circumstances that the authors provide

            The authors don’t outline specific circumstances. Their papers just throw acceleration at gifted kids, and they generally do well. There is some variance based on the definition of gifted and the level of acceleration. This is all 100% expected.

            Now, it’s nice that these studies have been done. For example, if accelerated gifted kids produced the OPPOSITE result, that would be worth mentioning. But since they have been done and the result is very, very strongly predicted by common sense, you need between 0-1 sentences describing this. For example:

            “In general, accelerating children in proportion to their giftedness produces modestly positive results [citation 1-5].”

            I’m afraid I do not know exactly what you mean by this. If it is an important point then you will have to explain it further.

            See the going to med school example. If you know you want to go to med school, there’s a laundry list of things you need to do. Go do them.

            Someone wanting to learn Algebra who did not know where to start would not get very far with advice like “just learn it.”

            They could google “learn algebra” and get first rate material.

            Maybe the mechanics of learning are really complicated. Maybe you can do crazy analyses to show that method X is objectively the best way to learn. Who cares. The difference between random public school curricula and dedicated attempts to “just learn” the material is so great as to dwarf observations that there might be different ways to learn things.

            Every single student goes to college and “just studies”. They normally achieve their goals. And if they don’t, the advice is to study harder, not different.

            If there were a silver bullet study method, students would have found it. But there’s nothing there – no shortcut. You have an exam on algebra, you practice doing algebra problems. I don’t need to do a literature review to show me that some learning method is better by 6.3% (p=0.021).

            This fruit hangs low man.

      • Douglas Knight says:

        two people with two different opinions on a topic to perform a literature review

        Is that the point? Do we really need more literature reviews? Why aren’t the authors deferring to existing literature reviews? This seems like the worst of both worlds. If the authors spend all this time signaling how much they’ve read, I’d like them to signal that they’ve filtered out the complete garbage. Mainstream literature reviews have difficulty sending such signals, or even ignoring bad work, so the main value of having amateurs doing literature reviews is that they are not beholden to existing hierarchies.

        I think this would be much more valuable with the context of the evolution of the authors’ views over time. Maybe they disagreed because one or both just had no idea what the literature said, so this is just reporting what the it says. Or maybe they had beliefs based on specific works and one of them convinced the other that one side had problems. Then it seems valuable to point out those problems.

        • pontifex says:

          I enjoyed the post a lot. I think it’s one of the best things to appear on SSC in a while. This post is far from being a “literature review.” It takes some very controversial positions in favor of tracking and acceleration.

          A few people in this thread seem to be saying that because the post comes to similar conclusions to what they already intuitively thought, that it has no value. Guys, opinions are cheap and easy to come by online. Carefully done research is not.

          I agree that it would have been nice to hear a little bit more about how the authors’ opinions differed, and how they changed after the collaboration (if at all).

          • Douglas Knight says:

            Are they controversial to people who read the literature? Or are they controversial because activists lie about the literature? In the latter case, they should have said that they were doing a simple literature review, and that this was the reason. In the first case, which you seem to endorse, they should have explained how they chose these studies over the other studies. For example, they cite a paper that says that Gamoran is blight upon the earth. So why do they cite his other papers?

          • Douglas Knight says:

            To put it another way, merely taking sides in a controversy is not very useful. Lots of people have done it before.

          • pontifex says:

            Are they controversial to people who read the literature?

            Yes. Go back and read the part about how “we are all living in Bob Slavin’s world.”

            Or are they controversial because activists lie about the literature?

            I’m not sure what point you’re trying to make. Opposition to tracking and acceleration is not some fringe position being pushed by activists. It’s mainstream policy.

            To put it another way, merely taking sides in a controversy is not very useful. Lots of people have done it before.

            Your comments are kind of incoherent. First you accused it of being just a literature review. Now you’re accusing it of being just an opinion piece with no research. I’m starting to question whether you read the whole thing.

    • TracingWoodgrains says:

      Thanks for the feedback!

      If our common-sense statements were widely accepted as common sense and practiced within the education system, we wouldn’t have bothered writing anything. If one or two sentences could provide sufficient impetus for people to re-examine their views on some of these issues, we’d use one or two sentences. I’m happy if you learned nothing from the post, since ideally that means you’ve already internalized the useful parts of it for whatever position you happen to be in. Given that many of our “simple, milquetoast points” are pretty far out of the educational mainstream, particularly surrounding education and ability grouping, that’s just not the case for a lot of influential people.

      That said, the summary you gave of the online education section is a good example of why we did write 99 pages and not 9 words. Our aim was not to convey that “virtually no one uses [free lectures online], which implies that high levels of education are not important to most people,” but to point out that even for people who find high levels of education genuinely important, there are more important structural factors to learning than providing free lectures from the best educators. It’s not that people don’t want to learn, but that the online lecture format is a poor way to teach people, and if we’re serious about developing good education tools, we should proceed based on how people actually act and not on some idealized version of the “motivated learner” who does everything on their own.

      • VolumeWarrior says:

        If our common-sense statements were widely accepted as common sense and practiced within the education system, we wouldn’t have bothered writing anything.

        Actually you could have written about this. Why aren’t super obvious improvements made? Seems a lot more interesting than pointing out that advanced students sometimes do better with advanced coursework.

        If one or two sentences could provide sufficient impetus for people to re-examine their views on some of these issues, we’d use one or two sentences.

        Yes. I do think it is possible that spilling a very large volume of text and citations signals intelligence, conscientiousness, and authority. What I’m saying is that the SSC/rationalist community shouldn’t fall for the whole form over substance shtick.

        It’s not that people don’t want to learn, but that the online lecture format is a poor way to teach people

        How do you know this? Because no one uses it? There are multiple reasons not to use it. Being a poor resource is a possibility, but you can directly observe that online material actually seems very good. Not only are there lectures from the best teachers in the world, there are tons of simple tutorials and quizzes online for popular topics. And I suppose they could still be awful, but the Kahn Academy youtube video for “Introduction to limits” has 3.7m views and a tiny ratio of negative feedback. This isn’t what you’d expect from material that is difficult to learn from.

        The way better hypothesis for the unpopularity of online learning is that people don’t care. Even a very smart student gains no benefit from learning calculus early – he’ll just get an A when the time comes. And forget about more advanced topics. How, pray tell, could a high school student leverage knowledge of differential equations into a better college outcome that is more effective than just studying for the SAT?

        and if we’re serious about developing good education tools

        See my point about how “good education” is vague and not a goal that anyone actually has.

        we should proceed based on how people actually act and not on some idealized version of the “motivated learner” who does everything on their own.

        To be clear, I am not recommending that people use online videos to get super educated. The fact that unlimited education is available for free implies that people do not actually value education. Common sense says they value passing tests for courses they’re actually taking. Not a value judgment, just a fact.

        • Michael Pershan says:

          I personally feel as if we did write about why things that seem “obvious” don’t get implemented in education. That’s part of what I think our whole 4th Section (“Educational Goals In Conflict”) is about. And based on how everybody seems to talk about education I think it’s non-trivial.

  21. fluorocarbon says:

    I really liked this post, but there’s one question I have that wasn’t addressed in the online learning section: has there been any research into the efficacy of online learning beyond conscientiousness and completion rates?

    I’m pretty conscientious and about a year ago I completed the Coursera course in machine learning (including all the programming exercises) in addition to the English/German and German/English Duolingo trees (I’ve never taken an in-person class in either). The thing is, I don’t remember anything I “learned” in those courses. I remember more from the semester of Spanish I took in university than from the Duolingo trees and the same goes for the handful of CS classes I’ve taken in person and the Coursera course.

    For me, I think the benefit of learning in a non-virtual environment like a school lies in talking over the subject matter with other students, specifically live non-virtual humans who are learning the same material. I wonder if that’s actually true, if it just seems true to me, or if most people don’t learn this way? I’d be really interested in seeing research about this.

    • oqpvc says:

      How many hours in total did you invest into the Coursera course? How many into your other CS classes? From my very limited experience with Coursera, I remember that I could complete certain courses within two afternoons – can’t say that about my uni classes though.

      • fluorocarbon says:

        This is the course I took: https://www.coursera.org/learn/machine-learning
        It’s originally a Stanford course and it’s taught by Andrew Ng (who’s brilliant).

        For complete context: I wanted to treat it like a real course, so did about one section per week, which means it took me around 11 weeks to complete. I did the readings, listened to all the lectures, and did every assignment. I got a few questions wrong on the quizzes, but I found the programming assignments trivial. My overall grade was something like 97%.

        For whatever reason, the information did not stick with me. The only two things I remember are 1) that there need to be three sets of data: a test set, a training set, and something else I can’t recall and 2) that linear regressions are really good for a lot of problems. I don’t think I’m particularly stupid or a bad learner (although that’s certainly possible) but studying in a university environment was more effective for me, and not just for signaling or conscientiousness/motivation reasons. (Of course, that doesn’t mean universities aren’t wasteful or even particularly good, just that I found them better than online learning.)

        I also remember a lot less of a book if I read it on an e-reader versus on paper. I think this might be a related phenomenon?

        • Lambert says:

          The question is: Given a dataset, can you program a reasonably accurate classifier?

          One of these, perhaps. https://archive.ics.uci.edu/ml/index.php

        • thepenforests says:

          I took exactly that course earlier this year, and I also don’t remember much from it. But honestly, I think it was just not that good of a course.

          Sure, there were “assignments”, but they were basically paint-by-number affairs. “Here’s the complete structure of a neural net already made for you – write a few lines of code in each function”. Almost fill-in-the-blank programming.

          If they wanted to properly teach it, they would have had to, at a bare minimum, make students create a neural net entirely from scratch, preferably several times. Otherwise of course you’re not going to remember anything from it.

    • JohnBuridan says:

      I think that is true, matches my experience, and is in line with the research.
      When I learn something new, I have a drive to talk about it with someone and discuss it. In the course of conversation I discover whether or not I understand something, and reinforce what I know. In fact, in a weird way, I basically spend time rehearsing author’s arguments in my head, so that I can grok difficult concepts.

      Information is retained better in the context of a social or emotional connection. Sorry I am not digging up the research on this right now. Got to go.

    • R4P says:

      I don’t have the research on this but I suspect you are on the right track. Practice time (manipulating information into hierarchies of importance and categories of relevance for facts and trying to do a procedure with speed and accuracy for skills) should usually be longer than learning time if you’re interested in achieving at least a moderate long-term facility with a subject. This is why accredited university courses usually assume you will spend 1 to 2 hours of independent study time for each hour of class time, and it’s why Gwern estimated that to memorize a single simple fact (which would take only seconds to state aloud) across your life-time takes about five minutes of spaced repetition attempts.

      Lots of information is available online but it’s rare to see the volume of practice truly necessary for mastery packaged alongside it, even in official online courses. For instance, Khan Academy has excellent instructional videos but their practice materials and quizzes often take less time to complete than watching the instruction when the reverse should be true.

      Talking the material over with people is one great way to perform manipulation of the information in a complete and complex way – you’re forced to consider what’s most important and what’s relevant to the topic of conversation at every moment. It also helps create new memories that reference the information so there’s more chance of something triggering the memory you need. So it certainly makes sense that this helps you retain information. There are certainly other ways to do the same thing, like writing, answering question prompts, or revising your notes. Point is, though, unless you do that enough you’ll forget almost everything within a year, and I suspect many online courses provide mostly the information and leave the practice to the student who often fails to do enough of it.

    • Loriot says:

      As a counterpoint, I’ve independently studied several languages as an adult using Duolingo and I think it has been a success. I may not remember everything or be conversationally fluent, but I think if you are motivated and want to get to a basic level of skill in a language quickly, it is a good method. In my case, my goal was to be able to watch foreign movies without subtitles, and while I can’t understand French, Spanish, or Dutch as well as I can understand German (where I took 5 years of formal in person classes), I can still understand a lot of it. In simple cases like a children’s show that doesn’t use slang or accents, I can understand basically everything, which is pretty good considering that I spent an order of magnitude less time on them.

      P.S. I wasn’t only using Duolingo of course. I also spent a lot of time watching TV and videos in the target language, which may have helped with the “practice time” people mentioned above. But Duolingo is still a good starting point.

  22. Carl Milsted says:

    Fascinating.
    I’m frankly surprised that the evidence in favor of tracking is as weak as stated. My elementary school experience was that very little got taught in the subjects for which we didn’t change classes. (Homeroom was not tracked. Reading and math were.)

    Never heard of the Joplin Plan, but I have long thought something like it would be good. Where I went to school, there was great resistance to changing a student’s track, even if he was on the edge of failing. (And the “student” I am thinking of pulled the pace down continuously.) And there was no difference in track for language arts vs. math. The problem with it is that it is hard for teachers to specialize. You need all the English classes at the same time, and all the math classes at the same time. (Or you might be able to manage a split by have two times for each and students taking electives during one of each of the two periods.)

    Another reform that interests me is getting rid of letter grades and using a levels system like video games or Dungeons and Dragons. Letter grades inspire corruption by teachers (curving), and envy against brighter students (broke the curve!).

    • yodelyak says:

      Another reform that interests me is getting rid of letter grades and using a levels system like video games or Dungeons and Dragons. Letter grades inspire corruption by teachers (curving), and envy against brighter students (broke the curve!).

      So much this.

      I was up in arms for a hot second about my high school putting up a wall of photos of their valedictorians since days gone by, when they could just as easily put up a wall of students getting 4s or 5s on AP scores. Unfortunately I live in another town now, and my voice counts for bupkis. But seriously, there is zero correlation between the number of valedictorians a school has in a given year, and how well the school did that year!

      Excellence and achievement, not relative rank.

      • Matt M says:

        Eh, I see relative rank as a necessary pushback against things like grade inflation and participation trophies.

        It’s basically a way to force people who would otherwise want to hand out accolades too freely to be stingy with them. And it essentially guarantees that the accolade then cannot be doubted or called into question.

    • TracingWoodgrains says:

      Another reform that interests me is getting rid of letter grades and using a levels system like video games or Dungeons and Dragons. Letter grades inspire corruption by teachers (curving), and envy against brighter students (broke the curve!).

      The current initiative in schools that comes closest to this is probably mastery-based learning, where students are presented with a list of skills to master and ostensibly take the amount of time needed to learn them before moving on. This article from the Hechinger Report is the most recent one I’ve seen mention it. It has a lot of potential in my view, but there are some kinks to work out and it hasn’t yet demonstrated itself as clearly and unambiguously better than traditional grading. I’m keeping my eye on it.

  23. sclmlw says:

    Very nice. It feels like a great resource to parents and educators looking to improve results for their students. Very well-sourced, informative, and balanced.

    That said, I felt like only the first part adequately focused on the question itself: “Does The Education System Adequately Serve Advanced Students?” And there I found the discussion didn’t really address the core of the question. Does the system adequately serve advanced students compared to what? In other words, what’s the evidence that less-advanced students are more well-served than advanced students? For example, say you take children who would otherwise go to public school and transfer them to a private school. Do you get MORE of an improvement for the advanced students than you do for the other two? If so, that would suggest we’re using public school resources more effectively to help the low-achieving students and not to help the advanced ones – who get those resources once they start paying for them directly.

    Looking only at where resources are allocated doesn’t, to my mind, answer the question of whether low-achieving students are getting the better end of the bargain, to the detriment of high-achieving students. An analogy: say you’re teaching a kid to read and someone suggests that the best aid for this is expensive socks with hundreds of tiny built-in magnets. You apply these liberally to one group, but not another. After doing this for years, spending huge sums of money, you discover that fancy magnetic socks have no effect on learning how to read. Does the fact that you spent more money on one group mean that the other group was under-served? And there’s where I would like to have seen more discussion: to what extend do interventions for low-achieving students help in a way that is disproportionately meeting their needs versus other students? Where’s the direct comparison?

    I know what I’m asking for would make this post much longer, but I feel like it’s at the heart of the question that spawned the adversarial collaboration. It’s probably easier to craft a document saying what we should do. But what about what we’re already doing? Are we actually helping some kids, but ignoring others? Or is it equally crappy to have some new, bureaucratic, zombie fix applied that is driven by political motivations to “fix the system” but has no controls or accountability? What if it’s better to be left alone and bored in a classroom than to have well-meaning – but ultimately bad – interventions?

    • TracingWoodgrains says:

      The question that spawned the collaboration is honestly a bit of a muddle, and only tangentially related to the title (which was, to be honest, chosen for us). My initial invitation to collaborate was as follows:

      I strongly support ability-based grouping, early specialization, gamification and game-based learning, and the critical priority of developing and using high-quality online education tools with an explicit goal of being entertaining. My stance is that the current culture and focus of US/western education is actively damaging many of the brightest and most eager students within the system and that the bulk of the conversation around it is focused in the wrong direction.

      It circled around to mostly the question given in the title, but because of the roundabout process we took to reach that question, we focused more on the topics we started from (ability grouping, acceleration, online learning).

      On the topic of the efficacy of cost interventions, there’s a very solid case to be made that much of the money is being thrown away on minimum-impact initiatives. This case study is long, and politically motivated, but talks about the time Kansas City schools received something like a 4x increase of funds to help disadvantaged students and ended up moving the needle very little.

      My quick answer is that all children are being underserved from an academic standpoint, but that gets back to the competing goals within education–it’s deliberate, as much as such things can be, based on principles people value with schools. Interventions that work but go against those values, such as Direct Instruction (which has a complicated and fascinating history that we didn’t find space to share in the article) usually fall by the wayside. Private schools are not usually dramatically different from public ones in the core tools they use to teach kids.

      Resource allocation matters, but it’s less who the money’s being spent on and more how the money is being spent, and that’s where the problems really come. It’s a large topic, but a fascinating one and one I hope to write more on later. You raise a lot of useful questions.

      • sclmlw says:

        Thanks for the clarification. It makes a lot more sense why you chose the direction you did in writing this up. I would be very interested in a follow-up analysis of how effective programs tend to be at promoting the competing values you discuss – since my sense is that they are fairly awful at pretty-much everything. So the next obvious question is, “are these myriad programs actually good at anything?”

        My wife is a teacher (secondary Montessori, so cross-grade groupings and providing accelerated materials is sort of an article of faith at her school) and when she was teaching at public school it seemed clear that the many, layered, bureaucratic “fixes” weren’t just not helping, but that they were actively hurting. Four out of five nights a week she would stay after school to attend mandatory meetings for some defunct/ineffective top-down program intended to “fix education”. The programs ended up taking huge amounts of her time, such that it was difficult to do the grading and preparations she actually needed to do to ensure the students had high-quality materials and labs.

        So my prior bias is not that “these programs work, but are targeted toward different goals than academic achievement”, but that “everyone wants to be seen as trying to ‘fix education’; that’s the only real goal, and in the competition between wanting to be seen fixing education and actually fixing anything the perverse incentive is to look like you’re making things better when you’re actually making things worse.

        Take, for example, the program that gives student loan relief to teachers who teach in low-income schools. The problem with this program (my wife started out in a low-income school) as I saw it is that it encourages new teachers to begin their careers in high-demand environments, before they have a chance to develop good classroom management skills. Many leave teaching – and who can blame them – and those who don’t will try to get out of the failing school ASAP. So a program ostensibly billed as “getting teachers to teach in low-income schools” actually incentivizes the influx of the most inexperienced teachers to low-income schools.

        It’s a regressive policy that uses poor schools to make rich schools better; such that low-income schools serve to weed out the least-dedicated teachers and train up the rest who will go on to teach the rich kids. That’s great if your goal is, “we don’t care about the poor, we just want a crucible to produce better teachers for the rich kids.”

        How many other programs have real impacts that are not aligned with the stated values they are based on, I wonder?

  24. dodrian says:

    I’m confused as to what actually is meant by tracking. Does it apply only to different courses in elementary school? Or is it disapproved of in high school as well?

    My school began tracking math in 6th grade, science in 8th grade and most other subjects in high school. The tracks would either cover material in greater depth such (my geometry course had a greater focus on euclidean construction and formal proofs), or they’d start the next year’s material earlier (I finished with a year of calculus, whereas other students spent their final year in Algebra II). Students could move up or down a track: I moved up in math when my teacher realized my difficulties were with arithmetic and not comprehension, and I moved down in Spanish because the workload was too much for me.

    I was under the impression this was fairly normal in high schools, was I wrong, or have I misunderstood what the article says and this isn’t controversial?

    • HeelBearCub says:

      We have a federated educational system in the US. It is federated down to the municipal level, and even the school level. This means that educational experiences will be quite different depending on where (and when) you went to school. There is no “typical” school experience. Not really.

      • dodrian says:

        A quick google shows that over 1/3 of students take AP classes, which means a large percentage of schools must offer them. That’s the kind of thing I was imaginging by ‘tracking’, and while I suspect my school did more than most (it was a private school), the adversarial collaboration implies that it’s frowned upon by a lot of teachers/administrators/etc. Or is that only the case when it’s done in elementary school? Or have I misunderstood things?

        • HeelBearCub says:

          I agree that “AP” classes are normally offered in HS. But I don’t think that is the same as tracking. My kids took lots of AP classes, but also took non-AP classes in subjects where AP classes were offered. My sense of tracking is that is a more holistic approach to the academic curriculum, and starts much earlier, in grade school.

          Of course, I could be wrong.

        • Matt M says:

          AP classes are sometimes tracked, but sometimes not.

          Within my own high school, there were some AP classes that had hard pre-requisites (with the prereqs themselves being classes that were gated with a bit of soft-tracking), but others that were open to anyone who bothered to register for them (which, in a practical sense, was a group that overlapped almost completely with the same kids who found themselves in the stricter-tracked classes).

        • sclmlw says:

          “Tracking” is where you put kids in different classes based on projected academic capabilities. So if you have an honors English, a regular English, and a remedial English class – all in the same school (K-12) – we would call that “tracking”. Basically, instead of having all the kids in the same classroom, learning and interacting from the same material (a “heterogeneous” classroom) you have the kids on different “tracks”.

          My own reading of this literature is about ten years old, but all I can recall was being disgusted that most of it is critically flawed to the extent that it’s nigh unusable. One particularly bad study looked at the average of tracked classrooms (low, mid, high) and compared it to the average of heterogeneous classrooms. Their conclusion was that tracking helped advanced kids at the expense of kids who are behind. Of course, you’ll get that result when you compare the average of the low/high with the average of the total!

          There’s some bad research in education (more so than other subjects?); I’ll trust the authors that the body of the useful evidence points to no difference, though.

    • Plumber says:

      On tracking, I’m just going to copy and paste my post from a previous thread:

      “The idea is to change the culture into a monoculture where parents force children to honor their teachers. To be more blunt about it, turn black children Japanese, turn them Japanese.”

      @Le Maistre,

      Yeah I had the “New Clear Days” album as well.

      Anyway, anecdote isn’t evidence, but…. 

      …..1982/1983 despIte my begging to go to Maybeck (a private high school) for 9th grade I attended the Berkeley High School West Campus. 

      The legend passed around in elementary and junior high school was that West Campus was a war zone filled with violence, what I found instead was toilets with no paper or doors, mostly broken water fountains, and everything clearly sized for elementary school kids and too small for high school students (it was closed the next year), but far less violence than elementary and junior high school (no games of “smear the queer” or weekly fist-fights), and unlike “main campus” in subsequent years I was never punched into unconsciousness and then told “That’s what you get for walking alone” by the teacher when I afterwards came to class bleeding.

      For English I was first assigned to the “Intermediate” track rather than the “Advanced” track (the majority of students were “Advanced”), and IIRC I was the only white boy in class, and I don’t remember any white girls, and most of my classmates every day in that class were black girls, and the teacher was a black man.
      No reading was assigned, just the occasional essay (which a couple of the girls in class would ask my advice about, so in that sense they were “disruptive”), and mostly I did homework that was assigned in my other classes or read the Larry Niven novel “A Gift From Earth” which I found on one of the mostly empty seats one day.

      The only time that I really felt uncomfortable was when the teacher would play speeches of Martin Luther King Jr. and ask us to write are reactions, and I worried that I wasn’t writing a “proper” reaction (no I don’t remember much about it beyond my worry at that time).
      When my mother found that I had been assigned the non “Advanced” track she had a fit and insisted that I be moved in the middle of the semester to the “Advanced” English track, which actually had assigned reading (“Julius Caesar and “Great Expectations”), and mostly white students. 

      Every seat was taken, and I had to sit on the floor whenever a classmate wasn’t out sick, and after being yelled at enough times to “Get out of my seat”, I learned to wait for every other student to take their seats first, and wait to see if I had alternate sitting on the floor and leaning against the wall, or if I’d get a desk and chair that day.

      Both my teacher and my classmates were clear that my presence was not welcome in the “Advanced” class and despite loving to read I hated that class, and I grew to hate my fellow whites students and one day I vomited in class.

      I’d have rather have gotten the books to read from the mostly white class and had stayed with the welcoming black students (the girls), and the indifferent black students (the boys), and had a chair and desk, rather than have endured the cruelty of the “Advanced” track, and the disruptions there.

      In my sophomore year at high school I took an elective class and found that I was the only student in that class from “the flats” and not the Berkeley hills, and some of my classmates quizzed me about the location of some damn ski shop in Berkeley (which I had no clue of) and loudly proclaimed that “Your not really from Berkeley” despite my living with my mother in Berkeley since I was four years old (I still find I have to fight myself not to show my bitterness towards “UMC” people and their children to this day).
      I tested out early in the hopes that I could go to community college instead, but that’s another tale.

      I really don’t know if the fists of some of my classmates who also came from “the flats” or the disdain towards me I felt from some of the “hills kids” was more disruptive of my “education”.

      I learned more with a library card, sneaking into the University libraries and reading there, and from the math my union apprenticeship taught me than from Berkeley High School.

      A big part of me wants all my fellows who grew up in the flats to get to skip the indignity of high school and just go into trade apprenticeships.

      And the kids from the hills?

      Close the University, send the books to public libraries, have the professors teach junior high school students,  and have the hills kids pick tomatoes in the fields between Gilroy and Hollister. 

      No new Lawyers, Physicians, and Marketing?
      Have nurses be the doctors and have paralegals be the lawyers. 

      Or let me and my fellow flatlanders spend time in the palaces you call Universities.

      If you tell me “Compared to Stanford U.C. has more students from lower and ‘middle’ class backgrounds”, fine the Stanford students go to the Gilroy fields first (good lord I think I just turned myself into an American Pol Pot. I wear glasses and live in a city, the Khmer Rouge would kill me!).”

    • TracingWoodgrains says:

      It is disapproved of in high school as well. Jeannie Oakes wrote the influential book Keeping Track in the 80’s, arguing that all multi-tracked classes should be dismantled, at all levels of education. There was a big detracking push in the 90’s. The common wisdom that has persisted since then is that tracking and ability grouping are, at best, a necessary evil, and at worst, represents terrible inequity at the heart of schools–elementary, high school, all of them.

      • faoiseam says:

        all multi-tracked classes should be dismantled, at all levels of education.

        Is there any push for removing tracking in third level education? I can’t imagine what a university would be like if math classes were not tracked, and everyone in the same year got the same class.

  25. rminnema says:

    Thank you for writing this. It’s an excellent read. However, I have one quibble with a statement you made in your piece:

    Schools are the institutions in charge of educating kids.

    No, they’re not. The institution in charge of educating our kids is the parents. Our children are our responsibility, and not other peoples’. It’s true that the present culture has convinced many parents to offload their responsibilities to the school system, but I find that to be a cultural mistake. Engaged parents are the first point of education for all children. Everything flows outward from the family, and delegating those responsibilities is a bad idea. No one cares about a kid more than his parents, and they have the ability to focus one-on-one at need.

    I should clarify my biases, here: We homeschool our kids, and my wife is the primary educator for our family. In part this is value-based, but in part it has to do with exactly the topic under discussion here. My kids are sharp in some subjects. Homeschooling offers them the opportunity to work ahead when they need to be challenged. Homeschooling also offers the prime ingredient mentioned multiple times in this article: A teacher who is alive to the needs of the child, and who can go back to material when the kid doesn’t understand.

    Parents are the best and most effective educators of their children. That’s true whether they are in public, private, or home schools.

    • oqpvc says:

      This probably sounds much more confrontational than intended, but: How do you ensure that your teaching is of good quality?

      The reason I’m asking is this: I teach future teachers and often emphasize the importance of didactic reduction, i.e., dumbing the subject down to facilitate learning. For this to happen, teachers have to understand the subject matter on a significantly deeper level than the level that they teach at.

      Homeschooling is unheard of where I’m from, so I’m really interested in how this works.

    • educationrealist says:

      ” The institution in charge of educating our kids is the parents”

      Wrong. You might find it to be a cultural mistake, but parents who can’t afford to educate their children according to their own preferences are required to educate them nonetheless, and accept the state/federal system offered.

      The rest of your post–in fact, your entire premise–is restricted by range of experience.

    • Matt M says:

      The institution in charge of educating our kids is the parents. Our children are our responsibility, and not other peoples’.

      This is your own personal opinion, and I suspect it is held by a very small minority of the overall population, and that it is becoming less popular of an opinion every day.

      • educationrealist says:

        Pretty sure all one’s own opinions are personal. And it doesn’t matter how many people hold that opinion. I agree that most parents believe their children’s education is their own responsibility. But practically speaking, the federal governement is responsible for educating your child.

        Two obvious examples.

        1. If you show up in this country with no money and no English, you are required to send your child to school, where the child will learn English and other topics. You can’t choose not to.

        2. If you don’t send your kid to school without adequate substitute education, you will be prosecuted.

        You can think it’s your responsibility all you like. But if you fail, the state doesn’t shrug, because the state has primary responsibility that you can escape only by proving you have the means to provide a substitute the state considers adequate.

    • TracingWoodgrains says:

      Homeschooling is a fantastic option for those with the means, interest, and ability to do so. When I have kids of my own, it’s the most likely option for me. But most don’t have the luxury to do so, and for everyone else, schools as an institution serve a vital role, and delegating those responsibilities is a necessity. Given that reality, we should aim to refine them as much as possible.

  26. johan_larson says:

    I’m getting the impression that if your kid is a bit bored in regular classes, your best bet in most public schools is to tell them to not make a fuss in class, and encourage them to devote themselves to some intellectual pursuit outside of class. They’ll still be bored in class, but they’ll have something else they really care about.

    If your kid is really dramatically gifted, it sounds like the best bet is to pull them out of school completely and home-school using textbooks and online courses and whatnot. If they’re that talented, they should be able race on ahead. Maybe use the existing curriculum as a guide to what they should be studying, if you’re not highly educated yourself.

    (Assuming private school is financially out of the question, of course.)

    • Matt M says:

      your best bet in most public schools is to tell them to not make a fuss in class, and encourage them to devote themselves to some intellectual pursuit outside of class

      I agree that this is probably the best advice – but it’s pretty crappy advice. My parents tried this with me but it didn’t really work, because my ordinal preferences were something like.

      1. Play video games
      2. Watch syndicated reruns on TV
      3. Learn something interesting of my own choosing
      4. Be bored in school

      Given that I was forced to spend 8 hours a day on #4, when I got home, my instinct was to go for 1 or 2, not 3. As far as I can tell, most of my friends were in the same situation. We were all fairly bright and would have much rather been somewhere else learning more useful things as opposed to school. But when school was out, we weren’t going to the library or the math lab or whatever else the people on Big Bang Theory do. We went to Chris’ basement and played Halo all night.

      • JohnBuridan says:

        and drank Mountain Dew? Most of us drank Dew.

      • johan_larson says:

        I guess the question is why your parents didn’t push you harder to spend your time more productively. It sounds like you would have benefited from a parent who borrowed a chapter or two from the Dragon Mother book.

        • Matt M says:

          Maybe they should have, but my perception is that it would have backfired. I had a bit of rebellious personality, but ultimately was pretty well behaved. They outlined basically a minimum standard of “Maintain a 3.5 GPA” which I was able to achieve very easily, without doing much homework at all.

          Ultimately I think the problem is that, at that young age, I didn’t really understand the value of an education (signaling). I kind of intuitively latched onto the Caplan-esque mindset of “This is a bunch of bullshit, why should I bother?” I maintained the 3.5 so my parents wouldn’t yell at or punish me, but had no motivation to do any better, because nobody really bothered to explain to me how exactly it would benefit me to do any better.

          I suspect part of this has to do with being raised in a heavily progressive community where telling a kid “You want a good GPA so you can go to a good college so you can get rich and live a life of luxury” would have been anathema. Everyone was all about “get a GPA and go to a good college” but I never really understood why I should care about going to a “good” college. This is also the reason I didn’t do any extra-curricular.s I didn’t really see the point. Why should I want to go to Harvard? Oregon State is right over there, and I’ll be able to get in without expending much effort or detracting from my video-game playing time.

          If I could go back in time and talk to my younger self, I’d definitely encourage him to try harder and apply himself a little more. But he’d probably tell me to fuck off, he knows what he’s doing.

          • Plumber says:

            “If I could go back in time and talk to my younger self, I’d….”

            @Matt M,

            If I could go back in time and talk to my younger self, I’d…

            ….well I actually can’t think of much that would help, “It will be worse than you hope, but better than you fear” is about the best that I can think of.

            Though I suppose “Test out of High School earlier” may have helped, as would “Don’t listen to the guy telling you to work faster, that leads to injuries and working even slower”, and “Read less, and weld more”.

          • Mark Atwood says:

            I would have told my younger self: “drop out, then test out, then spend a couple of years in the library, then sue to be seated at the AP and SAT exams. They can’t physically force you back into public school.”

          • Plumber says:

            I would have told my younger self: “drop out, then test out, then spend a couple of years in the library, then sue to be seated at the AP and SAT exams. They can’t physically force you back into public school.”

            @Mark Atwood,

            I did test out (which was an easy test), and I took a “Cultural Anthropology” and a “European History” at Laney Community College in Oakland, instead of a senior year at Berkeley High School, and it was great, the classes were interesting and no one punched me!

            My ambition was to get enough credits to transfer to UC Berkeley (who’s wonderful libraries I’d been sneaking into starting in Junior High School), but after a year of academic classes (and my turning 18 years old), my mom told me “You’re not living here just to go to that ghetto school” and I had to get a job instead. 

            I did go back to Laney, but for welding, and blueprint reading classes. 

            I did get to learn some Trigonometry at a “Plumbers math” apprenticeship class from my Union, which did a far better job of teaching me mathematics in the early 2000’s than Berkeley High School did in the 1980’s.

            Community College and Apprenticeship classes are just better (in my experience), and if I was King of California I’d end high school earlier and let kids learn there instead.

  27. Something I’ve noticed going to an elite college is that people who went to public schools tended to have a much lower opinion of the average citizen than students who went to elite private high schools. So if we’re looking to promote democratic solidarity then separating out gifted students might actually be a means of accomplishing that.

    • Randy M says:

      Of course, if we’re looking to have opinion makers have an accurate view of the world…

    • Matt M says:

      I like the idea that the best way of making sure the elite are sympathetic towards the lower-classes is to keep the elite away from the lower-classes, so that they never get the chance to see how truly awful they can be.

    • Michael Watts says:

      As a low-level software engineer at eBay, I had a coworker, Valeria, who was much in demand to speak to local Hispanic students on the theme of You Too Can Succeed, where being a low-level software engineer counts as Success.

      She would give the speeches, but if you asked her about it her opinion was exactly the opposite. In her view, the problem these students had keeping them from Success was that they were so stupid.

      • AG says:

        I watched Grease recently, and Frenchy’s storyline is basically this. She’s the one go-getter out of her social group of deliberate slackers (who are Too Cool For School). Frenchy has gifts to share with the world! So she drops out of high school to attend a beauty school.

        It turns out Frenchy is so bad at it that she flunks. Some choice lyrics from the song “Beauty School Dropout”:

        Beauty school dropout,
        No graduation day for you.
        Beauty school dropout,
        Missed your mid-terms and flunked shampoo.
        Well at least you could have taken time,
        To wash and clean your clothes up.
        After spending all that dough,
        To have the doctor fix your nose up.

        You’ve got the dream but not the drive.

        Baby don’t sweat it.
        You’re not cut out to hold a job.
        Better forget it,
        Who wants their hair done by a slob?

        Sometimes they’re just dumbasses.

    • DragonMilk says:

      Academically, sure, but not generally. This may be a regional thing. I went to a top undergrad, but suburban Michigan public high school in a town with just one middle school and one high school. I was a total jerk in high school.

      For instance, a few days before graduation, some girl said to the US Government class, “I think we’ll all be successful.”
      My reply was, “Really? Why would you think that?”

      Yet I still respect my high school classmates and am quite eager to catch up with them to see how things are going – in contrast a lot of my classmates in undergrad who went to private school can’t even fathom having to live alongside folk in the Midwest.

    • eccdogg says:

      I’d say public school definitely had that effect on me.

      I went to very diverse schools both economically and racially in the Piedmont South. Prior to middle school I had fairly idealistic notions about people and my school experiences pretty much drove that out of me.

      But I was also tracked into the highest level during that time so I am not sure tracking made any difference. I still shared lunch, PE, sports (I was on the football and wrestling teams), some electives, and the halls with the lower socioeconomic kids and they were generally a huge nuisance. They were disproportionate in breaking the rules, starting fights, getting in your face over minor slights, disrupting class, etc. I do think one benefit of the experience was learning how to stick up for oneself and handle assholes.

      By the time I graduated I held most of those folks in generally low regard. My position has soften quite a bit since then, but I wonder if it doesn’t simply have to do with not having to put up with that crap day in and day out now since I seldom interact with those folks.

    • Plumber says:

      @Andrew Clough,

      I went to a large urban public high school (over 3,000 students), but I really didn’t think of most people as stupid until after I had worked retail and had spent a lot of time in traffic.

  28. Bla Bla says:

    This collaboration was far above my expectations.
    1. Are racially homogenous countries more open to tracking and ability grouping?

    2. A way to implement tracking in a more fair manner would be a sort of promotion and relegation system like those used in soccer leagues.
    Let’s say you have 100 first graders.
    You divide them in 4 classrooms, with the oldest in classroom A and the youngest in classroom D. That’s because we know that even minute age differences matter when so young. If you don’t like this age system use a ranking test.
    After a while, using math and english grades, you promote the top 25% towards the A class, 50% stay in their current class, while the bottom 25% are relegated towards D class. Repeat.
    The curricula for math and english is the same for everybody to allow continued comparison, but classroom D gets an education focused on these 2 subjects while those above study increasingly more subjects, with classroom A having the most subjects.
    For example, they study the same fifth grade math, class D in 10 math hours/week and class A in 3 math hours/week, but A also takes art history and military history (I like history)

    3. Maybe a solution for personalized learning would be to turn classrooms in locations for online learning – brick online school (BOS). Many adults work in coffee shops for a sort of companionship and accountability. After all, telecommuting doesn’t work for adults as well as it was hoped in the 90’s, so why expect it to work with easily distracted children?
    The teacher/supervisor in a brick online school makes sure that the student sticks to the app he is supposed to use during class, sets goal, advises and maintains order. Basically, what a boss does in an office environment, a teacher does in a BOS environment.
    This system would also allow students to socialize, do PE, do lab work, take part in intra and extramural sports, art, drama, proms etc.
    At the same time this BOS would still be a safe place for parents to send their children while they work and somewhere where ideologues can push their beliefs on the next generation, so there would be not much opposition, except from teachers.
    With less need for qualified teachers and the possibility of larger classrooms, labour costs would be lower while delivering the personalized education that we lost with the advent of mass education.

    4. We can now imagine an educational software that uses algorithms and machine-learning to personalize itself for each student becoming a more effective and more engaging educational tool. Use the force that gets us addicted to social media and news platforms for good.

    • educationrealist says:

      “Are racially homogenous countries more open to tracking and ability grouping?”

      Yes. Not only that, you’ll find much more tracking in racially homogeneous schools and districts, because racially diverse schools and districts are sued out of tracking.

      Your suggestion will, in racially diverse schools, result in black and Hispanic kids at the bottom, taking the same boring math and reading courses, with limited elective options, and the classes will be brutally hard to teach.

      And what software is it you think that will grow with the student’s needs, as opposed to being a boringly rote product that keeps going on and on to the next topic?

      • John Schilling says:

        That would be the Young Lady’s Illustrated Primer. Preferably a first edition, not one of the knock-offs.

        • Bugmaster says:

          I’ll try to avoid spoilers, but still: the primary difference between the main Primer and the knock-offs was not the software, but the manner of engagement with the student. The original Primer’s success was probably a unique occurrence, given the sacrifices that were involved in running it.

      • Bla Bla says:

        My suggestion will result in the most capable black and hispanic kids getting a more engaging and safer education while the others, including low performing whites, get what they are already getting. Which is what is already happening in a less formal manner with honors and AP classes vs remedial classes or public vs private schools.

        A tracking system would hopefully keep the boredom and brutality to the lowest performing class instead of having everybody suffer from it.
        Why spread the misery? It’s unethical to use good children as buffers for disruptive ones. I also doubt that the weakest students will find the curricula more boring than they already do. It is the teachers who will get bored without better students providing interaction.
        Give the teachers willing to tackle that class some extra payment and vacation days and make everyone happy.
        With large and constant population movement between different track classes you will get parents to, hopefully, dedicate energy in getting their kids promoted up the next rung. That, in turn, would also reduce political pressure.

        The biggest benefit of a software is that you are not tethered to your classmates anymore, so “going on and on to the next topic”, at your own pace, is a huge advantage for the student over “going on and on to the next topic” at a teacher-set pace which is the normal way.
        Another advantage could be personalized recommendations for optional lessons and reading materials using algorithms like youtube does. This area is going to see explosive development given the size of the market for target digital ads and content.
        Gamification was already mentioned.
        So, even if it’s just “a boringly rote product” for the teacher an educational app could be very engaging for students.

        • Matt M says:

          My suggestion will result in the most capable black and hispanic kids getting a more engaging and safer education

          But this won’t matter. People will look at your top track, notice that it’s 70% Asian, 25% white, and 5% everything else, and get you fired for your clear and obvious racism before you have the opportunity to explain why it’s actually really good for those high-performing “everything else”s.

  29. James Miller says:

    My son is off the charts in math. When he was in elementary school I convinced his school to let him do math problems I gave him and would go over with him when the rest of his class was doing math. The school didn’t really like it, but they went along in part because my son literally knew more math than his elementary school teachers.

  30. Elena Yudovina says:

    I’m a bit puzzled by the occasional but persistent references to private schools as a panacea for gifted students (not quite at the level of individual tutoring, but at least squarely on that route). Might someone expand on that argument in more detail? My personal interaction with the US private school system below, and the parts that I think might generalize out of it at the bottom.

    I moved to the US from Russia at age 12 (7th grade), went to a small private Jewish middle school for two years (not for religious reasons), and then went (on more scholarship) to a small-ish, good, private secular all-girls school for high school (grades 9-12). (This was on recommendation of middle school teachers, whose reasoning, I believe, was that given my lack of social graces, more individualized attention, a more genteel atmosphere, and strong interest in academics would be a good fit.) Note, I’d like to divorce the question of money from the question of private schooling: not that my family was ever poor by any objective standards (we moved to the US in part because my father was invited to work as an engineer), but new immigrants are hardly stinking rich.

    The middle school offered individual tutoring for English for the first few months (which may or may not have been more effective than just pairing me with someone who spoke some Russian and letting me sink or swim otherwise), and solved the thorny math issue by giving me a textbook, some assigned homework from it to make sure I actually did the work, and kicking me out into the hallway. (The school didn’t go past 8th grade, so doing more than that for math would’ve been genuinely difficult.) In high school, my school may have been more-than-usually open to letting me take more courses per year, including more APs, than they normally recommended, and was somewhat gracious about the fact that I ran out of math and French to take by senior year. On the other hand, it definitely suffered from simply Not Having Enough Students to support an ecosystem of advanced coursework or nerdy after-school activities. My younger brother attended a good public school, and I believe that by high school found that the nerd ecology there was at least as good as and probably better than what I saw in my private school.

    Note that I’m commenting specifically on middle and high school — perhaps the situation is qualitatively different in elementary schools?

    I think the observation of “private school may be better for an average good student, but at least by high school, a larger school will be better for a gifted student” might well generalize beyond my family’s case. Could someone who has brought up private schools explain in more details what benefits they had in mind? I understand that private schools can offer more flexibility within the range of options they have access to, but that range is far from unlimited.

    • JohnBuridan says:

      Great story. I liked your point about an ecosystem of advanced coursework. A critical mass is certainly necessary to achieve that.

      I have lots of experience in small private schoools, and think that a private high school which already attracts students and families of an academic bent needs about 200 HS students to support a strong nerdy ecosystem. Heck, even 140 is enough to offer 2 of each section and some electives.

      The school I work at has academic friendly benefits, because we are attracting already committed families who choose to go to a school which they know beforehand requires a lot of work. First, I established that the high school is academically challenging to prospective families and provided them the tools and support they need to make it doable, then overtime I have been able to make the program more academically rigorous. The system is self-reinforcing now, which is good. But since we don’t have a critical mass of students or resources, we have hit our ceiling.

    • TracingWoodgrains says:

      The only real requirement for a school for gifted students is that it is willing to accommodate them and help them stretch. Please correct me if I’m missing something, but I believe we only really mentioned private school once, in the case of a child who was miserable in his public school. Private and public schools have somewhat of a different set of advantages and disadvantages, but I don’t see private schools as inherently any more likely than public ones to serve advanced students well. Other factors (flexibility, presence of advanced programs, peer group) are much more important than whether the child’s school is public or private.

  31. Nietzsche says:

    An interesting piece, but the thesis is unclear. Is it: (1) there is more that public education could do to improve the education of the brightest students, or (2) there is more that public education should do to improve the education of the brightest students? (1) is an empirical claim, and the one that the authors seem to focus on. But (2) certainly does not follow from (1) and is really the vital, let’s-change-policy issue. School funding isn’t going up, so any improvements that aren’t cost neutral are going to have to come out of someone’s hide.

    I guess I have mixed feelings here. I suspect that most SSC readers are going to say “I was extra smart and bored as hell, my high school could have done a lot more to stimulate and nourish me.” My daughter has mental health issues, especially inattentive-type ADHD, and frankly public school has been a nightmare. And I have done everything—psychologists, psychiatrists, tutors, teacher meetings, 504 plans, everything. Nothing has helped in the slightest. I want resources and pedagogical changes to help her. I went to a crap high school in the middle of nowhere and got an Ivy League PhD when I was 25. Yeah, I was bored and hated HS like everyone else, but you know what? Maybe I shouldn’t have gotten more. I was going to make it anyway. My daughter is not, not without buckets of help.

    • Matt M says:

      I don’t mean to be cruel here, but how exactly do you jive your positions of “I’ve tried everything and nothing has helped” with “Clearly she needs more help.” Perhaps she can’t be helped. Or, perhaps, the cost of helping her isn’t equivalent to the cost of helping one precocious bored advanced student, but 10,000 of them.

      The utilitarian argument for allocating greater resources towards helping advanced students is that it’s a lot cheaper to help a bright kid become 10% brighter than it is to help a struggling kid improve by 10%, and that schools, as currently operated, are already assigning the overwhelming majority of their resources towards helping the less bright, while the bright get essentially ignored due to the logic you outlined above.

      • Nietzsche says:

        I know the utilitarian argument, and reject it, even assuming your make-up numbers are right. A civilized society does not write people off because they are struggling, and it is our worship of winner-take-all that has led to vast wealth disparity and shit life syndrome. I don’t know what the right balance is, but I know I’m not the kind of person that truly needed more in HS, even though I would have appreciated it.

        • Plumber says:

          @Nietzsche,

          The essay you linked to had some interesting links, thanks for that and sorry to learn that Britain is infected as well.
          The voters of California did away with a “civilized society” in 1978 (the last year my schools had toilet paper, and the latest year of publication for all but a few of the books in the school libraries, and by AN AMAZING COINCIDENCE that was the year my parents and most of my classmates parents all decided that they get divorced) because hey “do your own thing”, “if it feels good do it”, “government is the problem”, and “there is no such thing as society”.

          Civilization isn’t cheap, and the adults of that time clearly showed by their actions how little they cared about the fates of children.

          The adults of the 1940’s to the ’60’s made education better and more available (while also making a more just society, ANOTHER AMAZING COINCIDENCE!), while the adults of the 1970’s and ’80’s let it go to Hell, and I remember what my high school years in the ’80’s were like, and the weekly sounds of gunshots too well,

          But I’M NOT BITTER!

          NOPE NOT AT ALL!

          Well maybe a little.

          • baconbits9 says:

            The adults of the 1940’s to the ’60’s made education better and more available (while also making a more just society, ANOTHER AMAZING COINCIDENCE!), while the adults of the 1970’s and ’80’s let it go to Hell, and I remember what my high school years in the ’80’s were like, and the weekly sounds of gunshots too well,

            There is a bunch of tension in this post. The people going to school in the 40s-60s (particularly the 40s) were the adults ‘in charge’ in the 70s and 80s. If education was clearly better and more available in the 40s through 60s why did students who attended those schools mess things up so badly when they ended up in charge?

          • Education Hero says:

            If education was clearly better and more available in the 40s through 60s why did students who attended those schools mess things up so badly when they ended up in charge?

            Ideology

          • Plumber says:

            “….why did students who attended those schools mess things up so badly when they ended up in charge?”

            @baconbits9,

            As far as I can tell, in some ways mass education is itself to blame.

            The relative lack of births during the Great Depression and the Second World War, combined with a post war explosion in the birthrate (“the baby boom”), caused significant age based segregation. 

            Previous generations worked along side their elders in fields and factories, and that along with many people of intermediate age (older siblings, younger aunts and uncles, et cetera) created more cultural transfusion than experienced by the “baby boomers” who spent much more of their time in age segregated schools with peers, and that plus growing up in remarkable different circumstances (an older generation who grew up in times of depression and war, and a younger generation who grew up in unprecedented broad based prosperity, and with television) created the “generation gap” of legend, and the boomers had an individualist “counter-culture” in opposition to their elders.

            A draft for an unpopular war led to massive protests by those of draft age who had “student deferments” at college campuses, a widespread revulsion of those protests and the “youth culture” of the times led to the replacement of governor Edmund “Pat” Brown, who championed public works, and free college, with governor Ronald Reagan, who had a $150 fee for college enacted in 1970 for what had previously been a free education, plus he demonized students and the like then young and had tear gas used at UC Berkeley, which the wind carried to my nursery school and effected me and the other kids, at least according to my mother (I have no memory of it, but I looked it up and there was such an incident).

            Efforts to combat an economic recession in the early 1970’s and the 1973 oil embargo led to spiralling inflation, and increased property taxes on homes going up in nominal price spured a “tax payers revolt” in 1978 and the passage of “Proposition 13”, and the resulting cut-backs were obvious to me as a child (the missing toilet paper, classes with not enough chairs for each student, much fewer new books, all of which I’m still angry about forty years later).

            Age segregation led to the “baby boomers” not being properly socialized into responsible adults (the “me generation“), and who dissolved their marriages, leading to an epidemic of children from broken (yes, I’m still angry about that as well).

            By 1980, former governor Ronald Reagan campaigned for President on a ‘Government is the problem, not the solution’ slogan, the baby boomers (with their memories of the ‘gulf of Tonkin incident’, the draft, and the Watergate break-in), and their elders elected him, and on getting into office he signed massive tax cuts for corporations and the wealthy (the 1986 reforms to “save social security” meant that more Americans paid more of their income in taxes, even though total receipts were down, as the tax burden was shifted away from income taxes towards “payroll taxes”).

            By 1982 the efforts of the Federal Reserve to curb inflation resulted in an economic recession and massive unemployment, and soon beggers became very prominent (I tried to carefully choose which direction to leave school to avoid the many requests for “spare change”, but I was mostly unsuccessful).

            By 1985 “turf battles” of cocaine dealers meant that weeks in which I didn’t hear gunfire from outside while I was in my bedroom in either my mothers house in south Berkeley, or my fathers apartment in north Oakland were rare (and yes, I’m still bitter about that as well!).

            Is my rejection of both cultural and political libertarianism easier to understand? 

            When you see movies of the 1940’s and 50’s they have an “All in it together” ethos, but by now it’s been decades of “Rebel against the system” claptrap!

            The rebels won, and we live in the ruins.

        • C_B says:

          Isn’t there a coherent position somewhere between “expend all available educational resources on the least educate-able, even though it doesn’t work” and “write people off?” Involving some combination of advocating for UBI, campaigning against credentialism in fields that don’t really need college degrees, working for social safety nets, etc.?

          I’m sympathetic that society as currently set up is not doing your daughter any favors, but from your own “I have done everything…Nothing has helped in the slightest”, it sounds like throwing more educational resources at her may not be the solution you’re looking for.

          Do you have an argument for why it would be a good idea?

  32. rahien.din says:

    This essay is absolutely fantastic. I hope they are all this good!

    If I can add my own anecdata :

    I was ahead of my grade level in elementary school. I knew the answer to most questions and (being somewhat impulsive at baseline) I would try and answer any question being posed generally. This seems to have caused a problem for my peers, who were all the more reluctant to speak up in class. So, at a prent teacher conference, my teacher asked my parents to try and get me to chill, so that my classmates could actively participate. I distinctly remember them asking me, basically, to give the other kids a chance.

    I’m not bragging – a significant amount of that behavior stemmed from my lack of impulse control, and besides, bragging about elementary school is idiotic. And neither am I blaming my parents for the result of a conversation that must have been horribly difficult to navigate, maybe even a little painful. (I think of how I might attempt that conversation with my kid, and most roads lead to “Fuck it, kiddo, you should just go slay them.”)

    But as a kid, I remember coming away with the impression that being smart and exhibiting my smarts was imposing some unjust burden on my fellow students, or, was taking something from them that I had far less need of than did they. That, in some dysfunctional sense, my intelligence made me less deserving of the teacher’s attention, whatever my innocent intent.

    There is good wrapped up in that. Probably, I was receptive to that request only because I genuinely valued others and was willing to help. And I imagine my parents wanted to reinforce that good value, and they also knew I could and would take care of myself. Moreover, once I could get into a more tracked educational environment, I didn’t have so much of that problem to worry about.

    But it’s still kind of messed up. Who says that too a kid? And I feel like that concept bled over into other areas of my life, unrelated to school. To this day I am not entirely sure how to sort out its effects.

    Has anyone else had this experience?

    Can any of the educators here shed some light on it?

    • Skivverus says:

      Possible compromise position: tell them “Shhh! Spoilers!”, and have them write* their answer down so they can still prove they knew it before the teacher got it from someone else.

      *Or text, if they have a phone and those are allowed in the classroom.

    • educationrealist says:

      That’s pretty normal, except most of the time teachers address it with the child, rather than the parents. Unless the teacher had tried to address it with you first?

      A lot of kids need a chance to think. A teacher needs to care about all the kids in the room, and one kid’s urgent need to compete isn’t best serving the other kids in the room.

      I usually deal with it by saying “Hey, I know you know the answer, so from now on, I’ll only call on you directly when I have something challenging. But you’re not letting other kids get a chance to think and participate, so if I don’t call on you, don’t shout out the answer.”

      And your internalizing is a bit odd. It’s not as if the teacher is giving you a lower grade for knowing more. In fact, you were hogging every single bit of the attention by shouting out the answer first. The teacher was simply trying to create some equity by stopping you from sucking out everyone else’s chance to think.

      The problem, basically, is that you saw being publicly acknowledged as The Person With The Answer as fundamentally necessary to the act of education. Which is kind of weird, but pretty normal for hypercompetitive kids.

      • rahien.din says:

        Your experience is pretty typical for hypercompetitive kids who think the purpose of the educational system is to publicly acknowledge them as The Person With The Answer.

        I’m not sure why you internalized the way you did, though.

        I know, right? Odd. Maybe you are not responding to me, but rather, to some kid(s) whose hypercompetitiveness you resent.

        Furthermore, I’m not sure how else to demonstrate that, when faced with the information that I, selfish twit, was sucking out everyone else’s chance to think, you and I had the exact same reaction.

        • educationrealist says:

          You have it wrong. Like you, I was always answering questions and teachers would tell me to hold back. I wasn’t traumatized by it, though.

          Like any other bright kid, you didn’t see your answering as scooping up all the oxygen. Your reaction to being asked to hold back, though–the notion that if you’re not allowed to answer all the questions, you are somehow being robbed of an essential part of education, is what I thought of as odd. Not selfish. Self-centered, sure, but that’s pretty much par for the course.

          • rahien.din says:

            You believed that not being allowed to answer all the questions somehow deprived you of an essential part of education.

            This is a misunderstanding. That’s on me.

            I don’t know if other bright kids come to see their answering as scooping up all the oxygen, but that’s a good description of my reaction. I came to believe that if I answered any question, that I was depriving someone else of an essential part of education. I didn’t want to be an asshole, so I just stopped answering questions.

            I’m just like you, like any other bright kid : self-centered is par for the course.

            There’s a name for that.

          • educationrealist says:

            …you do realize I said no such thing, right?

            Like a lot of people here, your problems in school seem to have little to do with being bright.

          • rahien.din says:

            My intuition is that we are circling an instrumental belief, necessary to maximize learning in a diverse classroom. It seems like the right belief for an educator to hold. It’s at the very least a Chesterton fence, and (as you say) it is likely to be true in most circumstances.

            I perceive that it was not helpful to me specifically, but, I wouldn’t personally work to alter it. Broadly, it is probably the best open path to maximizing overall learning (and I turned out fine) and anyway my experience is not central. My specific reaction has little to do with school per se, but is merely further evidence that internalizing symptoms are difficult to recognize in children. Changing the education system is the wrong way to address that difficulty.

            Overall, this has – you have – been very helpful to me.

            I did a pretty bad job describing my problem initially, and subsequently layered in too much reaction, and both of those adversely affected discourse. I hope you’ll accept that.

  33. Thyle Dysig says:

    Well done.

    One last thing: if you’re an educator or a parent or just somebody who spends time around children, take their feelings seriously, OK? If a kid is miserable, that’s absolutely a problem that has to be solved, no matter what district policy happens to be.

    Thank you! My son is both really rambunctious and smart. He read books at 3 and by 1st grade was reading chapter books. On the first day of school he was really excited and happy. It didn’t take him long to get so bored that he would do something like tackle the boy next to him. We fought the school for a couple of years to get him accelerated and gave up by putting him in a private school. We fought that school for one year and then gave up and homeschooled him. IF his mom didn’t run her own business, and therefore could set her own hours, we wouldn’t have been able to do it.

    Fortunately, the local high school had embraced AP classes, so he was finally able to go to school (and play football and wrestle) and not be miserable. Given his combination of rambunctiousness and boredom, bad things would have happened if we had left him in school.

    So again, I say thank you.

    • TracingWoodgrains says:

      I’m glad to hear it worked out for y’all–it’s always good to hear happy endings for these situations. It takes a lot of effort sometimes to find a place where atypical kids fit, and it’s a lucky kid whose parents are willing to jump through all the hoops needed to do so. I’m glad you enjoyed the article and the final note. Thanks for sharing your story!

  34. DragonMilk says:

    If you take it as a given that public school will be a mediocre at best option for your kids, and that you would do a better job than the teachers (two big ifs, for sure), how many kids would you have before you choose to homeschool them?

    I would like to homeschool my (future) kids so long as I have at least two, but my fiancee prefers me working. I’m told that playing with kids is not the same as raising them.

    • Randy M says:

      my fiancee prefers me working

      I’m not sure that this issue is really related to how many kids you have.
      But, to answer your question, we have three homeschooled, and would probably do the same with one.

      I’m told that playing with kids is not the same as raising them.

      For various ages, it most certainly is a component of it. Play is how young children learn. For later ages, instruction takes other forms, but then, with direct individual instruction, and cutting out filler like changing classes, taking role, waiting for pick-up, homeschool lessons would be much quicker for the same instruction (either quicker in hours or quicker in days) leading to time left for play or socializing.

    • Thyle Dysig says:

      I homeschooled only one child. The other two were very social and, frankly, appreciated that school was easy, so it never occurred to us to homeschool until our last one made it all but mandatory. It was well worth it. I taught history and writing and my wife taught the rest.

      I’m told that playing with kids is not the same as raising them.

      No, I suppose not, but the psychological literature, as far as this amateur enthusiast can tell, says that playing is extremely important to a child’s physical, mental, and emotional development. So as I liked to point out to my wife, playing is a very important part of raising them.

      PS: Dragonmilk is my favorite beer! Are you familiar with it?

      • DragonMilk says:

        Haha, I have yet to try it, though many friends have posted pictures of themselves drinking it. I should tell one of them to save me one sometime.

    • marshwiggle says:

      At the moment we’re homeschooling one kid and sending the other to school. For a while we homeschooled both. For us at least it’s not really a huge difference in investment of time or money to homeschool vs public school. Anyway, the point is that homeschooling can under some circumstances be worth it with just one kid. We’ve gone with a mix of mostly free time for the kids to educate themselves plus 1-3 hours of intensive English/Math/Science/History, and not even all of that needs a parent’s full attention. Plus my wife and I have split the homeschooling jobs. So it hasn’t been a big burden.

      • baconbits9 says:

        One communication problem I have encountered when I talk about our plans to home school is the fact that many people default to thinking of home school as public school, but at home. Getting them to accept that there are numerous ways to approach education and that kids in seats ‘learning’ in 50 min segments doesn’t have to be the way things are done takes enough effort to make you want to skate around the question when it comes up.

  35. johan_larson says:

    Is there a country that really goes out of its way to identify and cultivate the intellectually talented early in life? I seem to recall some European countries have special (public!) math schools, just as they have special sports schools for talented athletes. That sort of thing got some attention in the US back in the Sputnik era, but it seems to have faded in the face of other concerns.

    • peterispaikens says:

      I’m not sure about how widespread that is, but we have a de-facto (perhaps not entirely intended/planned that way) tradition/system that’s sort of like that in Latvia; the main part being certain ‘top’ high schools that have strict entrance exams and accept the top scoring applicants (the selection generally happens at year 7, with partial “reshuffling” at year 10 for the last three years of high school), combined with a tradition of advanced students self-selecting and applying to such schools. This means that the ‘education floor level’ is quite high, that the lowest performer in such a high school is comparable to the best student in an average school, so they can afford a much more aggressive curriculum, generally covering large parts of the core university courses for the appropriate specialty by the time students finish high school, while still having the same not-that-individualized curriculum for all the students in a certain grade, and almost the same speed for all the students in a class. If someone can’t function well at that speed, which sometimes happens, then in general they get unsatisfactory grades, drop out to an ‘easier’ school and do very well there.

      The students (myself being one of them) seem to like this structure, however, the evaluation of this educational policy is mixed – it’s clear that these students achieve great results, however, it’s very hard to separate the effect of this curriculum from the self-selection effect of cherry-picking the best students from all over the country (and the related cherry-picking by being able to attract the best teachers); as far as I’m aware, there seems to be no conclusive evidence whether that practice should be facilitated or not, and the arguments for/against this closely mirror the ones discussed in this article.

      There seems to be a conceptual difference between this practice and the widespread tracking as practiced in e.g. (as far as I know) Germany, in that this sort of ‘tracking’ mostly affects a couple percent of top performers and doesn’t attempt to make a divide somewhere in between the poor and average performers.

    • eccdogg says:

      I don’t know what qualifies as early in life, but in the school system of my state and county teachers will start recommending you for special enrichment as early as kindergarten. In 3rd grade you will take what amounts to an IQ test and if you do well enough you will get special classes. You will also be eligible to skip a grade at that point in math and/or language arts if your parents want you to. Starting in 6th grade you will be eligible to go to a magnet middle school where you will only take classes with other “gifted” students in Language Arts/Social Science/Math/Science this includes the ability to be accelerated in mathematics. In high school if you qualify you can go to a special state (public) boarding school that focuses on math and science.

      My daughter is currently doing much of this in 6th grade. That is why I would answer the original question “DOES THE EDUCATION SYSTEM ADEQUATELY SERVE ADVANCED STUDENTS?” with an emphatically yes. In fact in my experience they lavish resources on bright students. They also do quite a bit to help bad students. The students that generally get a raw deal from what I can see are the so so students.

      And that is the difference between private and public schools in my area. If you are an advanced student the public schools are hard to beat. They have the scale to offer a lot to gifted students, but for your average kid you are probably better off going to private school if you can afford it.

    • Bugmaster says:

      Israel does — or it did when I was there, a long time ago. They separated middle schoolers into majors, where each major had different requirements in terms of academic rigor and focus of study; thus, tracking is built into the system. Students may be kicked out of their major due to low test scores, and yes, this does include IQ tests. Gifted students are promoted rather quickly.

  36. peterispaikens says:

    Thank you for your work on this article. I have a lot of opinions about this, but this is really solid, balanced and useful summary of important arguments about this topic.

  37. Garrett says:

    Great work!

    Follow-up questions:
    * Has there been any research on tracking-based student/teacher ratios? So instead of 3 classes of 30 students, you might have 1 class of 40 advanced students, 1 class of 30 regular students and 1 class of 20 dumb students? This would possibly distribute the chaos evenly per teacher.
    * If social skill development, etc., is supposed to be an important, is there any good classroom-like (eg. 25 students/teacher) which is effective at teaching that for students not so adept?

    • TracingWoodgrains says:

      Has there been any research on tracking-based student/teacher ratios? So instead of 3 classes of 30 students, you might have 1 class of 40 advanced students, 1 class of 30 regular students and 1 class of 20 dumb students? This would possibly distribute the chaos evenly per teacher.

      Nothing that we ran into. One form of this certainly exists in the form of special ed classes, which often have teacher-student ratios as low as 2-3:1 and can be unmanageable otherwise. If you just have one large class of advanced students, though, the more extreme outliers will see little improvement from the default situation–as we mention, tracking is effective primarily as the curriculum can adapt to the students’ levels, and something like that dilutes the level of adaptation possible. I’ll keep an eye out for research in that vein, though.

      * If social skill development, etc., is supposed to be an important, is there any good classroom-like (eg. 25 students/teacher) which is effective at teaching that for students not so adept?

      Nothing high-profile that I’ve seen. I’ve heard of programs like this one for autistic college students, but honestly this is one area where there’s a lot of work left to be done and where people are usually mostly on their own if the normal school environment doesn’t help them.

  38. morningpigeon says:

    I have some comments on acceleration. I used to be a teacher, for what it’s worth.

    For all the complication teachers want to throw at you, the fact that standardised tests work shows that the difficulty of a lot of tasks and the ability of students to complete them can be fairly well measured on a unidimensional scale.

    We have good data showing that student performance in maths and language (and fricken everything) increases by about 0.3 sd per year in middle/high school, and somewhat faster in primary school (up to around 1 sd from K-1, I think).

    Typical task difficulty is far less studied, but the fact that attrition is a bigger issue in higher grades might indicate that demands increase somewhat faster than the 0.3 sd per year that ability increases. (1 sd here is a standard deviation of ability among same-aged students)

    Accelerating a bright student by one year may not have much effect in terms of difficulty, because a bright student might be at +2 sd, and the work/context has only changed by ~0.3sd (“The dissatisfied third of those 1 in 10000 students, for the most part, reported wishing they had been offered more acceleration”). On the other hand, given demands rising faster than ability, it may mean they reach their level of challenge faster.

    Essentially no-one in education has an intuitive grasp of these basic figures, teachers & wonks alike. What’s worse, there is essentially no research on how well kids learn when there is a mismatch between difficulty and capability. I think there could be an interesting study involving, for example, intentionally fudging with a strength matching algorithm on an online chess site.

    I’m deeply skeptical of teachers who say “all bright students need are good teachers”. People saying this are usually underhandedly trying to signal that they are good teachers, while also saying that their own wisdom trumps one of the few well-established results in education literature. Here’s the thing: taking the data to be solid, and I do believe the data on acceleration, there are two ways to explain this mismatch. Either their teaching experience has not taught them to be good at predicting student learning and satisfaction, or otherwise their experience has been so non-representative as to be useless to everyone else. Given how many teachers have this opinion, there can’t be that many in the second category. This doesn’t mean they’re not good, but it means that their understanding is practical and limited in domain, and so their opinion on policy questions might be questionable.

    In any case, like everyone else, bright students typically end up with average teachers, and the research clearly shows that acceleration benefits bright students with average teachers doing average things.

  39. deciusbrutus says:

    If tracking is impermissible, but acceleration is permissible, why not set the base track to be -1sd, and accelerate most students to some degree or other?

  40. vV_Vv says:

    The authors try to emulate the writing style of Scott’s blog posts, but I would have preferred a structure resembling an academic paper. As it stands is difficult to read: what is the hypothesis being tested? what are the methods? what are the results?

    I didn’t read it all yet so maybe the answers to these questions are buried in the text, but it would have been better if they had been clearly stated each in their own section and if an abstract summarizing all the study in a few paragraphs had been included.

  41. Matt M says:

    Question for the authors:

    Has there been any research done on whether or not tracking can sometimes harm the more advanced track by under-emphasizing the basics of a topic?

    I went to a high-school that was heavily tracked, although I believe the tracking was self-selected. Through a scheduling quirk at registration, in my sophomore year, I ended up in an English class clearly designed for the very bottom track. But rather than being a negative experience, the class ended up being incredibly interesting and useful, because it started from the very beginning. Here’s what a noun is, here’s what a verb is, etc.

    While some of that was a boring and basic review, it quickly stopped becoming one. It turned out that I had been in the “advanced” track my whole life, such that everybody simply assumed I had a detailed mastery of the parts of speech. But I never really did, not explicitly at least. In my freshman English class, we didn’t spend any time on parts of speech. We were “advanced” so we just read Shakespeare because that’s how smart kids signal they’re good at English. But taking the time to review the basics, in-depth, really ended up helping me a lot more than reading the Great Gatsby would have (which is what my friends in the advanced course were doing).

    Is this a weird quirk exclusive to me? Or is there something to this?

    • arlie says:

      I don’t recall my schools ever teaching any grammar in English class. I learned about such things as parts of speech while taking Latin. This was the last year that Latin was even offered – after that the choices were slimmed down to French and Spanish, plus mandatory French.

      I learned things like names of tenses only in French – because they were used in French class, but not in English class. Thus the difference between “perfect” and “imperfect” past tenses would be entirely intuitive and unlabelled, for me, with only my English instrucion to rely upon. (To be fair, French was mandatory, so the folks planning the English curriculum may have counted on that.)

      This was in Canada, in the late 1960s and early 1970s. Tracking was considered obscene, except for French for a year or two late in high school. But Latin was optional, and so effectively ‘tracked’ – only really bright students wanted to take it. It was my favourite class, though I was furious when the provincial authorities dumbed down the final exam by making it 20% about Roman history – which we basically hadn’t bothered to cover, except incidentally.

    • pontifex says:

      If you’re already fluent in a language, does knowing the parts of speech really help? It seems like there is little or no correlation between knowing what a dipthong or a gerund is and being able to crank out a good essay. It’s kind of like the difference between being able to ride a bike and analyzing the dynamics of riding a bike in physics class.

      • Matt M says:

        Eh, I feel like learning about gerunds made me more likely to use them effectively in my own writing. And learning about the effective usage of parts of speech made it easier to analyze the writing and poetry of others – independent of the narrative content. Meaning that in my Freshman Honors class, we talked a lot about Shakespeare’s plots and symbolism and everything, but in the remedial class, we talked about how writers actually crafted their sentences in a way that made the writing effective.

        You might not need to know physics to know how to ride a bike, but it probably helps to know physics if you’re going to manufacture one.

        But to the overall point, what happened here is that if you isolate naturally bright students, most of them will be able to both analyze existing writing and write on their own fairly well somewhat intuitively, such that teaching the basics of the craft seems unnecessary. I’m just saying that in my own experience, it seemed to add value, but perhaps I am unique in that sense.

        • pontifex says:

          Writing is mostly intuitive for me. Sometimes I’ll rearrange a sentence to use active voice rather than passive, or break up a sentence that is too long. I can’t honestly say that I remember much of the rules any more. Occasionally I look something up and I’m surprised to find that something I regarded as an ancient rule is actually a recent invention.

      • Matt M says:

        I’ll also note that I wasn’t alone in this class. There were maybe 5 or us in total who were clearly much more advanced than what the class was intended for – and all of us seemed to generally enjoy the class and find it valuable.

      • arlie says:

        I think this is asking about goals in another form. What things are part of a basic education?

        If the goal is really just to bring everyone up to a (very low) baseline, as some have claimed in these responses, then reading at a 3rd grade level is probably more than enough.

        I find I react to people not knowing the meaning of “noun” or “verb” rather like I react to e.g. Americans who can’t name all the US states. Or to people who think that the King James translation is the original version of the Christian Bible – not e.g. that it’s a divinely inspired perfect translation.

        OTOH, English speakers who don’t e.g. know about the subjunctive – which is mostly gone from modern English – I kind of don’t care either way.

        But discussions of “what every person ought to know” swiftly turn into culture wars.

    • TracingWoodgrains says:

      I haven’t seen research in that vein, but it raises an educational issue that I’m pretty passionate about: understanding what students know before teaching them, and knowing the specifics of your goals for them. Right now, classes are each discrete units where a certain level of knowledge is sort of assumed, and it’s perfectly possible to pass from course to course without actually retaining much practical growth in understanding through any of them. This is, I feel, where problems like your mention of under-emphasizing the basics comes. At its core, it’s an issue with curriculum and continuity.

      Analyzing Shakespeare and knowing the basics of language composition are distinct enough skills that they both need to be taught separately. They just happen to be lumped in the general bin of “English.” And it’s entirely feasible to design curricula to teach either skill, and tests verifying how much people already know about those skills, taught at either a faster or slower pace and in greater or less depth depending on student needs. So you could, theoretically, have a high-track “basic language composition” course that taught those skills, but did so quickly and at a depth that advanced students would pick up on, or a low-track “Shakespeare analysis” course that did the reverse. Reality not matching that is less a mark on ability grouping, more a reflection of the priorities we’ve collectively landed on for each of those groups (and, of course, what ends up being practical).

      • educationrealist says:

        We’re a long way away from changing high school English from “let’s be a lit major” to two different classes: Composition AND lit analysis, alas. It’s a needed change.

        • Matt M says:

          Yeah, I really wish more time would have been spent on how to write well. Specifically, how to write well in a business setting, and how to write well for general public consumption.

          Instead, all I ever got were year after year of how to write research papers and other academia navel-gazing type instructions. It was never really made clear to me how valuable a skill being a “good writer” is in general in life.

    • dodrian says:

      I think part of this may be a comprehension problem in general (didn’t we talk reading speeds in a recent open thread, and how typical comprehension was something like 60% — I can imagine a similar case being made for teaching a class on a topic).

      I’m pretty sure I was taught parts of speech at some point in English class, probably upper-lower or early-middle school. I didn’t encounter that in English in high school, but I did learn it again in high school Spanish. It really helped me solidify my understanding of English and how to use it well – part of that came from having something else to compare it to, I’m sure a lot of it was due to re-introducing the fundamentals at a higher level, when you have a better understanding of why they’re important.

      As a software developer, it’s incredibly tempting to jump into a new programming language or framework with a difficult high-level project. Relying on my general programming knowledge and a bit [lot] of stackoverflow I can usually muddle through. But going back and performing the really basic exercises in that language will pretty much always give better long term results, and help me avoid mistakes further down the line, even if it seems like a lot of repetition of stuff I already know.

  42. Will James says:

    Couldn’t help but be reminded of a memorable moment from one of my favorite television shows from childhood, “Malcolm In the Middle”, while reading this piece,

    Mr. Woodward: I just don’t think you’d throw away the son who achieves for, well, Reese.

    Lois: You don’t think I’d sacrifice this one? Let me explain something to you. I would sell Malcolm down the river in a heartbeat to save Reese. Malcolm’s gonna be fine no matter what happens. Maybe he’ll have to go to junior college or start off blue collar, but he’ll work his way up to management eventually. Reese is the one who needs saving.

    Mr. Woodward: I don’t believe you. No mother could ever be that callous to her own son.

    [Francis appears in the window, pressed against the glass, while rain pours down and lightning flashes.]

    Francis: Mom, please let me come home! I’m cold and I’m hungry! Please, I’ll fix the roof, I’ll paint the house! I’ll do anything, Mom, please! Just let me live indoors, Mom! Please, I wanna be warm again! MOM, PLEASE! [sobbing]

    [Lois smiles victoriously. Woodward looks rather apprehensive.]

    Woodward: Maybe we can work something out.

    The curse of being the smartest kid in the classroom. Can’t say I’ve experienced it.

  43. bgulino99 says:

    The folks who don’t like ability grouping because its immoral absolutely lost me. I have no idea what that even means.

    A self evident problem is that low ability grouping must content with members who don’t have low ability but have behavioral issues that preclude them from achieving. If I had kids with low intellectual ability I would resist them being placed in a low ability group because the low ability group is more likely to have violent or otherwise badly behaved children in it then the high ability group. Curious that this is not mentioned as a problem as well as the growing tendency to not expel or transfer badly behaved children out of mainstream schools.

    • Michael Pershan says:

      I think we were going for that in this section:

      One thing that is decidedly not up for debate in the literature is that Slavin’s non-academic concerns are real. Opponents and defenders of tracking alike agree that low-track classes are often chaotic, poorly taught environments where bad behavior is endemic, and that this is a major problem.

  44. Edward Scizorhands says:

    This sounds horrible, but it really sounds like emotional support would have been more important than opportunity support.

    I went through a phase in 12th grade when I found out how many math opportunities I had missed (first time I ever heard of Putnam, despite being 1st in state several years earlier) and was resentful for a while. I felt like I had really missed out on lots of things. But I got over that, and had I stayed in that funk I would have been seriously destructive.

    (And edrealist’s “smart kids don’t know how to deal with challenges and avoid them” really strikes true for me.)

  45. Windward says:

    What about when gifted =/= STEM skills? Kids who can comprehend writing 3 or 4 grades above their level or already know how to read before preschool? (In these cases there’s one obvious argument against grade-skipping past a certain point: who wants their 9-year-old encountering the typical maturity level of AP Literature texts…) Being able to read and comprehend above your peers’ level is likely to lead to a *lot* or time spent waiting around in class for everyone else to finish reading. Also, it probably correlates to some extent with complexity of ideas exposed to (if you’re reading “up”).

  46. hnau says:

    As with all the teams, major props for doing a ton of work to produce this.

    Compared to the other entries so far, this one seems like the most thoroughly researched and documented. It has the feel of a Much More Than You Wanted to Know, which is awesome. (I would expect nothing less from someone who goes by TracingWoodgrains, of course.)

    Where I felt it came up short was that it failed to express and to some extent resolve, or at least clarify, the disagreement between the collaborators. In part this is probably due to adversarial-ness making it hard to reach bold conclusions. In part it’s because the topic is relatively broad and vague. But the other teams did a better job of specifying what claims they disagreed on and how they went about investigating those claims.

    Based on the quality of the write-up, I assume this was due to an oversight or space limitation rather than any actual flaws in the team’s approach. I might still vote for this entry due to the excellent information; I just thought that it could have been more to the point.

    • TracingWoodgrains says:

      Definitely a valid critique. Part of this was due to the goals we outlined for collaboration—we decided to research together without drawing firm “lines in the sand” first so we could more easily update our perspectives as we learned. In aiming to present a unified front, we also ended up shunting our differences to the background. If we were re-doing this, I would add brief biographies similar to the third entry and clarify our positions. As is, I’m excited to do a post-mortem on the process, specifically addressing our differences and what was and wasn’t resolved.

      • Michael Pershan says:

        I also think that, to a certain extent, this speaks to the differences between education and other fields. It’s a field with conflicting goals, it resists experimental research, it’s a much larger and more disparate profession. You start with a seemingly simple question as we did — does tracking/ability grouping work? — and quickly you get into some very messy questions like “what does it mean to work?” and “what sort of research can we agree to value in a field that fights over this stuff all the time?”

        That’s how you get from seemingly simple questions of meta-analysis score-keeping to feeling like you simply NEED to talk about the conflicting goals of education. So I think it’s hard to have clean disagreements about education, maybe harder than in some other fields like medicine.

  47. educationrealist says:

    I think all the Adversarial papers are impressive, but in terms of inspiring commenter activity, this one has been the HUGE winner thus far. Which is interesting considering that a number of commenters said “hey, this doesn’t have a clear position, can’t see the adversarial sides, etc”.

    Anyway, I just want to reiterate that you two did a terrific job and I hope you get it published or get more attention for it.